You are on page 1of 350

TABLE OF CONTENTS

PREFACE ...................................................................................................................................................... 1
Question of the Day #01: (13-Apr-09)....................................................................................................... 2
Question of the Day #02: (14-Apr-09)....................................................................................................... 2
Question of the Day #03: (15-Apr-09)....................................................................................................... 3
Question of the Day #04: (16-Apr-09)....................................................................................................... 3
Question of the Day #05: (17-Apr-09)....................................................................................................... 4
Question of the Day #06: (18-Apr-09)....................................................................................................... 4
Question of the Day #07: (19-Apr-09)....................................................................................................... 5
Question of the Day #08: (20-Apr-09)....................................................................................................... 5
Question of the Day #09: (21-Apr-09)....................................................................................................... 6
Question of the Day #10: (22-Apr-09)....................................................................................................... 6
Question of the Day #11: (23-Apr-09)....................................................................................................... 7
Question of the Day #12: (24-Apr-09)....................................................................................................... 7
Question of the Day #13: (25-Apr-09)....................................................................................................... 8
Question of the Day #14: (26-Apr-09)....................................................................................................... 8
Question of the Day #15: (27-Apr-09)....................................................................................................... 9
Question of the Day #16: (28-Apr-09)....................................................................................................... 9
Question of the Day #17: (29-Apr-09)..................................................................................................... 10
Question of the Day #18: (30-Apr-09)..................................................................................................... 10
Question of the Day #19: (01-May-09) .................................................................................................... 11
Question of the Day #20: (02-May-09) .................................................................................................... 11
Question of the Day #21: (03-May-09) .................................................................................................... 12
Question of the Day #22: (04-May-09) .................................................................................................... 12
Question of the Day #23: (05-May-09) .................................................................................................... 13
Question of the Day #24: (06-May-09) .................................................................................................... 13
Question of the Day #25: (07-May-09) .................................................................................................... 14
Question of the Day #26: (08-May-09) .................................................................................................... 14
Question of the Day #27: (09-May-09) .................................................................................................... 15
Question of the Day #28: (10-May-09) .................................................................................................... 15
Question of the Day #29: (11-May-09) .................................................................................................... 16
Question of the Day #30: (12-May-09) .................................................................................................... 17
Question of the Day #31: (13-May-09) .................................................................................................... 17
Question of the Day #32: (14-May-09) .................................................................................................... 18

Question of the Day #33: (15-May-09) .................................................................................................... 19


Question of the Day #34: (16-May-09) .................................................................................................... 19
Question of the Day #35: (17-May-09) .................................................................................................... 20
Question of the Day #36: (18-May-08) .................................................................................................... 21
Question of the Day #37: (19-May-09) .................................................................................................... 21
Question of the Day #38: (20-May-09) .................................................................................................... 22
Question of the Day #39: (21-May-09) .................................................................................................... 22
Question of the Day #40: (22-May-09) .................................................................................................... 23
Question of the Day #41: (23-May-09) .................................................................................................... 24
Question of the Day #42: (24-May-09) .................................................................................................... 25
Question of the Day #43: (25-May-09) .................................................................................................... 25
Question of the Day #44: (26-May-09) .................................................................................................... 26
Question of the Day #45: (27-May-09) .................................................................................................... 26
Question of the Day #46: (28-May-09) .................................................................................................... 27
Question of the Day #47: (29-May-09) .................................................................................................... 27
Question of the Day #48: (30-May-09) .................................................................................................... 28
Question of the Day #49: (31-May-09) .................................................................................................... 28
Question of the Day #50: (01-Jun-09) ..................................................................................................... 29
Question of the Day #51: (02-Jun-09) ..................................................................................................... 29
Question of the Day #52: (03-Jun-09) ..................................................................................................... 30
Question of the Day #53: (04-Jun-09) ..................................................................................................... 30
Question of the Day #54: (05-Jun-09) ..................................................................................................... 31
Question of the Day #55: (06-Jun-09) ..................................................................................................... 31
Question of the Day #56: (07-Jun-09) ..................................................................................................... 32
Question of the Day #57: (08-Jun-09) ..................................................................................................... 32
Question of the Day #58: (09-Jun-09) ..................................................................................................... 33
Question of the Day #59: (10-Jun-09) ..................................................................................................... 33
Question of the Day #60: (11-Jun-09) ..................................................................................................... 34
Question of the Day #61: (12-Jun-09) ..................................................................................................... 34
Question of the Day #62: (13-Jun-09) ..................................................................................................... 35
Question of the Day #63: (14-Jun-09) ..................................................................................................... 35
Question of the Day #64: (15-Jun-09) ..................................................................................................... 36
Question of the Day #65: (16-Jun-09) ..................................................................................................... 36
Question of the Day #66: (17-Jun-09) ..................................................................................................... 37

Question of the Day #67: (18-Jun-09) ..................................................................................................... 37


Question of the Day #68: (19-Jun-09) ..................................................................................................... 38
Question of the Day #69: (20-Jun-09) ..................................................................................................... 38
Question of the Day #70: (21-Jun-09) ..................................................................................................... 39
Question of the Day #71: (22-Jun-09) ..................................................................................................... 39
Question of the Day #72: (23-Jun-09) ..................................................................................................... 40
Question of the Day #73: (24-Jun-09) ..................................................................................................... 40
Question of the Day #74: (25-Jun-09) ..................................................................................................... 41
Question of the Day #75: (26-Jun-09) ..................................................................................................... 41
Question of the Day #76: (27-Jun-09) ..................................................................................................... 42
Question of the Day #77: (28-Jun-09) ..................................................................................................... 42
Question of the Day #78: (29-Jun-09) ..................................................................................................... 43
Question of the Day #79: (30-Jun-09) ..................................................................................................... 43
Question of the Day #80: (01-Jul-09) ...................................................................................................... 44
Question of the Day #81: (02-Jul-09) ...................................................................................................... 44
Question of the Day #82: (03-Jul-09) ...................................................................................................... 45
Question of the Day #83: (04-Jul-09) ...................................................................................................... 45
Question of the Day #84: (05-Jul-09) ...................................................................................................... 46
Question of the Day #85: (06-Jul-09) ...................................................................................................... 46
Question of the Day #86: (07-Jul-09) ...................................................................................................... 47
Question of the Day #87: (08-Jul-09) ...................................................................................................... 47
Question of the Day #88: (09-Jul-09) ...................................................................................................... 48
Question of the Day #89: (10-Jul-09) ...................................................................................................... 48
Question of the Day #90: (11-Jul-09) ...................................................................................................... 49
Question of the Day #91: (12-Jul-09) ...................................................................................................... 49
Question of the Day #92: (13-Jul-09) ...................................................................................................... 50
Question of the Day #93: (14-Jul-09) ...................................................................................................... 50
Question of the Day #94: (15-Jul-09) ...................................................................................................... 51
Question of the Day #95: (16-Jul-09) ...................................................................................................... 52
Question of the Day #96: (17-Jul-09) ...................................................................................................... 53
Question of the Day #97: (18-Jul-09) ...................................................................................................... 53
Question of the Day #98: (19-Jun-09) ..................................................................................................... 54
Question of the Day #99: (20-Jul-09) ...................................................................................................... 54
Question of the Day #100: (21-Jul-09) .................................................................................................... 55

Question of the Day #101: (22-Jul-09) .................................................................................................... 56


Question of the Day #102: (23-Jul-09) .................................................................................................... 57
Question of the Day #103: (24-Jul-09) .................................................................................................... 57
Question of the Day #104: (25-Jul-09) .................................................................................................... 58
Question of the Day #105: (26-Jul-09) .................................................................................................... 58
Question of the Day #106: (27-Jul-09) .................................................................................................... 59
Question of the Day #107: (28-Jul-09) .................................................................................................... 60
Question of the Day #108: (29-Jul-09) .................................................................................................... 61
Question of the Day #109: (30-Jul-09) .................................................................................................... 61
Question of the Day #110: (31-Jul-09) .................................................................................................... 62
Question of the Day #111: (01-Aug-09) .................................................................................................. 62
Question of the Day #112: (02-Aug-09) .................................................................................................. 63
Question of the Day #113: (03-Aug-09) .................................................................................................. 63
Question of the Day #114: (04-Aug-09) .................................................................................................. 64
Question of the Day #115: (05-Aug-09) .................................................................................................. 64
Question of the Day #116: (06-Aug-09) .................................................................................................. 65
Question of the Day #117: (07-Aug-09) .................................................................................................. 65
Question of the Day #118: (08-Aug-09) .................................................................................................. 66
Question of the Day #119: (09-Aug-09) .................................................................................................. 67
Question of the Day #120: (10-Aug-09) .................................................................................................. 67
Question of the Day #121: (11-Aug-09) .................................................................................................. 68
Question of the Day #122: (12-Aug-09) .................................................................................................. 68
Question of the Day #123: (13-Aug-09) .................................................................................................. 69
Question of the Day #124: (14-Aug-09) .................................................................................................. 69
Question of the Day #125: (15-Aug-09) .................................................................................................. 70
Question of the Day #126: (16-Aug-09) .................................................................................................. 70
Question of the Day #127: (17-Aug-09) .................................................................................................. 71
Question of the Day #128: (18-Aug-09) .................................................................................................. 71
Question of the Day #129: (19-Aug-09) .................................................................................................. 72
Question of the Day #130: (20-Aug-09) .................................................................................................. 72
Question of the Day #131: (21-Aug-09) .................................................................................................. 73
Question of the Day #132: (22-Aug-09) .................................................................................................. 73
Question of the Day #133: (23-Aug-09) .................................................................................................. 74
Question of the Day #134: (24-Aug-09) .................................................................................................. 74

Question of the Day #135: (25-Aug-09) .................................................................................................. 75


Question of the Day #136: (26-Aug-09) .................................................................................................. 75
Question of the Day #137: (27-Aug-09) .................................................................................................. 76
Question of the Day #138: (28-Aug-09) .................................................................................................. 76
Question of the Day #139: (29-Aug-09) .................................................................................................. 77
Question of the Day #140: (30-Aug-09) .................................................................................................. 77
Question of the Day #141: (31-Aug-09) .................................................................................................. 78
Question of the Day #142: (01-Sep-09)................................................................................................... 78
Question of the Day #143: (02-Sep-09)................................................................................................... 79
Question of the Day #144: (03-Sep-09)................................................................................................... 79
Question of the Day #145: (04-Sep-09)................................................................................................... 80
Question of the Day #146: (05-Sep-09)................................................................................................... 81
Question of the Day #147: (06-Sep-09)................................................................................................... 81
Question of the Day #148: (07-Sep-09)................................................................................................... 82
Question of the Day #149: (08-Sep-09)................................................................................................... 82
Question of the Day #150: (09-Sep-09)................................................................................................... 83
Question of the Day #151: (10-Sep-09)................................................................................................... 83
Question of the Day #152: (11-Sep-09)................................................................................................... 84
Question of the Day #153: (12-Sep-09)................................................................................................... 84
Question of the Day #154: (13-Sep-09)................................................................................................... 84
Question of the Day #155: (14-Sep-09)................................................................................................... 84
Question of the Day #156 (15-Sep-09).................................................................................................... 85
Question of the Day #157: (16-Sep-09)................................................................................................... 85
Question of the Day #158: (17-Sep-09)................................................................................................... 86
Question of the Day #159: (18-Sep-09)................................................................................................... 86
Question of the Day #160: (19-Sep-09)................................................................................................... 87
Question of the Day #161: (20-Sep-09)................................................................................................... 87
Question of the Day #162: (21-Sep-09)................................................................................................... 88
Question of the Day #163: (22-Sep-09)................................................................................................... 88
Question of the Day #164: (23-Sep-09)................................................................................................... 89
Question of the Day #165: (24-Sep-09)................................................................................................... 89
Question of the Day #166: (25-Sep-09)................................................................................................... 90
Question of the Day #167: (26-Sep-09)................................................................................................... 90
Question of the Day #168: (27-Sep-09)................................................................................................... 91

Question of the Day #169: (28-Sep-09)................................................................................................... 91


Question of the Day #170: (29-Sep-09)................................................................................................... 92
Question of the Day #171: (30-Sep-09)................................................................................................... 92
Question of the Day #172: (01-Oct-09) ................................................................................................... 92
Question of the Day #173: (02-Oct-09) ................................................................................................... 93
Question of the Day #174: (03-Oct-09) ................................................................................................... 93
Question of the Day #175: (04-Oct-09) ................................................................................................... 94
Question of the Day #176: (05-Oct-09) ................................................................................................... 94
Question of the Day #177: (06-Oct-09) ................................................................................................... 95
Question of the Day #178: (07-Oct-09) ................................................................................................... 95
Question of the Day #179: (08-Oct-09) ................................................................................................... 96
Question of the Day #180: (09-Oct-09) ................................................................................................... 96
Question of the Day #181: (10-Oct-09) ................................................................................................... 97
Question of the Day #182: (11-Oct-09) ................................................................................................... 97
Question of the Day #183: (12-Oct-09) ................................................................................................... 98
Question of the Day #184: (13-Oct-09) ................................................................................................... 98
Question of the Day #185: (14-Oct-09) ................................................................................................... 98
Question of the Day #186: (15-Oct-09) ................................................................................................... 99
Question of the Day #187: (16-Oct-09) ................................................................................................... 99
Question of the Day #188: (17-Oct-09) ................................................................................................. 100
Question of the Day #189: (18-Oct-09) ................................................................................................. 100
Question of the Day #190: (19-Oct-09) ................................................................................................. 101
Question of the Day #191: (20-Oct-09) ................................................................................................. 101
Question of the Day #192: (21-Oct-09) ................................................................................................. 102
Question of the Day #193: (22-oct-09).................................................................................................. 103
Question of the Day #194: (23-Oct-09) ................................................................................................. 103
Question of the Day #195: (24-Oct-09) ................................................................................................. 104
Question of the Day #196: (25-Oct-09) ................................................................................................. 104
Question of the Day #197: (26-Oct-09) ................................................................................................. 105
Question of the Day #198: (27-Oct-09) ................................................................................................. 105
Question of the Day #199: (28-Oct-09) ................................................................................................. 105
Question of the Day #200: (29-Oct-09) ................................................................................................. 106
Question of the Day #201: (30-Oct-09) ................................................................................................. 106
Question of the Day #202: (31-Oct-09) ................................................................................................. 106

Question of the Day #203: (01-Nov-09) ................................................................................................ 107


Question of the Day #204: (02-Nov-09) ................................................................................................ 107
Question of the Day #205: (03-Nov-09) ................................................................................................ 107
Question of the Day #206: (04-Nov-09) ................................................................................................ 108
Question of the Day #207: (05-Nov-09) ................................................................................................ 108
Question of the Day #208: (06-Nov-09) ................................................................................................ 108
Question of the Day #209: (07-Nov-09) ................................................................................................ 109
Question of the Day #210: (08-Nov-09) ................................................................................................ 109
Question of the Day #211: (09-Nov-09) ................................................................................................ 110
Question of the Day #212: (10-Nov-09) ................................................................................................ 110
Question of the Day #213: (11-Nov-09) ................................................................................................ 111
Question of the Day #214: (12-Nov-09) ................................................................................................ 111
Question of the Day #215: (13-Nov-09) ................................................................................................ 112
Question of the Day #216: (14-Nov-09) ................................................................................................ 112
Question of the Day #217: (15-Nov-09) ................................................................................................ 113
Question of the Day #218: (16-Nov-09) ................................................................................................ 113
Question of the Day #219: (17-Nov-09) ................................................................................................ 113
Question of the Day #220: (18-Nov-09) ................................................................................................ 114
Question of the Day #221: (19-Nov-09) ................................................................................................ 114
Question of the Day #222: (20-Nov-09) ................................................................................................ 115
Question of the Day #223: (21-Nov-09) ................................................................................................ 115
Question of the Day #224: (22-Nov-09) ................................................................................................ 116
Question of the Day #225: (23-Nov-09) ................................................................................................ 116
Question of the Day #226: (24-Nov-09) ................................................................................................ 117
Question of the Day #227: (25-Nov-09) ................................................................................................ 117
Question of the Day #228: (26-Nov-09) ................................................................................................ 118
Question of the Day #229: (27-Nov-09) ................................................................................................ 118
Question of the Day #230: (28-Nov-09) ................................................................................................ 119
Question of the Day #231: (29-Nov-09) ................................................................................................ 119
Question of the Day #232: (30-Nov-09) ................................................................................................ 120
Question of the Day #233: (01-Dec-09) ................................................................................................ 120
Question of the Day #234: (02-Dec-09) ................................................................................................ 121
Question of the Day #235: (03-Dec-09) ................................................................................................ 121
Question of the Day #236: (04-Dec-09) ................................................................................................ 122

Question of the Day #237: (05-Dec-09) ................................................................................................ 122


Question of the Day #238: (06-Dec-09) ................................................................................................ 123
Question of the Day #239: (07-Dec-09) ................................................................................................ 123
Question of the Day #240: (08-Dec-09) ................................................................................................ 124
Question of the Day #241: (09-Dec-09) ................................................................................................ 124
Question of the Day #242: (10-Dec-09) ................................................................................................ 125
Question of the Day #243: (11-Dec-09) ................................................................................................ 125
Question of the Day #244: (12-Dec-09) ................................................................................................ 125
Question of the Day #245: (13-Dec-09) ................................................................................................ 126
Question of the Day #246: (14-Dec-09) ................................................................................................ 126
Question of the Day #247: (15-Dec-09) ................................................................................................ 127
Question of the Day #248: (16-Dec-09) ................................................................................................ 127
Question of the Day #249: (17-Dec-09) ................................................................................................ 128
Question of the Day #250: (18-Dec-09) ................................................................................................ 128
Question of the Day #251: (19-Dec-09) ................................................................................................ 129
Question of the Day #252: (20-Dec-09) ................................................................................................ 129
Question of the Day #253: (21-Dec-09) ................................................................................................ 129
Question of the Day #254: (22-Dec-09) ................................................................................................ 130
Question of the Day #255: (23-Dec-09) ................................................................................................ 130
Question of the Day #256: (24-Dec-09) ................................................................................................ 131
Question of the Day #257: (25-Dec-09) ................................................................................................ 131
Question of the Day #258: (26-Dec-09) ................................................................................................ 132
Question of the Day #259: (27-Dec-09) ................................................................................................ 132
Question of the Day #260: (28-Dec-09) ................................................................................................ 133
Question of the Day #261: (29-Dec-09) ................................................................................................ 133
Question of the Day #262: (30-Dec-09) ................................................................................................ 134
Question of the Day #263: (31-Dec-09) ................................................................................................ 134
Question of the Day #264: (01-Jan-10) ................................................................................................. 134
Question of the Day #265: (02-Jan-10) ................................................................................................. 135
Question of the Day #266: (03-Jan-10) ................................................................................................. 135
Question of the Day #267: (04-Jan-10) ................................................................................................. 135
Question of the Day #268: (05-Jan-10) ................................................................................................. 136
Question of the Day #269: (06-Jan-10) ................................................................................................. 136
Question of the Day #270: (07-Jan-10) ................................................................................................. 136

Question of the Day #271: (08-Jan-10) ................................................................................................. 137


Question of the Day #272: (09-Jan-10) ................................................................................................. 137
Question of the Day #273: (10-Jan-10) ................................................................................................. 137
Question of the Day #274: (11-Jan-10) ................................................................................................. 138
Question of the Day #275: (12-Jan-10) ................................................................................................. 138
Question of the Day #276: (13-Jan-10) ................................................................................................. 138
Question of the Day #277: (14-Jan-10) ................................................................................................. 138
Question of the Day #278: (15-Jan-10) ................................................................................................. 139
Question of the Day #279: (16-Jan-10) ................................................................................................. 139
Question of the Day #280: (17-Jan-10) ................................................................................................. 139
Question of the Day #281: (18-Jan-10) ................................................................................................. 140
Question of the Day #282: (19-Jan-10) ................................................................................................. 140
Question of the Day #283: (20-Jan-10) ................................................................................................. 141
Question of the Day #284: (21-Jan-10) ................................................................................................. 141
Question of the Day #285: (22-Jan-10) ................................................................................................. 142
Question of the Day #286: (23-Jan-10) ................................................................................................. 142
Question of the Day #287: (24-Jan-10) ................................................................................................. 142
Question of the Day #288: (25-Jan-10) ................................................................................................. 143
Question of the Day #289: (26-Jan-10) ................................................................................................. 143
Question of the Day #290: (27-Jan-10) ................................................................................................. 144
Question of the Day #291: (28-Jan-10) ................................................................................................. 144
Question of the Day #292: (29-Jan-10) ................................................................................................. 144
Question of the Day #293: (30-Jan-10) ................................................................................................. 145
Question of the Day #294: (31-Jan-10) ................................................................................................. 145
Question of the Day #295: (01-Feb-10) ................................................................................................ 146
Question of the Day #296: (02-Feb-10) ................................................................................................ 146
Question of the Day #297: (03-Feb-10) ................................................................................................ 146
Question of the Day #298: (04-Feb-10) ................................................................................................ 147
Question of the Day #299: (05-Feb-10) ................................................................................................ 147
Question of the Day #300: (06-Feb-10) ................................................................................................ 147

SOLUTIONS .................................................................................. 148


Solution #01: (13-Apr-09) ...................................................................................................................... 149
Solution #02: (14-Apr-09) ...................................................................................................................... 150
Solution #03: (15-Apr-09) ...................................................................................................................... 151

Solution #04: (16-Apr-09) ...................................................................................................................... 152


Solution #05: (17-Apr-09) ...................................................................................................................... 152
Solution #06: (18-Apr-09) ...................................................................................................................... 153
Solution #07: (19-Apr-09) ...................................................................................................................... 153
Solution #08: (20-Apr-09) ...................................................................................................................... 154
Solution #09: (21-Apr-09) ...................................................................................................................... 154
Solution #10: (22-Apr-09) ...................................................................................................................... 156
Solution #11: (23-Apr-09) ...................................................................................................................... 157
Solution #12: (24-Apr-09) ...................................................................................................................... 157
Solution #13: (25-Apr-09) ...................................................................................................................... 157
Solution #14: (26-Apr-09) ...................................................................................................................... 158
Solution #15: (27-Apr-09) ...................................................................................................................... 158
Solution #16: (28-Apr-09) ...................................................................................................................... 158
Solution #17: (29-Apr-09) ...................................................................................................................... 159
Solution #18: (30-Apr-09) ...................................................................................................................... 159
Solution #19: (01-May-09) ..................................................................................................................... 160
Solution #20: (02-May-09) ..................................................................................................................... 161
Solution #21: (03-May-09) ..................................................................................................................... 161
Solution #22: (04-May-09) ..................................................................................................................... 162
Solution #23: (05-May-09) ..................................................................................................................... 162
Solution #24: (06-May-09) ..................................................................................................................... 163
Solution #25: (07-May-09) ..................................................................................................................... 163
Solution #26: (08-May-09) ..................................................................................................................... 164
Solution #27: (09-May-09) ..................................................................................................................... 164
Solution #28: (10-May-09) ..................................................................................................................... 165
Solution #29: (11-May-09) ..................................................................................................................... 165
Solution #30: (12-May-09) ..................................................................................................................... 166
Solution #31: (13-May-09) ..................................................................................................................... 167
Solution #32: (14-May-09) ..................................................................................................................... 168
Solution #33: (15-May-09) ..................................................................................................................... 169
Solution #34: (16-May-09) ..................................................................................................................... 170
Solution #35: (17-May-09) ..................................................................................................................... 170
Solution #36: (18-May-09) ..................................................................................................................... 171
Solution #37: (19-May-09) ..................................................................................................................... 172

Solution #38: (20-May-09) ..................................................................................................................... 173


Solution #39: (21-May-09) ..................................................................................................................... 173
Solution #40: (22-May-09) ..................................................................................................................... 174
Solution #41: (23-May-09) ..................................................................................................................... 174
Solution #42: (24-May-09) ..................................................................................................................... 175
Solution #43: (25-May-09) ..................................................................................................................... 175
Solution #44: (26-May-09) ..................................................................................................................... 176
Solution #45: (27-May-09) ..................................................................................................................... 177
Solution #46: (28-May-09) ..................................................................................................................... 177
Solution #47: (29-May-09) ..................................................................................................................... 178
Solution #48: (30-May-09) ..................................................................................................................... 178
Solution #49: (31-May-09) ..................................................................................................................... 179
Solution #50: (01-Jun-09) ...................................................................................................................... 180
Solution #51: (02-Jun-09) ...................................................................................................................... 180
Solution #52: (03-Jun-09) ...................................................................................................................... 181
Solution #53: (04-Jun-09) ...................................................................................................................... 181
Solution #54: (05-Jun-09) ...................................................................................................................... 182
Solution #55: (06-Jun-09) ...................................................................................................................... 182
Solution #56: (07-Jun-09) ...................................................................................................................... 183
Solution #58: (09-Jun-09) ...................................................................................................................... 183
Solution #59: (10-Jun-09) ...................................................................................................................... 184
Solution #60: (11-Jun-09) ...................................................................................................................... 184
Solution #61: (12-Jun-09) ...................................................................................................................... 185
Solution #62: (13-Jun-09) ...................................................................................................................... 185
Solution #63: (14-Jun-09) ...................................................................................................................... 186
Solution #64: (15-Jun-09) ...................................................................................................................... 187
Solution #65: (16-Jun-09) ...................................................................................................................... 187
Solution #66: (17-Jun-09) ...................................................................................................................... 188
Solution #67: (18-Jun-09) ...................................................................................................................... 188
Solution #68: (19-Jun-09) ...................................................................................................................... 188
Solution #69: (20-Jun-09) ...................................................................................................................... 189
Solution #70: (21-Jun-09) ...................................................................................................................... 189
Solution #71: (22-Jun-09) ...................................................................................................................... 190
Solution #72: (23-Jun-09) ...................................................................................................................... 191

Solution #73: (24-Jun-09) ...................................................................................................................... 191


Solution #74: (25-Jun-09) ...................................................................................................................... 193
Solution #75: (26-Jun-09) ...................................................................................................................... 194
Solution #76: (27-Jun-09) ...................................................................................................................... 195
Solution #77: (28-Jun-09) ...................................................................................................................... 195
Solution #78: (29-Jun-09) ...................................................................................................................... 196
Solution #79: (30-Jun-09) ...................................................................................................................... 196
Solution #80: (01-Jul-09)........................................................................................................................ 197
Solution #81: (02-Jul-09)........................................................................................................................ 197
Solution #82: (03-Jul-09)........................................................................................................................ 197
Solution #83: (04-Jul-09)........................................................................................................................ 198
Solution #84: (05-Jul-09)........................................................................................................................ 199
Solution #85: (06-Jul-09)........................................................................................................................ 199
Solution #86: (07-Jul-09)........................................................................................................................ 200
Solution #87: (08-Jul-09)........................................................................................................................ 200
Solution #88: (09-Jul-09)........................................................................................................................ 202
Solution #89: (10-Jul-09)........................................................................................................................ 202
Solution #90: (11-Jul-09)........................................................................................................................ 203
Solution #91: (12-Jul-09)........................................................................................................................ 203
Solution #92: (13-Jul-09)........................................................................................................................ 204
Solution #93: (14-Jul-09)........................................................................................................................ 204
Solution #94: (15-Jul-09)........................................................................................................................ 205
Solution #95: (16-Jul-09)........................................................................................................................ 205
Solution #96: (17-Jul-09)........................................................................................................................ 206
Solution #97: (18-Jul-09)........................................................................................................................ 206
Solution #98: (19-Jul-09)........................................................................................................................ 207
Solution #99: (20-Jul-09)........................................................................................................................ 208
Solution #100: (21-Jul-09) ..................................................................................................................... 208
Solution #101: (22-Jul-09) ..................................................................................................................... 209
Solution #102: (23-Jul-09) ..................................................................................................................... 210
Solution #103: (24-Jul-09) ..................................................................................................................... 211
Solution #104: (25-Jul-09) ..................................................................................................................... 212
Solution #105: (26-Jul-09) ..................................................................................................................... 212
Solution #106: (27-Jul-09) ..................................................................................................................... 213

Solution #107: (28-Jul-09) ..................................................................................................................... 213


Solution #108: (29-Jul-09) ..................................................................................................................... 216
Solution #109: (30-Jul-09) ..................................................................................................................... 216
Solution #110: (31-Jul-09) ..................................................................................................................... 217
Solution #111: (01-Aug-09) ................................................................................................................... 218
Solution #112: (02-Aug-09) ................................................................................................................... 218
Solution #113: (03-Aug-09) ................................................................................................................... 219
Solution #114: (04-Aug-09) ................................................................................................................... 219
Solution #115: (05-Aug-09) ................................................................................................................... 220
Solution #116: (06-Aug-09) ................................................................................................................... 220
Solution #117: (07-Aug-09) ................................................................................................................... 221
Solution #118: (08-Aug-09) ................................................................................................................... 221
Solution #119: (09-Aug-09) ................................................................................................................... 222
Solution #120: (10-Aug-09) ................................................................................................................... 222
Solution #121: (11-Aug-09) ................................................................................................................... 223
Solution #122: (12-Aug-09) ................................................................................................................... 223
Solution #123: (13-Aug-09) ................................................................................................................... 224
Solution #124: (14-Aug-09) ................................................................................................................... 224
Solution #125: (15-Aug-09) ................................................................................................................... 225
Solution #126: (16-Aug-09) ................................................................................................................... 225
Solution #127: (17-Aug-09) ................................................................................................................... 225
Solution #128: (18-Aug-09) ................................................................................................................... 226
Solution #129: (19-Aug-09) ................................................................................................................... 226
Solution #130: (20-Aug-09) ................................................................................................................... 227
Solution #131: (21-Aug-09) ................................................................................................................... 227
Solution #132: (22-Aug-09) ................................................................................................................... 228
Solution #133: (23-Aug-09) ................................................................................................................... 228
Solution #134: (24-Aug-09) ................................................................................................................... 229
Solution #135: (25-Aug-09) ................................................................................................................... 229
Solution #136: (26-Aug-09) ................................................................................................................... 230
Solution #137: (27-Aug-09) ................................................................................................................... 231
Solution #138: (28-Aug-09) ................................................................................................................... 232
Solution #139: (29-Aug-09) ................................................................................................................... 232
Solution #140: (30-Aug-09) ................................................................................................................... 233

Solution #141: (31-Aug-09) ................................................................................................................... 233


Solution #142: (01-Sep-09) .................................................................................................................... 235
Solution #143: (02-Sep-09) .................................................................................................................... 235
Solution #144 (03-Sep-09) ..................................................................................................................... 236
Solution #145: (04-Sep-09) .................................................................................................................... 236
Solution #146: (05-Sep-09) .................................................................................................................... 237
Solution #147: (06-Sep-09) .................................................................................................................... 237
Solution #148: (07-Sep-09) .................................................................................................................... 238
Solution #149: (08-Sep-09) .................................................................................................................... 239
Solution #150: (09-Sep-09) .................................................................................................................... 240
Solution #151: (10-Sep-09) .................................................................................................................... 240
Solution #152: (11-Sep-09) .................................................................................................................... 241
Solution #153: (12-Sep-09) .................................................................................................................... 242
Solution #154: (13-Sep-09) .................................................................................................................... 242
Solution #155: (14-Sep-09) .................................................................................................................... 243
Solution #156: (15-Sep-09) .................................................................................................................... 245
Solution #157: (16-Sep-09) .................................................................................................................... 246
Solution #158: (17-Sep-09) .................................................................................................................... 247
Solution #159: (18-Sep-09) .................................................................................................................... 248
Solution #160: (19-Sep-09) .................................................................................................................... 248
Solution #161: (20-Sep-09) .................................................................................................................... 249
Solution #162: (21-Sep-09) .................................................................................................................... 249
Solution #163: (22-Sep-09) .................................................................................................................... 250
Solution #164: (23-Sep-09) .................................................................................................................... 250
Solution #165: (24-Sep-09) .................................................................................................................... 251
Solution #166: (25-Sep-09) .................................................................................................................... 251
Solution #167: (26-Sep-09) .................................................................................................................... 252
Solution #168: (27-Sep-09) .................................................................................................................... 252
Solution #169: (28-Sep-09) .................................................................................................................... 253
Solution #170: (29-Sep-09) .................................................................................................................... 254
Solution #171: (30-Sep-09) .................................................................................................................... 254
Solution #172: (01-Oct-09) .................................................................................................................... 255
Solution #173: (02-Oct-09) .................................................................................................................... 256
Solution #174: (03-Oct-09) .................................................................................................................... 256

Solution #175: (04-Oct-09) .................................................................................................................... 257


Solution #176: (05-Oct-09) .................................................................................................................... 257
Solution #177: (06-Oct-09) .................................................................................................................... 258
Solution #178: (07-Oct-09) .................................................................................................................... 259
Solution #179: (08-Oct-09) .................................................................................................................... 260
Solution #180: (09-Oct-09) .................................................................................................................... 261
Solution #181: (10-Oct-09) .................................................................................................................... 262
Solution #182: (11-Oct-09) .................................................................................................................... 262
Solution #183: (12-Oct-09) .................................................................................................................... 263
Solution #184: (13-Oct-09) .................................................................................................................... 263
Solution #185: (14-Oct-09) .................................................................................................................... 264
Solution #186: (15-Oct-09) .................................................................................................................... 264
Solution #187: (16-Oct-09) .................................................................................................................... 265
Solution #188: (17-Oct-09) .................................................................................................................... 265
Solution #189: (18-Oct-09) .................................................................................................................... 266
Solution #190: (19-Oct-09) .................................................................................................................... 266
Solution #191: (20-Oct-09) .................................................................................................................... 267
Solution #192: (21-Oct-09) .................................................................................................................... 267
Solution #193: (22-Oct-09) .................................................................................................................... 268
Solution #194 (23-Oct-09) ..................................................................................................................... 268
Solution #195: (24-Oct-09) .................................................................................................................... 269
Solution #196: (25-Oct-09) .................................................................................................................... 270
Solution #197: (26-Oct-09) .................................................................................................................... 270
Solution #198: (27-Oct-09) .................................................................................................................... 272
Solution #199: (28-Oct-09) .................................................................................................................... 272
Solution #200: (29-Oct-09) .................................................................................................................... 273
Solution #201: (30-Oct-09) .................................................................................................................... 273
Solution #202: (31-Oct-09) .................................................................................................................... 274
Solution #203: (01-Nov-09) ................................................................................................................... 274
Solution #204: (02-Nov-09) ................................................................................................................... 275
Solution #205: (03-Nov-09) ................................................................................................................... 275
Solution #206: (04-Nov-09) ................................................................................................................... 276
Solution #207: (05-Nov-09) ................................................................................................................... 276
Solution #208: (06-Nov-09) ................................................................................................................... 278

Solution #209: (07-Nov-09) ................................................................................................................... 278


Solution #210: (08-Nov-09) ................................................................................................................... 280
Solution #211: (09-Nov-09) ................................................................................................................... 280
Solution #212: (10-Nov-09) ................................................................................................................... 281
Solution #213: (11-Nov-09) ................................................................................................................... 282
Solution #214: (12-Nov-09) ................................................................................................................... 283
Solution #215: (13-Nov-09) ................................................................................................................... 283
Solution #216: (14-Nov-09) ................................................................................................................... 285
Solution #217: (15-Nov-09) ................................................................................................................... 285
Solution #218: (16-Nov-09) ................................................................................................................... 285
Solution #219: (17-Nov-09) ................................................................................................................... 286
Solution #220: (18-Nov-09) ................................................................................................................... 287
Solution #221: (19-Nov-09) ................................................................................................................... 287
Solution #222: (20-Nov-09) ................................................................................................................... 287
Solution #223: (21-Nov-09) ................................................................................................................... 288
Solution #224: (22-Nov-09) ................................................................................................................... 288
Solution #225: (23-Nov-09) ................................................................................................................... 289
Solution #226: (24-Nov-09) ................................................................................................................... 290
Solution #227: (25-Nov-09) ................................................................................................................... 290
Solution #228: (26-Nov-09) ................................................................................................................... 291
Solution #229: (27-Nov-09) ................................................................................................................... 292
Solution #230: (28-Nov-09) ................................................................................................................... 293
Solution #231: (29-Nov-09) ................................................................................................................... 293
Solution #232: (30-Nov-09) ................................................................................................................... 293
Solution #233: (01-Dec-09) ................................................................................................................... 294
Solution #234: (02-Dec-09) ................................................................................................................... 294
Solution #235: (03-Dec-09) ................................................................................................................... 295
Solution #236: (04-Dec-09) ................................................................................................................... 296
Solution #237: (05-Dec-09) ................................................................................................................... 296
Solution #238: (06-Dec-09) ................................................................................................................... 297
Solution #239: (07-Dec-09) ................................................................................................................... 297
Solution #240: (08-Dec-09) ................................................................................................................... 298
Solution #241: (09-Dec-09) ................................................................................................................... 299
Solution #242: (10-Dec-09) ................................................................................................................... 299

Solution #243 (11-Dec-09)..................................................................................................................... 300


Solution #244: (12-Dec-09) ................................................................................................................... 301
Solution #245: (13-Dec-09) ................................................................................................................... 301
Solution #246: (14-Dec-09) ................................................................................................................... 301
Solution #247: (15-Dec-09) ................................................................................................................... 302
Solution #248: (16-Dec-09) ................................................................................................................... 302
Solution #249: (17-Dec-09) ................................................................................................................... 303
Solution #250: (18-Dec-09) ................................................................................................................... 303
Solution #251: (19-Dec-09) ................................................................................................................... 304
Solution #252: (20-Dec-09) ................................................................................................................... 305
Solution #253: (21-Dec-09) ................................................................................................................... 305
Solution #254: (22-Dec-09) ................................................................................................................... 306
Solution #255: (23-Dec-09) ................................................................................................................... 306
Solution #256: (24-Dec-09) ................................................................................................................... 307
Solution #257: (25-Dec-09) ................................................................................................................... 307
Solution #258: (26-Dec-09) ................................................................................................................... 308
Solution #259: (27-Dec-09) ................................................................................................................... 308
Solution #260: (28-Dec-09) ................................................................................................................... 308
Solution #261: (29-Dec-09) ................................................................................................................... 309
Solution #262: (30-Dec-09) ................................................................................................................... 310
Solution #263: (31-Dec-09) ................................................................................................................... 311
Solution #264: (01-Jan-10) .................................................................................................................... 311
Solution #265: (02-Jan-10) .................................................................................................................... 312
Solution #266: (03-Jan-10) .................................................................................................................... 312
Solution #267: (04-Jan-10) .................................................................................................................... 313
Solution #268: (05-Jan-10) .................................................................................................................... 313
Solution #269: (06-Jan-10) .................................................................................................................... 314
Solution #270: (07-Jan-10) .................................................................................................................... 314
Solution #271: (08-Jan-10) .................................................................................................................... 315
Solution #272: (09-Jan-10) .................................................................................................................... 316
Solution #273: (10-Jan-10) .................................................................................................................... 316
Solution #274: (11-Jan-10) .................................................................................................................... 317
Solution #275: (12-Jan-10) .................................................................................................................... 317
Solution #276: (13-Jan-10) .................................................................................................................... 318

Solution #277: (14-Jan-10) .................................................................................................................... 318


Solution #278: (15-Jan-10) .................................................................................................................... 319
Solution #279: (16-Jan-10) .................................................................................................................... 319
Solution #280: (17-Jan-10) .................................................................................................................... 320
Solution #281: (18-Jan-10) .................................................................................................................... 320
Solution #282: (19-Jan-10) .................................................................................................................... 321
Solution #283: (20-Jan-10) .................................................................................................................... 321
Solution #284: (21-Jan-10) .................................................................................................................... 322
Solution #285: (22-Jan-10) .................................................................................................................... 322
Solution #286: (23-Jan-10) .................................................................................................................... 322
Solution #287: (24-Jan-10) .................................................................................................................... 323
Solution #288: (25-Jan-10) .................................................................................................................... 324
Solution #289: (26-Jan-10) .................................................................................................................... 324
Solution #290: (27-Jan-10) .................................................................................................................... 325
Solution #291: (28-Jan-10) .................................................................................................................... 325
Solution #292: (29-Jan-10) .................................................................................................................... 325
Solution #293: (30-Jan-10) .................................................................................................................... 326
Solution #294: (31-Jan-10) .................................................................................................................... 326
Solution #295: (01-Feb-10).................................................................................................................... 327
Solution #296: (02-Feb-10).................................................................................................................... 328
Solution #297: (03-Feb-10).................................................................................................................... 328
Solution #298: (04-Feb-10).................................................................................................................... 329
Solution #299: (05-Feb-10).................................................................................................................... 329
Solution #300: (06-Feb-10).................................................................................................................... 330
About TestFunda.com ............................................................................................................................. 331

PREFACE
For the past couple of years, CAT and other MBA entrance exams have shown a trend towards
questions testing a students ability to apply Mathematical Principles and Analytical Reasoning to
solve problems. The unpredictable nature of CAT has ensured that most students are never fully
prepared to ace the exam. This is because students limit their preparation to just the learning and
practice of core concepts of Mathematics, Verbal Ability and Data Interpretation & Logical
Reasoning.
This book is a compilation of the questions with a difficulty level typically on par with CAT. Every
single question is original and unique, created by our dedicated team of subject matter experts.
The questions are designed to give our readers greater exposure to the types of questions that
appear in CAT. The detailed solutions in this book may also provide alternate strategies and
shortcuts to solve problems. This book will give students that extra edge and confidence needed to
be ready for any surprise that CAT might throw their way.
This book is the 7th in a series of books on the Question of the Day featured on the TestFunda site.
We are sure that our readers will benefit greatly from these books.

The LEADER in Online Test Prep

Question of the Day #01: (13-Apr-09)


A bridge 'AB' has a single railway track spanning its entire width. A cat is standing on the bridge 20 m
away from the centre of the bridge. The cat sees a train coming at a constant speed of 72 km/hr. The
distance of the train (assumed as a point object) from the nearer end (point A) of the bridge is twice
the length of the bridge. If the cat runs towards the train with a constant speed, it will get off the
bridge to safety when the train is still 50 m from the bridge. If it runs away from the train at the
same constant speed, the train will hit it when it is still 12.5 m from the end of the bridge. A man is
also standing on the same bridge at a distance x from point A, what would be the range of the values
of x (in m) for which the man will be safe, if he starts running (at the same time and with the same
speed of cat) towards the train? (All speeds remaining the same)

OPTIONS
1) 0 to 20
2) 0 to 30
3) 0 to 40
4) 0 to 50
5) None of these

Question of the Day #02: (14-Apr-09)


The question below contains a paragraph followed by alternative summaries. Choose the option that
best captures the essence of the paragraph.
In The Birth of Tragedy Nietzsche presented his theory of the ancient dualism between two types of
aesthetic experience, namely the Apollonian and the Dionysian; a dualism between the plastic "art
of sculpture", of lyrical dream-inspiration, identity, order, regularity, and calm repose, and, on the
other hand, the non-plastic "art of music", of intoxication, forgetfulness, chaos, and the ecstatic
dissolution of identity in the collective. The analogy with the world of the Greek gods typifies the
relationship between these extremes: two godsons, incompatible and yet inseparable. According to
Nietzsche, both elements are present in any work of art.

OPTIONS
1) In The Birth of Tragedy Nietzsche stated that any work of art contains the ancient Greek
elements typified by Apollo and Dionysius.
2) In The Birth of Tragedy Nietzsche said that the elements of identity and the dissolution of
identity symbolised by the Apollonian and the Dionysian are both present in any work of art.
3) In The Birth of Tragedy Nietzsche argued that any work of art contains the incompatible but
inseparable elements of identity and the dissolution of identity in the collective.
4) In The Birth of Tragedy Nietzsche presented his theory of dualism, that of identity and the
lack of it typified by the Greek gods Apollo and Dionysius.
5) In The Birth of Tragedy Nietzsche presented his theory of dualism in any work of art that of
art of sculpture and of art of music.

Book 2- Question of the Day

www.TestFunda.com

The LEADER in Online Test Prep

Question of the Day #03: (15-Apr-09)

Where y is a real number and y > 1.


If log10 5 0.7, what is the approximate value of log10 y?
OPTIONS
1) 3.4
2) 2.1
3) 4.2
4) 1.7
5) 2.6

Question of the Day #04: (16-Apr-09)


Given below is a passage followed by several statements that can be drawn from the facts stated in
the passage. Examine each statement separately in the context of the passage and decide whether
they are implied from the passage.
Apparently at ease with risk, the businessmen and bankers who are investing in the phoenix-like
rebirth of Alitalia submitted an offer for the airlines assets to the bankrupt flag-carriers
administrator on October 31st, even after failing to convince pilots and cabin crew to sign new
contracts. But although fuel costs have fallen lately, the outlook for aviation has worsened. The
economic slowdown is weakening demand. And now fierce competition is threatening the airlines
services between Rome and Milan - a core part of its business.

A. Competition threatens Alitalias revival.


B. The outlook for aviation industry is bleak because of spiralling fuel costs.
C. The businessmen and bankers havent anticipated the weakening demand in aviation.
D. But for the businessmen and bankers, Alitalia would have been fully out of business.

OPTIONS
1) A only
2) A and D
3) B and C
4) A, C and D
5) D only
Book 2- Question of the Day

www.TestFunda.com

The LEADER in Online Test Prep

Question of the Day #05: (17-Apr-09)


What is the largest 3 digit prime factor of the integer 2000C1000?
OPTIONS
1) 661
2) 211
3) 733
4) 859
5) None of these

Question of the Day #06: (18-Apr-09)


Given below is a passage followed by several statements that can be drawn from the facts stated in
the passage. Examine each statement separately in the context of the passage and decide whether
they are implied from the passage.
Compared with groups who lobby to save animals or trees, campaigners who lobby to preserve
languages are themselves a rare breed. But they are trying both to mitigate and publicise an
alarming acceleration in the rate at which languages are vanishing. Of some 6,900 tongues spoken in
the world today, some 50% to 90% could be gone by the end of the century. In Africa, at least 300
languages are in near-term danger, and 200 more have died recently or are on the verge of death.
Some 145 languages are threatened in East and South-East Asia.

A. Languages are endangered just the way animals or trees are endangered.
B. Very few people campaign for preserving languages.
C. African languages are more susceptible to extinction.
D. In a century the world will have fewer languages than it has today.

OPTIONS
1) A only
2) A and B
3) A, B and D
4) C and D
5) B and C

Book 2- Question of the Day

www.TestFunda.com

The LEADER in Online Test Prep

Question of the Day #07: (19-Apr-09)


Let N = 101 10001 100000001 (100000001) where the last factor of N has (27 1) zeros
between the two one's at the extremes of the number. What will be the number of one's in the final
product of N?

OPTIONS
1) 256
2) 129
3) 128
4) 127
5) None of these

Question of the Day #08: (20-Apr-09)


The following question has five word pairs. Choose the option that presents the best choice of words
in each pair.
The (A) castor/caster (B) had just arrived home. He had (A) munched/minced (B) on raw onions
with his food. As soon he started talking, his wife gave him a (A) baleful/baneful (B) look. For (A)
awhile/a while (B) he did not understand. Once he did, he immediately put a (A) cashew/cachou (B)
in his mouth.

OPTIONS
1) AAABB
2) AABBB
3) BAABB
4) BBABB
5) BABAA

Book 2- Question of the Day

www.TestFunda.com

The LEADER in Online Test Prep

Question of the Day #09: (21-Apr-09)


A group of boys is playing cricket. Intezam is at the striker's end of the pitch and Sachit is at the nonstriker's end. Intezam hits the ball far into the outfield and the batsmen start running. Each of them
runs at a constant speed, though their speeds are not necessarily equal. They first cross each other
at a distance of 9 ft from the striker's end while running the first run. They immediately turn back for
the second run and cross each other at a distance of 3 ft from the non-striker's end while running for
the second run. After this, they turn back for the third run and so on. What is the total distance (In
feet) Sachit has run when they cross each other for the third time?
OPTIONS
1) 45
2) 54
3) 60
4) 75
5) None of these

Question of the Day #10: (22-Apr-09)


The following question has five word pairs. Choose the option that presents the best choice of words
in each pair.
The heads of the nation worked for the (A) ascent/assent (B) of the country. They wanted it to reach
(A) autarchy/autarky (B). With economic self-sustainability and good governance, they hoped to
pick up the (A) dissembled/disassembled (B) pieces and put them back together. They asked the (A)
depositary/depository (B) to release funds with due wisdom. They asked the people to (A)
detract/detrain (B) themselves from anything against development and freedom.
OPTIONS
1) ABBAA
2) AABAA
3) BBBAA
4) ABAAB
5) AABBA

Book 2- Question of the Day

www.TestFunda.com

The LEADER in Online Test Prep

Question of the Day #11: (23-Apr-09)


The length of the non equal side of an isosceles triangle is 12 units. The semiperimeter of the
triangle (in units) is a root of the equation x2 32x + 240 = 0. What is the area of the triangle (in
square units)?

OPTIONS
1) Such a triangle does not exist
2)
3) 20
4)
5) None of these

Question of the Day #12: (24-Apr-09)


Answer the question based on the passage given below.
The most remarkable feature of the recent virus attack (agent.btz) on the classified network that
alarmed pentagon may not be the breach of security, but the cost of dealing with it. In the civilian
world, at least one bank has dealt with agent.btz by blocking all its computers USB ports with glue.
Every bit of portable memory in the sprawling American military establishment now needs to be
scrubbed clean before it can be used again. In the meantime, soldiers will find it hard or outright
impossible to share, say, vital digital maps, let alone synch their iPods or exchange pictures with
their families.
Which of the following can be validly concluded about agent.btz?

OPTIONS
1) It is designed specifically to target military networks.
2) It infects only portable memory devices.
3) It spreads through portable memory devices.
4) Glue is an effective measure against agent.btz.
5) Portable memory devices can be scrubbed clean using glue.

Book 2- Question of the Day

www.TestFunda.com

The LEADER in Online Test Prep

Question of the Day #13: (25-Apr-09)


Which of the following numbers cannot be written as a sum of squares of any four integers?

OPTIONS
1) 310
2) 123
3) 59
4) 67
5) None of these

Question of the Day #14: (26-Apr-09)


Answer the question based on the passage given below.
All around the world, species are vanishing as their habitat is turned over to crops. Such land
conversion is particularly severe in the tropics, where most of the worlds species live. And though
nature reserves offer some respite, they are unlikely to be enough on their own. However, a survey
in the Western Ghats conducted by the National Academy of Sciences shows that the area had been
cultivated for over 2,000 years, but still remains a hotspot for biodiversity. The Academy looked at
birds in different areas of vegetation, including intact forest, plantations and shrub land and found
that local plantations of areca palms retained 90% of the birds associated with native forest.
Which of the following can be inferred from the passage?

OPTIONS
1) Farming and wildlife are not easy bedfellows.
2) The cultivation of the areca palms is widespread in the Western Ghats.
3) The areca palms provides habitat for the forest birds in the region.
4) Areca plantations are biodiversity-friendly.
5) The Western Ghats are hospitable to birds.

Book 2- Question of the Day

www.TestFunda.com

The LEADER in Online Test Prep

Question of the Day #15: (27-Apr-09)


In a ABC, point D is the midpoint of side BC. It is also known that BC > AC > AB and BC = 2AB. The
lengths of the sides of the triangle form an arithmetic progression with the middle term equal to 4.5
units. What is the approximate length of AD (in units)?

OPTIONS
1) 11.25
2) 2.35
3) 3.45
4) 1.95
5) 3.15

Question of the Day #16: (28-Apr-09)


The following question has a paragraph from which the last sentence has been deleted. From the
given options, choose the one that completes the paragraph in the most appropriate way.
A community or social group sustains itself through continuous self-renewal, and this renewal takes
place by means of the educational growth of the immature members of the group. By various
agencies, unintentional and designed, a society transforms uninitiated and seemingly alien beings
into robust trustees of its own resources and ideals. Education is thus a fostering, a nurturing, a
cultivating, process. All of these words mean that it implies attention to the conditions of growth.
We also speak of rearing, raising, bringing up- words which express the difference of level which
education aims to cover. Etymologically, the word education means just a process of leading or
bringing up. When we have the outcome of the process in mind, we speak of education as shaping,
forming, molding activity.___________

OPTIONS
1) Since what is required is a transformation of the quality of experience till it partakes in the
interests, purposes, and ideas.
2) Beliefs and aspirations can be physically extracted and inserted.
3) The required beliefs can be hammered in; the needed attitudes can be plastered on.
4) That is, a shaping into the standard form of social activity.
5) Thus it gradually produces in an individual a certain system of behavior, a certain disposition
of action.

Book 2- Question of the Day

www.TestFunda.com

The LEADER in Online Test Prep

Question of the Day #17: (29-Apr-09)


If 0< x, y, z < 90, what is the value that the following expression cannot attain?

OPTIONS
1) 8.7
2) 8
3) 7.9
4) 9
5) None of these

Question of the Day #18: (30-Apr-09)


The following question has a paragraph from which the last sentence has been deleted. From the
given options, choose the one that completes the paragraph in the most appropriate way.
There is a certain sense in which we should feel a sympathy with children in the wrong that they do.
It would seem paradoxical to say that in any sense there should be sympathy with sin, and yet there
is a sense in which this is true, though perhaps, strictly speaking, it is sympathy with the trial and
temptation which led to the sin, rather than with the act of transgression itself. In whatever light a
nice metaphysical analysis would lead us to regard it, it is certain that the most successful efforts
that have been made by philanthropists for reaching the hearts and reforming the conduct of
criminals and malefactors have been prompted by a feeling of compassion for them, not merely for
the sorrows and sufferings which they have brought upon themselves by their wrongdoing, but for
the mental conflicts which they endured, the fierce impulses of appetite and passion, more or less
connected with and dependent upon the material condition of the bodily organs, under the onset of
which their feeble moral sense, never really brought into a condition of health and vigor, was overborne. These merciful views of the diseased condition and action of the soul in the commission of
crime are in themselves right views for man to take of the crimes and sins of his fellow-man._____.

OPTIONS

1) For what would otherwise differentiate a man from a beast?


2) They also lie at the foundation of all effort that can afford any serious hope of promoting
reformation.
3) From mercy comes compassion and from compassion, all forms of love and co-habitation.
4) It helps to pardon the sins, rather than punish the person who in the first place is incapable
of discerning victory and glory from humiliation and defeat.
5) They also provide a basis for starting a world where one gets a chance to relive and conjoin
rather than living a life of a destitute.

Book 2- Question of the Day

10

www.TestFunda.com

The LEADER in Online Test Prep

Question of the Day #19: (01-May-09)


In ABC the length of sides AB, AC and BC are 8 cm, 6 cm and 10 cm respectively. If point D is on
side BC and BD : DC = 2 : 3, what is the approximate length of AD?

OPTIONS
1) 5.4 cm
2) 6 cm
3) 4.5 cm
4) 5.6 cm
5) 6.1 cm

Question of the Day #20: (02-May-09)


Fill in the blanks in the passage with the most appropriate set of words from the options for each
blank.
He sighed again, and Rachel knew that for the hundredth time he was _______________ his own
past weakness. He had been so foolish in money matters, _____________ away his once
considerable capital in aimless ________________. He and his sister had shared equally under their
father's will, but while he had been at last ____________ to sink the greater part of what was left to
him in an annuity, she had probably increased her original inheritance.

OPTIONS
1) Regretting, Frittering, Speculations, Compelled
2) Implying, Frittering, Day dreaming, Infamous
3) Regretting, Squandering, Trifles, Emulated
4) Implying, Squandering, Speculations, Pressured

Book 2- Question of the Day

11

www.TestFunda.com

The LEADER in Online Test Prep

Question of the Day #21: (03-May-09)


The totient (n) of a positive integer n is defined as the number of positive integers less than or
equal to n that are co prime to n. It follows from the definition that (1) = 1. Also, (9) = 6 since
there are six positive integers (1, 2, 4, 5, 7 and 8) less than 9 are co prime to 9. Which of the
following relations holds true for (n) if m and n are co prime?

OPTIONS
1) (m) + (n) (m + n)
2) (m n) (m) (n)
3) (m n) = (m) (n)
4) (m + n) = (m) + (n)
5) (m n) > 2(m) (n)

Question of the Day #22: (04-May-09)


The following question has a paragraph from which the last sentence has been deleted. From the
given options, choose the one that completes the paragraph in the most appropriate way.
If a plague carried off the members of a society all at once, it is obvious that the group would be
permanently done for. Yet the death of each of its constituent members is as certain as if an
epidemic took them all at once. But the graded difference in age, the fact that some are born as
some die, makes possible through transmission of ideas and practices the constant reweaving of the
social fabric. Yet this renewal is not automatic. ____________________________

OPTIONS
1) Now it is a work of necessity.
2) Unless pains are taken to see that genuine and thorough transmission takes place, the most
civilized group will relapse into barbarism and then into savagery.
3) In fact, the human young are so immature that if they were left to themselves without the
guidance and succor of others, they could not acquire the rudimentary abilities necessary for
physical existence.
4) The young of human beings compare so poorly in original efficiency with the young of many
of the lower animals that even the powers needed for physical sustentation have to be
acquired under tuition.
5) How much more, then, is this the case with respect to all the technological, artistic,
scientific, and moral achievements of humanity!

Book 2- Question of the Day

12

www.TestFunda.com

The LEADER in Online Test Prep

Question of the Day #23: (05-May-09)


Mr. Sharma goes to the market with Rs. 252 in his pocket. Each mango and watermelon costs Rs. 4
and Rs. 3, respectively. He cannot exactly remember the number of mangoes and water melons that
his wife had told him to get. He remembers that

The number of watermelons and mangoes were not co prime to each other.

The number of mangoes he has to buy has to be a multiple of 4.

The difference between the number of watermelons and mangoes bought has to be
minimum but it cannot be zero.

He has to buy at least one watermelon and mango, respectively.

The amount spent to buy mangoes is?

OPTIONS
1) Rs. 98
2) Rs. 96
3) Rs. 192
4) Rs. 188
5) Cannot be determined

Question of the Day #24: (06-May-09)


Fill in the blanks in the passage with the most appropriate set of words from the options for each
blank.
My object has been to produce a work which should be as _____ representative of the present state
of the logic of the Oxford Schools as any of the text-books of the past. The qualities which I have
aimed at before all others have been clearness and______. For the task which I have taken upon
myself I may claim one qualification- that of________; since more than seventeen years have now
elapsed since I took my first pupil in logic for the Honour School of Moderations, and during that
time I have been pretty continuously ______ in studying and teaching the subject.

OPTIONS
1) rarely, inconsistency, inexperience, disengaged.
2) thoroughly, consistency, experience, engaged.
3) thoroghly, consistency, experience, engaged.
4) rarely, consistency, inexperience, disengaged.
5) thoroughly, inconsistency, experience, disengaged.

Book 2- Question of the Day

13

www.TestFunda.com

The LEADER in Online Test Prep

Question of the Day #25: (07-May-09)


f is a polynomial function for real x such that f(x2 + 1) = x4 + 7x2 + 9. If f(x2 + 1) = g(x2 1), then what is
the value of p for which f(g(p)) = g(f(p))?
OPTIONS
1) 1
2) 3
3) 7
4) 49
5) None of these

Question of the Day #26: (08-May-09)


The question below consists of a paragraph in which the first and last sentences are identified.
Choose the option that has the most logical order of the intermediate sentences.
1. Internal combustion engines require lubrication in operation to allow moving parts to slide
smoothly over each other.
A. As engines were adapted for automotive and aircraft use, the need for a high power-toweight ratio led to increased speeds, higher temperatures, and greater pressure on bearings
which in turn required pressure-lubrication for crank bearings and connecting-rod journals.
B. Insufficient lubrication will subject the engine to rapid wear and ultimately, it may even seize
up entirely.
C. Early slow-speed stationary and marine engines were lubricated by gravity from small
chambers similar to those used on steam engines at the timewith an engine tender
refilling these as needed.
D. Simple two-stroke engines are lubricated by oil mixed into the fuel or injected into the
induction stream as a spray.
6. This was provided either by a direct lubrication from a pump, or indirectly by a jet of oil directed
at pickup cups on the connecting rod ends which had the advantage of providing higher pressures as
the engine speed increased.

OPTIONS
1) BDCA
2) BCAD
3) CBAD
4) CABD
5) ABCD

Book 2- Question of the Day

14

www.TestFunda.com

The LEADER in Online Test Prep

Question of the Day #27: (09-May-09)


If,

g(a, b, c) = min(2a, b, 3c),


h(a, b, c) = max{min(a, b), min(b, c), min(c, a)},
k(a, b, c) = min(m, n, p),
Where, m = max(a, b),
n = min(a, b, c),

Which of the following is true?

OPTIONS
1) k{f(2, 3, 6), g(2, 5, 1), h(2, 3, 1)} = 2
2) g(4, 8, 2) > f(6, 5, 12)
3) k(4, 9, 16) is a perfect square
4) Only two of the options are true
5) All the three options are true

Question of the Day #28: (10-May-09)


The question below consists of a set of labelled sentences. These sentences, when properly
sequenced, form a coherent paragraph. Choose the most logical order of sentences from among the
options.
A. Earlier this year Arvind Panagariya of Columbia University published India: The Emerging
Giant.
B. These expatriates rarely forget their country of origin.
C. It has enough of them to run the country with plenty left over to fill prestigious positions at
foreign universities and international organisations.
D. India has always produced more economists than it can consume locally.

OPTIONS
1) ACBD
2) DCBA
3) DBCA
4) ADCB
5) DACB

Book 2- Question of the Day

15

www.TestFunda.com

The LEADER in Online Test Prep

Question of the Day #29: (11-May-09)


The following algorithm is used to calculate the value of p.
Step 1: x = 1, y = 1 and p = 0
Step 2: If y is a perfect square other than 1, then go to step 7
Step 3: x = x + y
Step 4: y = y + x
Step 5: If x is prime number, then p = p + x + y, otherwise p = p
Step 6: Go to step 2
Step 7: p = p + x + y
Step 8: Stop
What is the final value of p?

OPTIONS
1) 518
2) 721
3) 156
4) 285
5) None of these

Book 2- Question of the Day

16

www.TestFunda.com

The LEADER in Online Test Prep

Question of the Day #30: (12-May-09)


The question below consists of a set of labelled sentences. These sentences, when properly
sequenced, form a coherent paragraph. Choose the most logical order of sentences from among the
options.
A. The policy change to a more liberalized economy is explicitly identified using instrumental
variables.
B. In the case of the Chinese regions, the presence of export processing zones may exert
positive effect on the regional growth rate but the increase in regional growth is even more
export inelastic than at the national level.
C. The results provide support that export growth does have a positive and statistically
significant effect on economic growth in these countries.
D. In a comprehensive econometric study on the SEZ policies in China and India, Leong
investigates the impact of opening up the China and Indian economy on economic growth in
these countries using new panel data sets for both the national economies and the regional
economies of China.
E. The growth rates of the countries are export and FDI inelastic, in the sense that a one
percentage point increase in growth rate of export or FDI will have a less than one
percentage point increase in the economic growth rate of these countries.

OPTIONS
1) DBCAE
2) ACDBE
3) DACBE
4) DCBEA
5) ABCED

Question of the Day #31: (13-May-09)


Three different percentage discounts are offered by a shopkeeper to 3 different customers on an
item. Shopkeeper marked the price of the item 25% above the cost price. If the prices at which 3
customers buys these items are in A.P. and the discount percentages offered are natural numbers,
then in how many ways shopkeeper can do this so that he does not bear loss from any of these
customers?

OPTIONS
1) 150
2) 90
3) 360
4) 270
5) None of these

Book 2- Question of the Day

17

www.TestFunda.com

The LEADER in Online Test Prep

Question of the Day #32: (14-May-09)


The question below contains a paragraph followed by four alternative summaries. Choose the option
that best captures the essence of the paragraph.
Allegory is generally treated as a figure of rhetoric, but an allegory does not have to be expressed in
language: it may be addressed to the eye, and is often found in realistic painting, sculpture or some
other form of mimetic or representative art. The etymological meaning of the word is broader than
the common use of the word. Though it is similar to other rhetorical comparisons, an allegory is
sustained longer and more fully in its details than a metaphor, and appeals to imagination, while an
analogy appeals to reason or logic. The fable or parable is a short allegory with one definite moral.

OPTIONS
1) An Allegory is a figure of rhetoric and can be visual too; it is broader and appeals to
imagination as opposed to a metaphor which appeals to logic.
2) An Allegory is a figure of rhetoric and can be visual as well as verbal, and appeals to
imagination as opposed to a metaphor which is verbal and appeals to logic or reason.
3) An Allegory, a figure of rhetoric, is visual and verbal; it is sustainable and appeals to the
imagination like a parable.
4) Though an allegory is similar to metaphor and analogy, it is a broader figure of rhetoric
appealing to imagination as a fable does.
5) Allegories are sustained more fully and longer than other rhetorical comparisons like
metaphor or analogy; They can be visual too.

Book 2- Question of the Day

18

www.TestFunda.com

The LEADER in Online Test Prep

Question of the Day #33: (15-May-09)


The figure ABCD is a kite, with a circle passing though the points A, B and D. The length of side AB = 5
units and the area of ABD is 12 square units. What is the radius of the circle (in units) given that all
sides of ABD have integral values and AE > BE = DE?

OPTIONS
1)
2)
3) 4
4) 6
5) None of these

Question of the Day #34: (16-May-09)


The following question has a sentence from which the last part has been deleted. From the given
options, choose the one that completes the sentence in the most appropriate way.
Vital religion begins for a man when lie first discovers the reality of the living GOD. Most men indeed
profess a belief in GOD, a vague acknowledgment of the existence of "One above"
____________________

OPTIONS
1) and that is when some men begin to move away from priesthood
2) but that is when some men begin to move to priesthood
3) yet the belief overpowers everything the society does
4) but the belief counts for little in their lives.
5) and the belief counts for little in their lives.

Book 2- Question of the Day

19

www.TestFunda.com

The LEADER in Online Test Prep

Question of the Day #35: (17-May-09)


A big circle of diameter '4a' units is drawn with centre O. Then a semi-circle (with diameter AB) is
drawn inside the big circle in such way that its circumference passes through the center of the big
circle and then a small circle is drawn inside the big circle touching the big circle at G and two
perpendicular radius OC and OD at E and F, as shown in the figure. What will be the area of the
shaded region in square units?

OPTIONS
1)
2)
3)
4)
5) None of these

Book 2- Question of the Day

20

www.TestFunda.com

The LEADER in Online Test Prep

Question of the Day #36: (18-May-08)


The question consists of a certain number of sentences. Some sentences are grammatically incorrect
or inappropriate. Select the option that indicates the grammatically correct and appropriate
sentence(s).
A. Her mother had never cut corners; even in Madison she had run her household as if to
satisfy her mother-in-laws fastidious eye.
B. The more haste, the lesser speed.
C. Field guides seldom explain which funguses are edible.
D. He was now sufficiently composed to order a funeral as modest magnificence.
E. Section 309 of the Indian Penal Code says that, whoever attempts to commit suicide and
does any act towards the commission of such offence shall be punished with simple
imprisonment for a term which may extend to one year or with fine, or with both.
OPTIONS
1) B and E
2) A and E
3) A, B and C
4) A and C
5) A, C, E

Question of the Day #37: (19-May-09)


As shown in the figure, two concentric circles are drawn with radius of the outer circle equal to '4a'
units. Two equal and perpendicular chords AD and BC touch the inner circle at Q and P respectively.
Chord BC bisects OL and MF FO. A semicircle is drawn with GH as a diameter. If MO is
perpendicular to GH, then what will be the area of the shaded region in square units?

OPTIONS
1)
2)
3)
4)
5) 44a2

Book 2- Question of the Day

21

www.TestFunda.com

The LEADER in Online Test Prep

Question of the Day #38: (20-May-09)


Answer the following question based on the information given below.
Everyone has a doctor in him or her; we just have to help it in its work. The natural healing force
within each one of us is the greatest force in getting well. Our food should be our medicine. Our
medicine should be our food. But to eat when you are sick is to feed your sickness.
Which of the following logically follows from the above?

OPTIONS
1) We should not eat when we are sick.
2) Everyone is a doctor.
3) Right food helps in healing.
4) The natural healing force comes from food.
5) All of the above.

Question of the Day #39: (21-May-09)

Where n > 15, which of the following is true?

OPTIONS
1)
2)
3)
4) 2 < P < 3
5) None of these

Book 2- Question of the Day

22

www.TestFunda.com

The LEADER in Online Test Prep

Question of the Day #40: (22-May-09)


The question below consists of a set of labelled sentences. These sentences, when properly
sequenced, form a coherent paragraph. Choose the most logical order of sentences from among the
options.
A. The nucleus is the more important of the two and, so to say, governs the life of the cellprotoplasm.
B. And for all life established in nature the cell remains the constant and unchanging form element.
C. It is composed of the most widely differing elements which, taken together, form the so-called
protoplasm or cellular substance.
D. Yet even this tiny cell is already a highly organized and perfected thing.
E. It comprises the cell-protoplasm and a nucleus imbedded in it whose substance is known as the
nucleoplasm.

OPTIONS
1) DCBEA
2) CAEDB
3) ADCEB
4) DEACB
5) EADCB

Book 2- Question of the Day

23

www.TestFunda.com

The LEADER in Online Test Prep

Question of the Day #41: (23-May-09)


As shown in the figure, an equilateral triangle ABC of side '3a' units is circumscribed by the circle
with center O. Now side AB and AC are extended to form another equilateral triangle ADE. Keeping
DE as diameter, a circle with center F is drawn which will pass through O. Now keeping chord BC as a
diameter, a semicircle is drawn which will touch segment DE at F. Then what will be the area of
shaded region in square units? (Take = 3)

OPTIONS
1)
2)
3)
4)
5) None of these

Book 2- Question of the Day

24

www.TestFunda.com

The LEADER in Online Test Prep

Question of the Day #42: (24-May-09)


The following question has a paragraph from which the last sentence has been deleted. From the
given options, choose the one that completes the paragraph in the most appropriate way.
We are very apt to regard the Gospels conventionally. An inherited orthodoxy which has made
peace with the world takes them for granted as "a tale of little meaning, though the words are
strong." An impatient reaction from orthodoxy sets them aside as incomprehensible or unimportant.
It is worthwhile making the effort to empty our minds of prejudice, and to allow the Gospels to tell
their own tale.______________________

OPTIONS
1) For, if we dont, the Gospels will find another way to tell us the tales
2) We shall find that they bring us face to face with a Portrait of surprising freshness and
power.
3) For, an impatient reaction from orthodoxy setting them aside as incomprehensible or
unimportant may be justified
4) Although the Gospels may actually reveal the true nature of tales
5) We shall find that they bring us face to face with a reality that may not be very easy to digest

Question of the Day #43: (25-May-09)


What is the value of p?
Given:
, are the roots of the equation x2 + px + q 2 = 0,
, are the roots of the equation x2 px + q 4 = 0,

= 48

OPTIONS
1) 9
2) 12
3) 16
4) 24
5) Cannot be determined

Book 2- Question of the Day

25

www.TestFunda.com

The LEADER in Online Test Prep

Question of the Day #44: (26-May-09)


The following question has a paragraph from which the last sentence has been deleted. From the
given options, choose the one that completes the paragraph in the most appropriate way.
Heat kills. A heat wave in France in 2003 caused an estimated 35,000 deaths, and a hot spell similar
to the one Britain had last month caused more than 2,000 deaths, according to official estimates.
Although no particular heat wave can be directly attributed to global warming, it will make such
events more frequent. Moreover, if global warming continues unchecked, the number of deaths that
occur when rainfall becomes more erratic, causing both prolonged droughts and severe floods, will
dwarf the death toll from hot weather in Europe. More frequent intense hurricanes will kill many
more. ________________

OPTIONS
1) Tropical diseases will spread, killing still more people.
2) Overwhelmingly, the dead will be those who lack the resources to adapt, and who do not
have access to health care.
3) Even in rich countries, it usually isnt the rich who die in natural disasters.
4) California has just emerged from its own record-breaking heat wave.
5) But most Americans still fail to realize that their countrys refusal to sign the Kyoto protocol
is a moral failing of the most serious kind.

Question of the Day #45: (27-May-09)


If , are the roots of the equation x2 4x + p = 0 and , are the roots of the equation x2 36x + q =
0 and , , , form an increasing G.P. Find the value of (p + q).

OPTIONS
1) 198
2) 225
3) 298
4) 228
5) 246

Book 2- Question of the Day

26

www.TestFunda.com

The LEADER in Online Test Prep

Question of the Day #46: (28-May-09)


The question below consists of a paragraph in which the first and last sentences are identified.
Choose the option that has the most logical order of the intermediate sentences.
1. Transistors come in many different packages (chip carriers).
A. Because they are smaller and have shorter interconnections, SMDs have better high frequency
characteristics.
B. The two main categories are through-hole (or leaded), and surface-mount, also known as surface
mount device (SMD).
C. Transistor packages are made of glass, metal, ceramic or plastic.
D. The package often dictates the power rating and frequency characteristics.
6. However this leads to a lower power rating.
OPTIONS
1) ABCD
2) ACBD
3) CDBA
4) CABD
5) BCAD

Question of the Day #47: (29-May-09)


Let x and y be nonnegative integers such that

What is the sum of all the values of N for different solution sets (x, y)?

OPTIONS
1) 2000
2) 2400
3) 1024
4) 2048
5) None of these

Book 2- Question of the Day

27

www.TestFunda.com

The LEADER in Online Test Prep

Question of the Day #48: (30-May-09)


Answer the question based on the passage given below.
Notwithstanding the obscurity which thus envelops the date of the foundation of
Vondervotteimittis, and the derivation of its name, there can be no doubt, as I said, that it has
always existed as we find it at this epoch. The oldest man in the borough can remember not the
slightest difference in the appearance of any portion of it; and, indeed, the very suggestion of such a
possibility is considered an insult. The site of the village is in a perfectly circular valley, about a
quarter of a mile in circumference, and entirely surrounded by gentle hills, over whose summit the
people have never yet ventured to pass. For this they assign the very good reason that they do not
believe there is anything at all on the other side.
Which of the following comes close to describing the people of Vondervotteimittis?
OPTIONS
1) Change is the only constant when it comes to the people and the town of
Vondervotteimittis.
2) The people of Vondervotteimittis do not like adventure and thereby have not ventured to
see the world beyond their citys own narrow confines.
3) The people of Vondervotteimittis are not given to variation in their life.
4) Alterations and risk are not part of the lives of people of Vondervotteimittis.

Question of the Day #49: (31-May-09)


A question is followed by two statements, A and B. Answer the question using the following
instructions:
Mark (1) if the question can be answered by using statement A alone but not by using statement B
alone.
Mark (2) if the question can be answered by using statement B alone but not by using statement A
alone.
Mark (3) if the question can be answered by using either of the statements alone.
Mark (4) if the question can be answered by using both the statements together but not by either of
the statements alone.
Mark (5) if the question cannot be answered on the basis of the two statements.
What is the thirteenth term of the geometric progression?
A. The sixth term of the geometric progression is 4.
B. The product of the tenth term and the sixteenth term of the geometric progression is 4096.
OPTIONS
1) 1
2) 2
3) 3
4) 4
5) 5

Book 2- Question of the Day

28

www.TestFunda.com

The LEADER in Online Test Prep

Question of the Day #50: (01-Jun-09)


The following question has a paragraph from which the last sentence has been deleted. From the
given options, choose the one that completes the paragraph in the most appropriate way.
Drinking 500ml of beetroot juice a day can significantly reduce blood pressure, research suggests.
The key beneficial ingredient appears to be nitrate, which is also found in green, leafy vegetables.
The researchers found that in healthy volunteers blood pressure was reduced within an hour of
drinking the juice. The study, could suggest a low-cost way to treat hypertension. _____________

OPTIONS
1) Drinking beetroot juice, or consuming other nitrate-rich vegetables, might be a simple way
to maintain a healthy cardiovascular system.
2) Previously the protective effects of vegetable-rich diets have been attributed to their
antioxidant vitamin content, rather than to nitrate.
3) Beetroot juice lowers blood pressure in the short term in volunteers with normal blood
pressure.
4) What we need now is research to see whether it has an effect on people with high blood
pressure over a much longer period of time.
5) There is a growing body of work showing that a diet rich in fruit and vegetables had a
beneficial impact on hypertension.

Question of the Day #51: (02-Jun-09)


If , are the roots of the equation 7x2 5x + 2 = 0, then what is the value of (1 + + 2 + 3 + ... up
to )(1 + + 2 + 3 + ... up to )?

OPTIONS
1)
2)
3)
4)
5) None of these

Book 2- Question of the Day

29

www.TestFunda.com

The LEADER in Online Test Prep

Question of the Day #52: (03-Jun-09)


The question below contains a paragraph with a missing sentence or part of a sentence. Choose the
option that most logically completes the paragraph.
Their methods then became so uncanny, and their man-stalking so well-timed and so certain of
success, that the workmen firmly believed that they were not real animals at all, but devils in lions'
shape. Many a time the coolies solemnly assured me that it was absolutely useless to attempt to
shoot them. They were quite convinced that the angry spirits of two departed native chiefs had
taken this form in order to protest against a railway being made through their country,_______
OPTIONS
1) And by attacking humans to avenge the insult thus shown to them.
2) And by stopping its progress to avenge the insult thus shown to them
3) And by not stopping its progress to avenge the insult thus shown to them.
4) And by not attacking humans to avenge the insult thus shown to them.
5) And by stopping work in the area to avenge the insult thus shown to them.

Question of the Day #53: (04-Jun-09)


A function p(k, n) is defined as
p(k, n) = 0 if k > n
p(k, n) = 1 if k = n
p(k, n) = p(k + 1, n) + p(k, n k) otherwise
Where n and k are positive integers .What is the value of p(4, 16)?

OPTIONS
1) 11
2) 12
3) 15
4) 16
5) None of these

Book 2- Question of the Day

30

www.TestFunda.com

The LEADER in Online Test Prep

Question of the Day #54: (05-Jun-09)


The question below contains a paragraph followed by alternative summaries. Choose the option that
best captures the essence of the paragraph.
Civilisation is a kind of mould that each nation is busy making for itself to shape its men and women
according to its best ideal. All its institutions, its legislature, its standard of approbation and
condemnation, its conscious and unconscious teachings tend toward that object. The modern
civilisation of the west, by all its organised efforts, is trying to turn out men perfect in physical,
intellectual, and moral efficiency. There the vast energies of the nations are employed in extending
man's power over his surroundings, and people are combining and straining every faculty to possess
and to turn to account all that they can lay their hands upon, to overcome every obstacle on their
path of conquest. They are ever disciplining themselves to fight nature and other races; their
armaments are getting more and more stupendous every day; their machines, their appliances, their
organisations go on multiplying at an amazing rate. This is a splendid achievement, no doubt, and a
wonderful manifestation of man's masterfulness which knows no obstacle, and which has for its
object the supremacy of himself over everything else.

OPTIONS
1) The entire concept of civilization is just a farce that man has created to establish himself as
the most superior being on earth.
2) As long as man conceptualizes civilization as a way to rule the world, he will be beset with
hazards and problems.
3) Though civilization is a splendid achievement, the objective has been to establish man's
supremacy over everything else on earth.
4) Civilization should not concentrate on achievements through its vanquishing nature.
5) Man should create a civilization that would embrace everything else on earth too.

Question of the Day #55: (06-Jun-09)


What is the positive value of x which satisfies the equation given below?

OPTIONS
1) 2.34
2) 3.33
3) 0.64
4) 0.67
5) None of these

Book 2- Question of the Day

31

www.TestFunda.com

The LEADER in Online Test Prep

Question of the Day #56: (07-Jun-09)


The question below contains a paragraph followed by alternative summaries. Choose the option that
best captures the essence of the paragraph.
The term Impressionists quickly gained favour with the public. It was also accepted by the artists
themselves, even though they were a diverse group in style and temperament, unified primarily by
their spirit of independence and rebellion. They exhibited together- albeit with shifting membershipeight times between 1874 and 1886. Monet, Sisley, Morisot, and Pissarro may be considered the
purest Impressionists, in their consistent pursuit of an art of spontaneity, sunlight, and colour.
Degas rejected much of this, as he believed in the primacy of drawing over colour and belittled the
practice of painting outdoors. Renoir turned against Impressionism for a time in the 1880s, and
never entirely regained his commitment to its ideas. douard Manet, despite his role as a leader to
the group, never abandoned his liberal use of black as a colour, and never participated in the
Impressionist exhibitions. He continued to submit his works to the Salon, where his Spanish Singer
had won a 2nd class medal in 1861, and he urged the others to do likewise, arguing that the Salon is
the real field of battle where a reputation could be made.
OPTIONS
1) Impressionists were an odd bunch with artists of all extremities forming the group.
Proponents of impressionism often drifted away from it. Monet, Sisley, Morisot, Renoir and
Pissarro were considered the purest impressionists.
2) Purest impressionists were Manet, Sisley, Morisot, Renoir and Pissarro. They were famous
for their spirit of independence and rebellion. They were consistently in pursuit of an art of
spontaneity, sunlight, and colour.
3) Impressionists were a diverse group united by their spirit of independence and rebellion.
There were those who displayed the consistent pursuit of an art of spontaneity, sunlight,
and colour. But some former proponents turned against impressionism.
4) Pure impressionists were the only true impressionists. Their spirit of independence and
rebellion set them apart from other artists. Their constant pursuit of spontaneity, sunlight,
and colour set them apart in spite of criticism.
5) Impressionists became popular with the masses because of their spirit of independence and
rebellion. In the face of criticism from its proponents, impressionists were consistent in their
pursuit of an art of spontaneity, sunlight, and colour.

Question of the Day #57: (08-Jun-09)


The speed of a rail engine is 42 km/hr when no compartment is attached to it and the reduction in
speed is directly proportional to the square root of the number of compartments attached. If the
speed of the train carried by this engine is 24 km/hr when 9 compartments are attached, then what
is the maximum number of compartments that can be carried by the engine so that it can move?

OPTIONS
1) 48
2) 49
3) 46
4) 47
5) None of these
Book 2- Question of the Day

32

www.TestFunda.com

The LEADER in Online Test Prep

Question of the Day #58: (09-Jun-09)


Answer the question based on the statement given below.
A stupid mans report of what a clever man says can never be accurate, because he unconsciously
translates what he hears into something he can understand.
Which of the following is an assumption underlying the above statement?

OPTIONS
1) Stupidity and cleverness are not relative concepts.
2) A clever man can never say anything that a stupid person can accurately report.
3) What a stupid person can understand, he reports accurately.
4) A stupid person can report only what he understands.
5) A stupid person cannot report what a clever person says.

Question of the Day #59: (10-Jun-09)


If x is a real number and x 0, also

Then which of the following is true?

OPTIONS
1)
2)
3)
4)
5) None of these

Book 2- Question of the Day

33

www.TestFunda.com

The LEADER in Online Test Prep

Question of the Day #60: (11-Jun-09)


Answer the following question based on the information given below.
One man is taking the heat so that all men can compete in the kitchen for Best Cook of Mexico City
honours. Benjamin Garcia is outraged that Mexico City officials barred him from entering the contest
last year because he is a man. He has filed a complaint with the citys Human Rights Commission,
arguing the contest discriminated against men and stereotyped women, and that men should be
allowed to participate.
Which of the following argument will weaken Garcias demand?

OPTIONS
1) Traditionally men are not good cooks; hence, it is pointless for them to try and compete with
women.
2) Garcia is the only man who has expressed his desire to participate in a competition meant
for only women.
3) There are several competitions meant only for men and women never complain.
4) This is a competition meant to honour housewives.
5) There is a separate competition for Best Chef of Mexico City meant only for men.

Question of the Day #61: (12-Jun-09)


A question is followed by two statements, A and B. Answer the question using the following
instructions:
Mark (1) if the question can be answered by using statement A alone but not by using statement B
alone.
Mark (2) if the question can be answered by using statement B alone but not by using statement A
alone.
Mark (3) if the question can be answered by using either of the statements alone.
Mark (4) if the question can be answered by using both the statements together but not by either of
the statements alone.
Mark (5) if the question cannot be answered on the basis of the two statements.
Is the two digit number XY divisible by 7?
A. X + Y = 7
B. 4X + 6Y is divisible by 7
OPTIONS
1) 1
2) 2
3) 3
4) 4
5) 5
Book 2- Question of the Day

34

www.TestFunda.com

The LEADER in Online Test Prep

Question of the Day #62: (13-Jun-09)


Answer the question based on the passage given below.
David Colquhoun, a pharmacologist at University College London and a member of the
Complementary and Natural Healthcare Council, wants the National Institute for Health and Clinical
Excellence, which rules on the cost-effectiveness of traditional therapies, to examine the evidence
for complementary medicine. The whole problem of regulating alternative medicine will remain
impossibly chaotic until the government grasps the nettle of deciding what works and what doesnt,
he says.
Which of the following best strengthens the argument of the pharmacologist David Colquhoun?
OPTIONS
1) Consumer protection laws that forbid false claims that a product can cure a disease need to
be strengthened.
2) Alternative medicines do not work or have never been tested.
3) Agencies specially constituted to license conventional drugs, excuse homeopathy, unlike
other treatments, from proving it is more effective than a placebo.
4) In traditional therapies, there are bodies that oversee doctors and nurses and ascertain
whether their practitioners efforts actually work.
5) The Advertising Standards Authority can decide on whether alternative medicine does what
it claims to.

Question of the Day #63: (14-Jun-09)


What is the positive real value of x which satisfies the following equation?

OPTIONS

1)
2)
3)
4)
5) None of these

Book 2- Question of the Day

35

www.TestFunda.com

The LEADER in Online Test Prep

Question of the Day #64: (15-Jun-09)


The economic downturn has made the world more violent and unstable in the last year, according to
a study. The impact of high food and fuel prices in early 2008 and the deepening recession later in
the year eroded peace, according to the Global Peace Index, compiled by a unit of The Economist
magazine group. Economic weakening has increased political instability, demonstrations and crime
in some countries, according to the study. "Rapidly rising unemployment, pay freezes and falls in the
value of house prices, savings and pensions is causing popular resentment in many countries, with
political repercussions," the report says.
Which of the following, if true, would seriously challenge the conclusion of the study?
OPTIONS
1) The study ranked the countries on 23 indicators including political stability, warfare, human
rights, murder rates, military spending, and international relations.
2) Peace is a leading indicator on economic prosperity.
3) Iceland, the most peaceful nation last year, fell to fourth place after violent protests over its
economic meltdown.
4) New Zealand at the head of the table improved its position because of the new coalition
government with a strong parliamentary majority and its low homicide rate and defence
spending.
5) Afghanistan and Iraq, which are the bottom two of the list, were in the last year riddled by
violent terrorist activities arising from anti-west sentiments and ideologies.

Question of the Day #65: (16-Jun-09)


A question is followed by two statements, I and II. Answer the question using the following
instructions:
Mark (1) if the question can be answered by using statement I alone but not by using statement II
alone.
Mark (2) if the question can be answered by using statement II alone but not by using statement I
alone.
Mark (3) if the question can be answered by using either of the statements alone.
Mark (4) if the question can be answered by using both the statements together but not by using
any of the statements alone.
Mark (5) if the question cannot be answered even after using both the statements.
If M = (3n 2n 1), then is M divisible by 4?
I. n is a natural number greater than 1.
II. n3 > 1
OPTIONS
1) 1
2) 2
3) 3
4) 4
5) 5
Book 2- Question of the Day

36

www.TestFunda.com

The LEADER in Online Test Prep

Question of the Day #66: (17-Jun-09)


Each of the following questions has a paragraph from which the last sentence has been deleted.
From the given options, choose the one that completes the paragraph in the most appropriate way.
The youth in todays India are actually a silver spoon generation. Not only have they been born to a
time thats relatively more affluent and buoyant, they are also making the maximum of what they
have in careers or in relationships. This generation has no baggage of yesterday and has no gaping
need-gaps as of today. Such a state of its consumer is a nightmare for classical marketing which is
designed over the years to identify large need-gaps in its consumers and find ways of fulfilling
them._____.

OPTIONS
1) A brand for this audience has to make them feel liberated and empowered.
2) A brand for this audience has to be built on a model of the need gap approach.
3) A brand for this audience has to legitimize their way of life.
4) A brand for this audience cannot be built on the traditional models of youth marketing.
5) A brand for this audience cannot be built by being one with them.

Question of the Day #67: (18-Jun-09)


In a group of 15 children, each child has 15 chocolates of the same kind and no two children have
chocolates of the same kind. What is the minimum number of transactions required for each child to
have chocolates of all kinds, given that a transaction can take place only between two children at a
time and any number of chocolates can be exchanged in a transaction?
OPTIONS
1) 27
2) 105
3) 225
4) 14
5) None of these

Book 2- Question of the Day

37

www.TestFunda.com

The LEADER in Online Test Prep

Question of the Day #68: (19-Jun-09)


Americas once mighty car industry is heading for the scrap yard. Plunging sales amid a financial
crisis and economic downturn forced General Motors and Chrysler to seek a government bailout
Ford is the only one of Detroits Big Three that reckoned it could survive without help. But by the
end of April, Chrysler, the weakest car maker, was forced to file for bankruptcy protections after its
creditors refused to agree on a survival plan that included a tie-up with Italys Fiat. GM, which has
announced a radical restructuring, also faces bankruptcy at the end of May if its aggrieved
bondholders gamble on getting a better deal in the courts.

Which of the following can be validly concluded from the above?

OPTIONS
1) General Motors, Ford and Chrysler are considered the Big Three in the car Industry.
2) Americas car industry may be saved from decline through government bail-outs.
3) Ford may be the only American car manufacturer that will survive the crisis.
4) At least two of Detroits Big three car manufacturers are going to go bankrupt.
5) America's car industry appears to be in terminal decline.

Question of the Day #69: (20-Jun-09)


A question is followed by three statements, I, II and III. Answer the question using the following
instructions:
Mark (1) if the question can be answered by using any one of the statements alone. [I or II or III]
Mark (2) if the question can be answered by using any two of the statements together. [(I and II) or
(II and III) or (III and I)]
Mark (3) if the question can be answered by using all the three statements together. [I and II and III]
Mark (4) if the question cannot be answered on the basis of the three statements.
|x2 y2| = n, Is n an odd number?
I. (x + y) is odd.
II. x and y are distinct.
III. n is an integer.
OPTIONS
1) 1
2) 2
3) 3
4) 4

Book 2- Question of the Day

38

www.TestFunda.com

The LEADER in Online Test Prep

Question of the Day #70: (21-Jun-09)


Each of the following questions has a paragraph from which the last sentence has been deleted.
From the given options, choose the one that completes the paragraph in the most appropriate way.
It is a challenge for democracy how animal rights issues can get an important place in politics. There
is no doubt that the issue is important, but tigers, buffaloes and birds dont have any votes. The
thing to do, then, is to link up the protection of animals with the protection of sustainable
livelihoods. In other words, if we have a concept of wild life protection in which tribals and forest
dwellers can be involved on a large scale in wild life protection by providing them sustainable
livelihoods for this work, we can have schemes with a lot of potential to actually protect wild life.
____.
OPTIONS
1) In addition, it will generate a great deal of interest from political parties who want to attract
tribal votes.
2) In addition, such schemes can obviously be in the forefront of election campaigns.
3) In addition, mass mobilisation based on issues of sustainability becomes possible.
4) In addition, animal rights issues can get an important place in politics.
5) In addition, it will compel political parties to give their whole attention to issues of critical
importance like protection of environment..

Question of the Day #71: (22-Jun-09)


Two equal circles having radii of 2a units and centres P and Q respectively intersect each other as
shown in figure, such that the first circle passes through the centre of the second circle and the
second circle passes though the centre of the first circle. Two identical squares are drawn inside the
circles as shown in the figure. What is the area of the shaded region?

OPTIONS
1)
2)
3)
4)
5) None of these

Book 2- Question of the Day

39

www.TestFunda.com

The LEADER in Online Test Prep

Question of the Day #72: (23-Jun-09)


The big question about the influenza with pandemic potential that struck in April generally referred
to as swine flu is why it kills some people and not othersin particular, why it hits the young (and
thus, presumably, healthy) harder than the elderly. A study has found that nearly two-thirds of
swine-flu infections in America have been in people aged between five and 24, whereas only 1% of
cases affected those over 65. This is the reverse of the pattern seen in seasonal flu, which kills more
old people every winter.
Which of the following, if true, could possibly explain the reverse patterns seen in seasonal and
swine flu?
OPTIONS
1) The young are more susceptible to all kinds of illnesses and especially to fatal illnesses
because of their undeveloped immune system.
2) The older people are susceptible to all kinds of illnesses and especially to fatal ones because
their immune system is worn out.
3) The virus that causes the swine flu takes about fifty years to attain maturity and cause
symptoms after it has entered the human system.
4) The older people might have been in contact with similar viruses for a long time which might
give them some immunity.
5) The young people might have been in contact with similar viruses for some time which might
give them some immunity.

Question of the Day #73: (24-Jun-09)


A question is followed by two statements, I and II. Answer the question using the following
instructions:
Mark (1) if the question can be answered by using statement I alone but not by using statement II
alone.
Mark (2) if the question can be answered by using statement II alone but not by using statement I
alone.
Mark (3) if the question can be answered by using either of the statements alone.
Mark (4) if the question can be answered by using both the statements together but not by using
any of the statements alone.
Mark (5) if the question cannot be answered even after using both the statements.
If n is a natural number,
I. n is an even number.
II. x = n
OPTIONS
1)
2)
3)
4)
5)

1
2
3
4
5

Book 2- Question of the Day

40

www.TestFunda.com

The LEADER in Online Test Prep

Question of the Day #74: (25-Jun-09)


Each of the following questions has a paragraph from which the last sentence has been deleted.
From the given options, choose the one that completes the paragraph in the most appropriate way.
Central to Kallats work is the relationship between text, images, traditional and contemporary
symbols. The urban milieu of Mumbai remains his primary muse; its nurturing of high glamour,
human suffering, simmering aggression and swarming masses an ongoing preoccupation. One of his
most ambitious installations till date- Aquasaurus, a seven-metre long water-tanker, fashioned from
bones, that morphs to become a macabre prehistoric creature for instance, personifies the radical
transformation of Indian city life. _____.
OPTIONS
1) I am liberalisations child; my worldviews are different from artists born twenty years
before me, explains Kallat
2) .If youre a remotely interesting artist, easy classification should be difficult, Kallat says.
3) There was a certain shifting in some of the known territories of painting, Kallat says.
4) Rules were being dismantled and I knew I could make a value addition as a young artist,
Kallat says
5) .Your work is largely what you breathe in, explains Kallat.

Question of the Day #75: (26-Jun-09)


What is the length of the chord PQ of the circle x2 + y2 4x 1 = 0, if the coordinates of point Q are
(1, 2)? The chord PQ makes an angle of 60 with the radius of the circle at the point Q.

OPTIONS
1)
2)
3)
4) 2 units
5) None of these

Book 2- Question of the Day

41

www.TestFunda.com

The LEADER in Online Test Prep

Question of the Day #76: (27-Jun-09)


With the Congress winning a thumping mandate (leaving mathematicians aghast and wondering
how and why the media can describe the inability to reach a simple majority using such superlatives)
political experts are offering every and any possible explanation for the victory. One of the many
explanations is the advertising strategy done by the various political groupings. Currently televised
debates are dime a dozen on the positivity, inclusiveness, youthfulness, pan-India appeal of the ads
of the victors and the negativity, incoherence, exploitative nature of the ads of the losers. The
experts cannot be faulted. Any practising manager can tell you, post-victory, virtually every misstep
can be recast as strategy.
Which of the following is the underlying assumption in the analysis offered by the political experts?

OPTIONS
1) Popular ad campaigns are not enough to win elections.
2) Post victory the mistakes of the election campaign are not highlighted.
3) Popular ad campaigns can win elections.
4) The ad campaigns of the losers are seen to be faulty only after the results of the election.
5) Attributing success and failure in the elections to ad campaigns is an oversimplification.

Question of the Day #77: (28-Jun-09)


A question is followed by two statements, I and II. Answer the question using the following
instructions:
Mark (1) if the question can be answered by using statement I alone but not by using statement II
alone.
Mark (2) if the question can be answered by using statement II alone but not by using statement I
alone.
Mark (3) if the question can be answered by using either of the statements alone.
Mark (4) if the question can be answered by using both the statements together but not by either of
the statements alone.
Mark (5) if the question cannot be answered on the basis of the two statements.
What is the remainder when the largest possible 6 digit odd number is divided by the sum of the
digits of a natural number X?
I. Square of half of X is the same as product of X with the average of the first two odd prime numbers.
II. X is a two digit number.
OPTIONS
1) 1
2) 2
3) 3
4) 4
5) 5
Book 2- Question of the Day

42

www.TestFunda.com

The LEADER in Online Test Prep

Question of the Day #78: (29-Jun-09)


Each of the following questions has a paragraph from which the last sentence has been deleted.
From the given options, choose the one that completes the paragraph in the most appropriate way.
There was a time when models were celebrities in their own right. Karen Lunel as the Liril girl, Jugal
Hansraj as the Nutramul dada and Vicks kid, Kavita Choudhari as Lalitaji, Col Raj Kapoor in the
Volfarm (a tomato ketchup by Voltas) ad and Preity Zinta as the Perk girl to name but a few, will be
recalled by those on the wrong side of 30. In time though models have had to cede stardom to
Bollywood and cricketers and today we tend to think of models as the cutie-pie Sardarji kid in the
Maruti-800 ad, the Malayali fisherman who used FeviKwik as his bait and the funny guy in the
Centreshock commercials. __________________

OPTIONS
1)
2)
3)
4)
5)

They are remembered for their great performances, though the faces are forgotten.
They are no longer celebrities or stars though one can still put a face to them.
Little surprise then that they are household names though they have no identity.
They have no identities and are quickly forgotten, though the ads are remembered.
They span a wide variety of cultures and identities and endear themselves to the audience.

Question of the Day #79: (30-Jun-09)


P, Q and R played a number of card games, which required three players per game. Each game had
only one winner and there were no draws. Each player scored 5 points for a win and lost 3 points for
a loss. At the end, P had 20 points, Q had 12 points and R had 12 points. How many games did the
three play in all?

OPTIONS
1) 30
2) 20
3) 15
4) 18
5) 10

Book 2- Question of the Day

43

www.TestFunda.com

The LEADER in Online Test Prep

Question of the Day #80: (01-Jul-09)


With lay-offs and salary cuts, and of course, more work being assigned to less number of people, it
has become all the more challenging to innovate ideas in order to keep up the motivation level of
employees. Unique methods to do so can only provide stronger means of running a successful
organisation. The key is to use newer approaches that continually reinforce the fact that employees'
ideas are welcome, valued, and rewarded. "Engaged and motivated associates deliver better
effectiveness and efficiency. Constantly improving and innovating the practices deployed to
motivate the associates is a must, irrespective of the market situation," says Satyanarayana
Vinjamoori, head - human resources, ADP India, a US based ITES company.
The ideas in the above argument are based on which of the following assumptions?

OPTIONS
1) Engaged and motivated employees create successful organisations.
2) Strategies and corporate plans cannot be successful without the committed and passionate
involvement of the employees.
3) Engaged and motivated employees help create positive perceptions about the company in
the market.
4) During tough times, employees will unite to ensure that their organisation is able to meet its
objectives.
5) Both options 1 and 2.

Question of the Day #81: (02-Jul-09)


In the given diagram, AB and DC are arcs of concentric circles with centre O. The perimeter of the
figure ABCD is 22 cm. What is the area of the figure ABCD?

OPTIONS
1) 30 cm2
2) 28 cm2
3) 49 cm2
4) 48 cm2
5) Cannot be determined
Book 2- Question of the Day

44

www.TestFunda.com

The LEADER in Online Test Prep

Question of the Day #82: (03-Jul-09)


Each of the following questions has a paragraph from which the last sentence has been deleted.
From the given options, choose the one that completes the paragraph in the most appropriate way.
People walking in the streets look out for different things. Bargain sunglasses, for example, or the
latest pirated movie or, more practically, for broken pavements and puddles. Me, I look out for
bananas . But not any bananas and certainly not the identically bright yellow, elegantly curved
Cavendish variety which is fast predominating with banana sellers in places like Mumbai.
Cavendishes, I have to keep reminding myself, are not that bad. They are agreeably sweet, well
textured and have a picture perfect banana look but that is their problem. Cavendishes were
developed by the banana industry for the huge American market where they are almost the only
banana variety known. ______________
OPTIONS
1) Thanks to this, American consumers have very definite ideas about what bananas are like,
and that is Cavendish and no other.
2) But this is India where we eat more bananas than anyone else and have a matchless range of
local varieties of all shapes, sizes and flavours.
3) But it is fine if one was unfortunate enough to be American.
4) The good thing is that some still sell desi varieties, and this is what I look out for while
walking.
5) Cavendish growers are dumping their surplus on the local market, branding it with names
like Golden Banana to persuade us that it is superior.

Question of the Day #83: (04-Jul-09)

OPTIONS
1) 1
2) 2
3)
4) 0
5) None of these

Book 2- Question of the Day

45

www.TestFunda.com

The LEADER in Online Test Prep

Question of the Day #84: (05-Jul-09)


Will our new government cling to the disaster management agenda of its past term? 'Am admi'
demands a fundamental reorientation. I spent a significant part of the pre-election period in the
flood and tsunami-affected areas of Orissa, West Bengal, Gujarat, Tamil Nadu and Bihar. Talking with
people in recovering communities revealed that for 'am admi', the purpose of national disaster
management is two-fold: the national government must promote safe economic growth as well as
help reduce risks faced by the poor.
Which of the following, if true, would make the 'am admis' demands (about fundamental
reorientation of the governments disaster management agenda) unattainable?

OPTIONS
1) Economic growth and safety of the poor do not go hand in hand.
2) Disaster management strategies protect economic growth by helping communities and
businesses insulate themselves against the impacts of disasters.
3) Disasters and their responses are extremely costly.
4) The 'am admi' that voted for the new government lives on a small farm and makes a living in
agriculture, often using NREGS benefits (National Rural Employment Guarantee Scheme)
when without work.
5) By protecting agriculture, disaster mitigation management programs also protect the poor
from poverty aggravated by disasters.

Question of the Day #85: (06-Jul-09)


What are the number of co-primes of y less than y, where y is the largest number with which when
486, 686 and x are divided the remainders are the same and x is the largest 3 digit number which
when divided by 3 or 8 leaves a remainder of 2 in each case.
OPTIONS
1) 10
2) 20
3) 30
4) 40
5) None of these

Book 2- Question of the Day

46

www.TestFunda.com

The LEADER in Online Test Prep

Question of the Day #86: (07-Jul-09)


Scientists say a natural supplement made from tomatoes, taken daily, can stave off heart disease
and strokes. Ateronon, made by a biotechnology spin-out company of Cambridge University, is being
launched as a dietary supplement and will be sold on the high street. Preliminary trials involving
around 150 people with heart disease indicate that Ateronon can reduce the oxidation of harmful
fats in the blood to almost zero within eight weeks. Lycopene is an antioxidant contained in the skin
of tomatoes which gives them their red colour. But lycopene ingested in its natural form is not
absorbed.
All these provide further support to the views stated in the paragraph EXCEPT
OPTIONS
1) The tomato pill contains an active ingredient lycopene which blocks LDL cholesterol that can
clog the arteries.
2) In clinical trials, the dietary supplement Ateronon has been found to be more effective than
statin drugs that are currently used by doctors to treat high cholesterol.
3) The consumption of tomatoes can help reduce the oxidation of harmful fats in the blood.
4) Ateronon contains a refined, more readily absorbed version of lycopene
5) None of these.

Question of the Day #87: (08-Jul-09)


An ant is at point P and its food is at point Q. To reach its food, the ant has to traverse through the
path as shown by the arrows, reach L, then again go along the path as shown to point M, continue as
directed by the arrows and reach Q. A, B, C and D are the centres of the 4 circles where points P, D, C
and Q are collinear. All the circles are of radius 1 metre. What is the total distance travelled by the
ant to reach Q from P?

OPTIONS
1) 2
2)
3) 3
4)
5) 4
Book 2- Question of the Day

47

www.TestFunda.com

The LEADER in Online Test Prep

Question of the Day #88: (09-Jul-09)


Maruti Suzuki (India), for long Suzuki Motor Corps (Japan) biggest overseas operation in volume
terms, has emerged as the biggest driver of its Japanese parents profits. The Indian car makers
share of Suzukis consolidated profit rose to 46% during the year ended March 2009, up from 30% in
the previous year. Marutis topline is around 13% of the Japanese groups consolidated revenues.
This when major markets, including the US, Europe and most of Asia along with Japan are down, and
car sales have plummeted globally.
Which of the following, if true, must have helped Maruti Suzuki (India) to increase its share of
Suzukis profits?
OPTIONS
1) Suzuki Motors sales had fallen 14% in the fiscal year.
2) Maruti Suzuki Indias share of rural sales has gone up to 20% of the total sales from 3.5% two
years ago.
3) Maruti Suzuki India has a range of two wheelers, the sales of all of which rose from 10% to
60 % in the fiscal year, riding on the back of increased demand from rural India.
4) Suzuki Motors had launched all their top line models of cars in the fiscal year in the Indian
market first.
5) The shares of profits of other subsidiaries of Suzuki Motors in Europe and various other
markets had declined in the fiscal year.

Question of the Day #89: (10-Jul-09)


A question is followed by two statements, A and B. Answer the question using the following
instructions:
Mark (1) if the question can be answered by using the statement A alone but not by using the
statement B alone.
Mark (2) if the question can be answered by using the statement B alone but not by using the
statement A alone.
Mark (3) if the question can be answered by using either of the statements alone.
Mark (4) if the question can be answered by using both the statements together but not by either of
the statements alone.
Mark (5) if the question cannot be answered on the basis of the two statements.
Is the given polygon cyclic?
A. Product of both of its diagonals is 45 cm2.
B. Sum of product of all pairs of opposite sides of the polygon is 45 cm2.
OPTIONS
1)
2)
3)
4)
5)

1
2
3
4
5

Book 2- Question of the Day

48

www.TestFunda.com

The LEADER in Online Test Prep

Question of the Day #90: (11-Jul-09)


The following question has a paragraph from which the last sentence has been deleted. From the
given options, choose the one that completes the paragraph in the most appropriate way.
The Hyderabad-headquartered Insurance Regulatory and Development Authority (IRDA), has the
dubious reputation of having only ex-bureaucrats from one particular state, Andhra, as its Chairmen.
Last week, it added another feather in its cap. In a blatant instance of babu-creep, the IRDA
sought applications for appointment as insurance ombudsman in Hyderabad; but only from retired
IAS or IRS officers. Insurance, as everyone knows, is a highly technical field. It is far removed from
the kind of file-pushing done by our bureaucrats. ____________.

OPTIONS
1) So can IAS or IRS officers possibly deliver as ombudsmen?
2) An ombudsman is an official to provide a check on improper government activity against
the citizen.
3) An Insurance Ombudsman is an official appointed to look into specific complaints against
insurance companies.
4) The strangle-hold of the bureaucracy increases!
5) An insurance ombudsman must have sufficient knowledge about insurance.

Question of the Day #91: (12-Jul-09)


A question is followed by two statements, A and B. Answer the question using the following
instructions:
Mark (1) if the question can be answered by using statement A alone but not by using statement B
alone.
Mark (2) if the question can be answered by using statement B alone but not by using statement A
alone.
Mark (3) if the question can be answered by using either of the statements alone.
Mark (4) if the question can be answered by using both the statements together but not by either of
the statements alone.
Mark (5) if the question cannot be answered on the basis of the two statements.
ABC is an equilateral triangle and point P lies inside ABC. What is the sum of the distances of point
P from the sides of the triangle?
A. Co-ordinates of points P and A are (2, 2) and (3, 4) respectively.
B. Co-ordinates of points B and C are (1, 1) and (5,1) respectively.

OPTIONS
1) 1
2) 2
3) 3
4) 4
5) 5
Book 2- Question of the Day

49

www.TestFunda.com

The LEADER in Online Test Prep

Question of the Day #92: (13-Jul-09)


An Air France plane carrying 228 people from Brazil to France has vanished over the Atlantic after
flying into turbulence, airline officials say. The Airbus sent an automatic message at 0214 GMT, four
hours after leaving Rio de Janeiro, reporting a short circuit. It may have been damaged by lightning.
Which of the following, if true, most seriously weakens the hypothesis that the plane may have
been damaged by lightning?

OPTIONS
1) Aeroplanes get hit by lightning on quite a routine basis without generally any problems
occurring at all.
2) Lightning can strike a plane flying into turbulence - the charge flows around the plane's skin
and can damage electrical systems.
3) All aircraft wings have what are called "static wicks" which dissipate electricity safely and
completely.
4) The Aviation Safety Network database lists just 15 incidents of damage due to lightning in
more than 50 years of aviation history.
5) Lightning can ignite vapour in a fuel tank of an aeroplane, causing an explosion.

Question of the Day #93: (14-Jul-09)


In a polygon, the interior angles are in A.P such that the product of the 4th and the 5th term of the
progression is 18200 and 5th term when divided by the second term gives a quotient of 1 and
remainder of 30. What could be the number of diagonals that can be drawn in the polygon.

OPTIONS
1) 18
2) 35
3) 9
4) 20
5) 14

Book 2- Question of the Day

50

www.TestFunda.com

The LEADER in Online Test Prep

Question of the Day #94: (15-Jul-09)


Since the end of the Cold War in 1989, the world has become a more peaceful place, as more wars
have ceased than have started, the Global Peace Index observes. Promoting peace may boost
economic growth. Between 2000 and 2007, the number of conflicts fell from 40 to 30. One of the
biggest beneficiaries of this has been business. Living without the threat of conflicts and instability
gives rise to an environment in which productive employment - which can only arise if there is peace
- leads to wealth creation. People become motivated by the improved standard of their lives, rather
than seeking retribution for past wrongs. it says. Conversely, when economic development
contracts, violence increases, thus harming the business environment. Military might delivers
geopolitical supremacy, but peace delivers economic prosperity and stability. And that, the report
insists, is what is good for business.
Which of the following most strongly strengthens the argument above?

OPTIONS

1) Between 2000 and 2007 global GDP, or economic output, rose from $32tn to $55tn.
2) Currently global expenditure on military personnel is 40% of global defence spending.
3) In 2007 alone, global defence spending rose 8.4% to $1,140.5bn, and is predicted to increase
by 34% to $1,527.6bn by 2012.
4) Defence spending has a tendency to rise during times of economic hardship.
5) Many governments increase their defence spending in order to provide a fiscal stimulus to
get the economys wheels turning even faster.

Book 2- Question of the Day

51

www.TestFunda.com

The LEADER in Online Test Prep

Question of the Day #95: (16-Jul-09)


Oh! Thou four-headed lord
The planet created by you
With all its animals.
One-fourth of all animals with Fishes
That swarm the seas.
One-fourth with insects
That are omnipresent.
One-sixth with amphibians
Who took the first step in land.
One-twelfth with reptiles
Who crawl on the earth
One-eighth with birds
Who fly in the sky
One-tenth with mammals
Who roam on the land
But after all this, thou great one
Thou did go a step further.
Thou created man.
Who now wants to rule over
Everything thou created.
So can you tell me thou arrogant human being
What percentage of all animals is man?

OPTIONS
1) 2
2) 2.5
3) 3
4) 3.5
5) 4

Book 2- Question of the Day

52

www.TestFunda.com

The LEADER in Online Test Prep

Question of the Day #96: (17-Jul-09)


The following question has a paragraph from which the last sentence has been deleted. From the
given options, choose the one that completes the paragraph in the most appropriate way.
How will climate change affect agriculture? In the run up to the 15th Conference of Parties of the UN
Framework Convention on Climate Change in Copenhagen in December 2009, a policy brief
prepared by the Washington-based International Food Policy Research Institute (IFPRI) carries a dire
warning. It warns water sources will become more variable, droughts and floods will stress
agricultural systems, some coastal food-producing areas will be inundated by the seas, and food
production will fall in some places in the interior. Developing economies and the poorest of the poor
likely will be hardest hit. _____.

OPTIONS
1) As the brief states, ongoing negotiations to address climate change provide a unique
opportunity to combine low-cost mitigation and essential adaptation outcomes with poverty
reduction.
2) However, uncertainties about where climate change will take place and how agriculture will
respond make it difficult to move forward on policies to combat the effects of climate
change.
3) The global community must seize the opportunity if only to ensure the most basic of all
securities food security does not remain a pipe dream for all of mankind.
4) The paper advocates funding research to improve understanding of the interaction between
climate change and agriculture and to find cost-effective ways of reducing agricultures
contribution to green house gas emissions.
5) The fact is even with the best efforts to mitigate climate change it is inevitable that the poor
will be affected.

Question of the Day #97: (18-Jul-09)


How many real values of t exist (t 0), such that

OPTIONS
1) 0
2) 1
3) 2
4) 3

Book 2- Question of the Day

53

www.TestFunda.com

The LEADER in Online Test Prep

Question of the Day #98: (19-Jun-09)


The following question has a paragraph from which the last sentence has been deleted. From the
given options, choose the one that completes the paragraph in the most appropriate way.
Job losses slowed dramatically in May 2009, according to the latest government reading on the
battered labor market, even as the unemployment rate rose to a 26-year high. But some experts
cautioned that the job market remains weak. Employers cut 345,000 jobs from their payrolls in the
month, down from the revised decline of 504,000 jobs in April. This was the fewest jobs lost in a
month since last September, when the bankruptcy of Lehman Brothers caused a crisis in U.S.
financial markets and choked off credit for many businesses. There were still widespread job losses,
as most sectors of the economy, including manufacturing, construction, retail, and business and
professional services posted declines in jobs. _____.

OPTIONS
1) Still, the unemployment rate rose to 9.4% from 8.9% in April.
2) But the unemployment rate rose as people who had stopped looking for work started
looking once again, and thus were classified as unemployed.
3) But economists cautioned that even though it was a better-than-expected jobs report, there
are still signs of weakness in the economy.
4) But there were also some signs of growth, in education and health services, as well as the
leisure and hospitality sector.
5) What's more, the average work week slipped again to a record low 33.1 hours.

Question of the Day #99: (20-Jul-09)


If three times the sum of the roots of the quadratic equation x2 (K + 5)x + 3(2K + 1) = 0 is equal to
the product of the roots of the equation, then for what value of m and n does the equation x4 Kx3 +
mx2 + nx + 1 = 0 have four positive roots?

OPTIONS
1) m = 4, n = 6
2) m = 6, n = 4
3) m = n = 12
4) m = 2, n = 3
5) m = 3, n = 2

Book 2- Question of the Day

54

www.TestFunda.com

The LEADER in Online Test Prep

Question of the Day #100: (21-Jul-09)


The following question has a paragraph from which the last sentence has been deleted. From the
given options, choose the one that completes the paragraph in the most appropriate way.
Last week, I sat down with my producer and a flip cam and she started asking me questions about
my time in Iraq. I hadnt prepared for it at all, but the memories came flooding back. There was a hat
that I used to wear during my 12 weeks out there that was particularly effective for shielding me
from the Iraqi desert sun. I brought that hat home, and hadnt thought about it for a year until one
day my wife and I were planning a hike. I pulled out the hat and put it on at the beginning of the
trail. Inexplicably, I started to sweat, developed a pit in my stomach and almost threw up. At first, I
thought it was something I ate, until I realized it was the smell and feel of that hat that immediately
propelled me back to the battlefield. I had found a trigger. _______________.

OPTIONS
1) I was only there for three months, as compared to military personnel that have been on the
battlefield for years.
2) The symptoms of Post-Traumatic Stress Disorder really never go away.
3) There is a profound psychological and physiological reaction to something traumatic.
4) A traumatic event cant be completely undone, though it can be diminished in the mind.
5) I threw away that hat.

Book 2- Question of the Day

55

www.TestFunda.com

The LEADER in Online Test Prep

Question of the Day #101: (22-Jul-09)


In a game show the contestants are required to search and hit six buzzers which are placed inside a
hexagonal hall having six gates. The below given figure gives an idea about the hall but the
contestants are unaware about the structure of the hall. They just know that they can enter and exit
from any of the six gates. The first three contestants are good friends and decide which doors they
will use and latch those doors in order to trouble other contestants. The first contestant enters from
a door, latches it from inside, hits the buzzers and leaves from another door latching it from outside.
The same is done by the other 2 contestants. The 4th contestant is a nice person, he enters by
opening a door latched from outside, leaves it unlatched, hits the buzzers and leaves by opening a
door latched from inside without latching it from outside. The 5th contestant comes to know that
some doors are latched, hence he even latches the doors which he uses i.e. if he comes across a
latched door and he can open it he uses it and latches it from other side and if he moves through an
unlatched door he latches it after using it. Now the 6th contestant starts. If he picks a door at
random to enter what is the probability that he will be able to enter through that door?

OPTIONS

1)
2)
3)
4)
5) Cannot be determined

Book 2- Question of the Day

56

www.TestFunda.com

The LEADER in Online Test Prep

Question of the Day #102: (23-Jul-09)


The following question has a paragraph from which the last sentence has been deleted. From the
given options, choose the one that completes the paragraph in the most appropriate way.
The tech scene may seem to be going a little out of hand, if you will, with big brother Microsoft
teeming with activity all over again. If the word 'Windows' had started sounding like the anti-thesis
of innovation to you by now, it's time to shake up and wake up a bit. Microsoft is once again trying
to change the rules of many games simultaneously. Catch the software giant's many fingers in
different pies - in operating systems with Windows 7, search engines with Bing, and of course in
gaming with the new Xbox 360 console that would eventually become a one-stop-shop for your
gaming, social networking and communications needs. _______________.
OPTIONS
1) Good or bad there is great activity for sure in Billys kitchen.
2) It is an elaborate spread after a somewhat long innovation famine.
3) It's official that great things are cooking at Microsoft.
4) The new Microsoft hasnt left anything for competition.
5) That would have meant significant expenditure.

Question of the Day #103: (24-Jul-09)

for all integral values of n and f(n) 0.


What is the least value of p for which f(n + p) = f(n)?
OPTIONS
1) 7
2) 8
3) 9
4) 10
5) None of these

Book 2- Question of the Day

57

www.TestFunda.com

The LEADER in Online Test Prep

Question of the Day #104: (25-Jul-09)


The following question has a paragraph from which the first sentence has been deleted. From the
given options, choose the one that completes the paragraph in the most appropriate way.
_____________. There are only local issues being debated in state after state. There is hardly any
overarching national theme being articulated by the Lok Sabha aspirants. It is true that local
development themes bijli, sadak, paani do dominate the discourse in most parts of India.
However, it now appears there simultaneously exists a silent undercurrent, even a mini-wave, at the
national level in favour of the Congress party simply on the "stability and continuity" plank. This
"stability and continuity" plank encompasses many narratives.

OPTIONS
1) The Congress government at the centre has a big challenge in the years ahead.
2) Indian politics seems to have reached a point where the electorate seems to be uninterested
in national issues.
3) It is a positive development bound to improve the delivery mechanism for public goods.
4) India's politics seems to have reached a point where the local appears to have subsumed the
national.
5) Political parties at the Centre and states will have to compete to deal with the changing
politics of aspiration.

Question of the Day #105: (26-Jul-09)


How many sequences of 5 natural numbers (a, b, c, d, e) exist such that abcde a + b + c + d + e
10?
OPTIONS
1) 80
2) 90
3) 115
4) 116
5) 105

Book 2- Question of the Day

58

www.TestFunda.com

The LEADER in Online Test Prep

Question of the Day #106: (27-Jul-09)


The following question has a paragraph from which the first sentence has been deleted. From the
given options, choose the one that completes the paragraph in the most appropriate way.
____________. Cows have digestive bacteria in their stomachs that cause them to belch methane,
the second most significant heat-trapping gas after carbon dioxide. Although it is far less common in
the atmosphere than carbon dioxide, it has 20 times the heat-trapping ability. Frank Mitloehner, a
professor who places cows in air-tight tent enclosures and measures what he calls their eruptions,
says the average cow expels 200 to 400 pounds of methane per year. More broadly, with worldwide
production of milk and beef expected to double in the next 30 years, the United Nations has called
livestock one of the most serious near-term threats to the global climate. In a 2006 report that
looked at the environmental impact of cows worldwide, including forest-clearing activity to create
pasture land, it estimated that cows might be more dangerous to the earths atmosphere than
trucks and cars combined.

OPTIONS
1) Sweetening cow breath is a matter of some urgency, climate scientists say.
2) Chewing her cud on a recent sunny morning, Libby, a 630-kg Holstein, paused to do her part
in the battle against global warming, emitting a fragrant burp.
3) Cows in America accounts for 20 per cent of the countrys emissions of heat-trapping gases.
4) Changes in feed have been the most promising in reducing the methane belched by cows.
5) Making cows belch less is a matter of some urgency, climate scientists say.

Book 2- Question of the Day

59

www.TestFunda.com

The LEADER in Online Test Prep

Question of the Day #107: (28-Jul-09)


There are 12 cities A L connected via State highways (denoted by arrows) and Expressways
(denoted by dotted arrows) as shown in the diagram below, where the arrow denotes the path
between two cities and the number on the arrow denotes the time taken in hours by the person to
travel from one city to another through that path:

A person goes from A to L. His car consumes 4 litres of fuel per hour and has a full tank with
a capacity of 30 litres of fuel (1 litre costs Rs. 50). He knows that there is no fuel available on
any of the paths between any two cities. Also he needs to fill up the entire tank while
refuelling, except in city K.

It is known that there is a toll of Rs. 50 per vehicle on the state highway while the
expressways have a toll of Rs. 120 per vehicle.

He takes a break of 15 minutes at every fuelling station that he stops at.

What route will be cheapest if he wants to reach city L latest by 1:30 am on 25th June and starts at 5
am on 24th June?

OPTIONS
1) A C D E G K L
2) A B C F E G H K L
3) A B D E G K L
4) A B D E F H K L
5) None of these

Book 2- Question of the Day

60

www.TestFunda.com

The LEADER in Online Test Prep

Question of the Day #108: (29-Jul-09)


The following question has a paragraph from which the first sentence has been deleted. From the
given options, choose the one that completes the paragraph in the most appropriate way.
____. Even the latest radar equipment can reach out no more than 550 kilometres (300 nautical
miles) from land. Once out of radar range, pilots flying intercontinental routes make scheduled radio
contact every half an hour or so with air-traffic control stations, behind or ahead of them, to report
their positions. The rest of the time, no one knows exactly where they are. A number of countries,
especially those surrounded by oceans or by vast expanses of rugged wilderness are none too happy
with this. America, Australia and Canada have been among the most active proponents of satellite
navigation for commercial aircraft.

OPTIONS
1) Aeroplanes need satellite navigation even more than anything else.
2) Knowing the plane location while they are flying over oceans is only half the problem.
3) Flying over oceans is one of the challenges of new air-traffic control technology.
4) One of the hazards of flying over oceans is the lack of radar coverage.
5) Some airline companies are planning to provide internet access to the passengers.

Question of the Day #109: (30-Jul-09)


In how many ways can 111 be written as the sum of three distinct integers in geometric
progression?

OPTIONS
1) 2
2) 3
3) 4
4) 5
5) None of these

Book 2- Question of the Day

61

www.TestFunda.com

The LEADER in Online Test Prep

Question of the Day #110: (31-Jul-09)


The following question has a paragraph from which the first sentence has been deleted. From the
given options, choose the one that completes the paragraph in the most appropriate way.
_______. Neither is unique to humans, but Homo sapiens have both in an abundance missing from
other species. Indeed, that abundanceof concern for the well-being of others, (even unrelated
others), and of finely crafted material objects both useful and ornamentalis seen by many as the
mark of man, as what distinguishes humanity from mere beasts. How these human traits evolved is
controversial.

OPTIONS
1) Two of the most peculiar things about people are morality and culture.
2) Two of the oddest things about mankind are violence and morality.
3) Two of the most peculiar things about people are intelligence and war.
4) Two of the oddest things about men are science and culture.
5) Two of the oddest things about people are love and art.

Question of the Day #111: (01-Aug-09)


A sequence of positive real numbers satisfies the following relations:

x1 + x2 + x3 + + x1005 = 2010
Find x25
OPTIONS
1) 25
2)
3)
4)
5)

Book 2- Question of the Day

62

www.TestFunda.com

The LEADER in Online Test Prep

Question of the Day #112: (02-Aug-09)


Answer the question based on the information given in the passage.
A tomato has been engineered to contain higher levels of a chemical which may offer protection
against cancer. Tomatoes, even in their processed form, are already considered to be beneficial to
health. This is because they contain various antioxidant chemicals which may be able to prevent cell
damage in the body. One of these chemicals is called lycopene, the pigment which gives the fruit its
traditional red colour. The new tomato strain developed by scientists at Purdue contains on average
between two and three and a half times the lycopene of the average tomato.
The scientists at Purdue assume that
OPTIONS
1) The higher the ingestion of lycopene in its natural form the better the prevention of cell
damage.
2) Consumption of lycopene as a drug does not have the same effect in the prevention of cell
damage.
3) Lycopene is more beneficial to health when it is contained in tomatoes that are genetically
engineered.
4) It is easier to genetically engineer tomatoes with higher lycopene than other vegetables.
5) Other fruits and vegetables do not contain lycopene. The Advertising Standards Authority
can decide on whether alternative medicine does what it claims to.

Question of the Day #113: (03-Aug-09)


A four digit number is called peculiar if the sum of the number formed by the first two digits and the
last two digits is equal to the number formed by the middle two digits (Example 1978, 19 + 78 = 97).
If all peculiar numbers are arranged in ascending order, find the sum of peculiar numbers just before
and after 1978.

OPTIONS
1) 4285
2) 2876
3) 3876
4) 4175
5) None of these

Book 2- Question of the Day

63

www.TestFunda.com

The LEADER in Online Test Prep

Question of the Day #114: (04-Aug-09)


While our understanding of the world is rapidly increasing, the answer to how life began on Earth
remains elusive. Scientists see evidence of early life in ancient rocks. It is presumed that life arose in
a warm soup rich in carbon compounds, but where did these organic molecules come from? The
answer may lie in interstellar dust, and the possibility that a comet or asteroid may have provided
Earth with the raw ingredients needed for life. Understanding how life emerged on Earth within
1,000 million years of its formation is both a fascinating scientific problem and an essential step in
predicting the presence of life elsewhere in the Universe.

Which of the following has to be true if the hypothesis (about how life began on earth) mentioned
above has to be true?

OPTIONS
1) Life had originated and existed elsewhere in the universe.
2) Life really began in hot volcanic springs on the ancient earth.
3) The first cells were not living cells but inorganic ones formed at the bottom of the oceans.
4) Chemical reactions in the Earth's most ancient atmosphere produced inorganic molecules.
5) Living systems on earth originated in the small compartments in iron sulphide rocks.

Question of the Day #115: (05-Aug-09)


Ram is writing all the natural numbers (starting from 1) in order. He is to stop as soon as he has used
any one digit 119 times. What is the sum total of the numbers he has written by the time he stops?

OPTIONS
1) 16110
2) 16290
3) 16471
4) 16653
5) None of these

Book 2- Question of the Day

64

www.TestFunda.com

The LEADER in Online Test Prep

Question of the Day #116: (06-Aug-09)


The question below contains a paragraph followed by alternative summaries. Choose the option that
best captures the essence of the paragraph.
Confusingly, rent has two different meanings for economists. The first is the commonplace
definition: the income from hiring out land or other durable goods. The second, also known as
economic rent, is a measure of market power: the difference between what a factor of production is
paid and how much it would need to be paid to remain in its current use. A soccer star may be paid
$50,000 a week to play for his team when he would be willing to turn out for only $10,000, so his
economic rent is $40,000 a week. In perfect competition, there are no economic rents, as new firms
enter a market and compete until prices fall and all rent is eliminated.
OPTIONS
1) The two types of rent are: the income from hiring and the excess payment above that
required to induce production. Competitive markets eliminate economic rent.
2) Rent is the income from hiring; in economics rent is the excess payment above that required
to induce production, which competitive markets eliminate.
3) The two types of rent in economics are: the income from hiring and the excess payment
made for production. Perfect competition eliminates the latter.
4) Rent has two different meanings: ordinary definition and economic rent. Economic rent is
eliminated in perfect competition as new firms enter and compete.
5) Rent is the income from hiring; in economics rent is the excess payment above that required
to induce production. Perfect competition eliminates economic rent.

Question of the Day #117: (07-Aug-09)


The question below is followed by two statements, A and B. Answer the question using the following
instructions:
Mark (1) if the question can be answered by using statement A alone but not by using statement B alone.
Mark (2) if the question can be answered by using statement B alone but not by using statement A alone.
Mark (3) if the question can be answered by using either of the statements alone.
Mark (4) if the question can be answered by using both the statements together but not by either of the
statements alone.
Mark (5) if the question cannot be answered on the basis of the two statements.
50 numbers are chosen from the set ,1, 2, 3, , 99-. Find the sum of the numbers?
A. No two numbers sum up to 99.
B. No two numbers sum up to 100.

OPTIONS
1) 1
2) 2
3) 3
4) 4
5) 5

Book 2- Question of the Day

65

www.TestFunda.com

The LEADER in Online Test Prep

Question of the Day #118: (08-Aug-09)


The question below contains a paragraph followed by alternative summaries. Choose the option that
best captures the essence of the paragraph.
Human beings and chimpanzees are closely related genetically, but because of historic differences in
environment, the behaviour of humans is, in many ways, more like that of wolves, which experience
many problems similar to those of ancient man. Such convergences and divergences are
commonplace in biological evolution. Convergence occurs when unrelated animals independently
evolve similar responses to similar environmental conditions- e.g., the similar body shapes of
porpoises and sharks; the similar social behaviour of wolves and humans. Divergence occurs when
closely related species are adapted to different conditions, with a resultant difference in behaviour
and structure. This is the usual type of response; sometimes, however, divergence is extreme
enough to obscure a close relationship. The males of many species of closely related hummingbirds,
birds of paradise, pheasants, and ducks, for example, are superficially so different from one another
that many of these species were formerly assigned to different genera.

OPTIONS
1) In evolution the convergence of structure and biology occurs when unrelated animals evolve
similar responses to similar environmental conditions and divergence occurs when unrelated
animals evolve similar responses to similar environmental conditions.
2) In evolution the divergence of animal behaviour and biology occurs when unrelated animals
evolve similar responses to similar environmental conditions and convergence occurs when
unrelated animals evolve similar responses to similar environmental conditions.
3) In evolution the convergence of animal behaviour and biology occurs when related animals
evolve dissimilar responses to dissimilar environmental conditions and divergence occurs
when unrelated animals evolve similar responses to similar environmental conditions.
4) In evolution the convergence of animal behaviour and biology occurs when unrelated
animals evolve similar responses to dissimilar environmental conditions and divergence
occurs when related animals evolve dissimilar responses to similar environmental
conditions.
5) In evolution the convergence of behaviour and biology occurs when unrelated animals
evolve similar responses to similar environmental conditions and divergence occurs when
related animals evolve dissimilar responses to dissimilar environmental conditions.

Book 2- Question of the Day

66

www.TestFunda.com

The LEADER in Online Test Prep

Question of the Day #119: (09-Aug-09)


The function f(A, B) is a function of two positive integers A and B. It is known that f(1, 0) = f(0, 1),
that f(0, k) = k + 1 (for natural number k) and that f(A, B) = f(A 1, f(A, B 1)). What is the value of
f(1, 1500)?

OPTIONS
1) 1498
2) 1499
3) 1500
4) 1501
5) None of these

Question of the Day #120: (10-Aug-09)


The question below contains a paragraph followed by alternative summaries. Choose the option that
best captures the essence of the paragraph.
Animals Spirits is the colourful name that Keynes gave to one of the essential ingredients of
economic prosperity: confidence. According to Keynes, animal spirits are a particular sort of
confidence, "naive optimism". He meant this in the sense that, for entrepreneurs in particular, "the
thought of ultimate loss which often overtakes pioneers, as experience undoubtedly tells us and
them, is put aside as a healthy man puts aside the expectation of death". Where these animal spirits
come from is something of a mystery.

OPTIONS
1) Animal Spirits, according to Keynes, is the mysterious and naive optimism that guides an
entrepreneur.
2) Animal Spirits, according to Keynes, is the spontaneous optimism of an entrepreneur that
makes him unmindful of the risks involved.
3) Animal Spirits, according to Keynes, is the confidence of an entrepreneur that helps him put
aside the inevitable loss.
4) Animal Spirits, according to Keynes, is the spontaneous optimism of an entrepreneur that
helps him put aside the thought of ultimate loss.
5) Animal spirits, according to Keynes, are a sort of irrational confidence that makes an
entrepreneur postpone the thought of the ultimate loss.

Book 2- Question of the Day

67

www.TestFunda.com

The LEADER in Online Test Prep

Question of the Day #121: (11-Aug-09)

Which of these is equal to the value of X?

OPTIONS
1)
2)
3)
4)
5) None of these

Question of the Day #122: (12-Aug-09)


The question below contains a paragraph followed by alternative summaries. Choose the option that
best captures the essence of the paragraph.
That Kabirs early life began as a Muslim there is no doubt, although he later became influenced by a
Hindu ascetic, Ramananda. Kabir, instead of choosing the Hindu religion or Islam, took what seemed
to him to be the best tenets of both and preached his own religion, called sahaja-yoga (simple
union). He thus became the forerunner of a number of cults, of which Kabirpanth is the most
important, as well as of a separate religion, Sikhism. From Hinduism he accepted the ideas of
reincarnation, or transmigration, and the law of karma, but he rejected idolatry, asceticism, and the
caste system. From Islam he accepted the idea of one God and the equality of man before God. The
ideas of the Muslim mystics, called Sufis, also influenced Kabir greatly.

OPTIONS
1) Kabir combined the best tenets of Hindu and Muslim religions to preach his own religion
called sahaja-yoga and became the forerunner of a number of cults including Sikhism.
2) Kabir combined the influences of Ramananda and Sufi masters in his own religion called
sahaja-yoga and established a number of cults which became the forerunner of Sikhism.
3) Kabir combined the influences of Ramananda and Sufi masters in his own religion called
sahaja-yoga and established a number of cults and founded Sikhism.
4) Kabir combined the best tenets of Hindu and Muslim religions to preach his own religion
called sahaja-yoga and became the forerunner of Sikhism.
5) Though the early influence on Kabir was that of Islam, and later that of Ramananda, Kabir
chose to preach a new religion. He thus became the forerunner of Sikhism.

Book 2- Question of the Day

68

www.TestFunda.com

The LEADER in Online Test Prep

Question of the Day #123: (13-Aug-09)


How many two digit numbers give a perfect square when they are added to the number formed by
reversing their digits?

OPTIONS
1) 0
2) 3
3) 5
4) 8
5) None of these

Question of the Day #124: (14-Aug-09)


The question below contains a paragraph followed by alternative summaries. Choose the option that
best captures the essence of the text.
From 1940 on, with the diffusion of Existentialism through continental Europe, its directions have
developed in terms of the diversity of the interests to which they are subject: the religious interest,
the metaphysical interest, the moral and political interest. This diversity of interests is rooted, at
least in part, in the diversity of sources on which Existentialism has drawn. One such source has been
the subjectivism of the 4th5th-century theologian St. Augustine, who exhorted man not to go
outside himself in the quest for truth, for it is within him that truth abides. If you find that you are
by nature mutable, he wrote, transcend yourself. Another source has been the Dionysian
Romanticism of Nietzsche, who exalted life in its most irrational and cruel features and made this
exaltation the proper task of the higher man, who exists beyond good and evil. Still another source
has been the nihilism of Dostoyevsky, who, in his novels, presented man as continually defeated as a
result of his choices and as continually placed by them before the insoluble enigma of himself.

OPTIONS
1) The diversity of interests of Existentialism in religious, metaphysical, moral and political
aspects of existence is rooted in parts in the subjectivism of St. Augustine, Dionysian
Romanticism of Nietzsche, and the nihilism of Dostoyevsky.
2) As a consequence of the diversity of the sources from which Existentialist doctrines have
been drawn, they focus on several aspects of existence.
3) As Existentialism has drawn from diverse sources like Subjectivism, Romanticism, and
Nihilism the interests to which Existentialism is subject in Europe are also diverse.
4) The diversity of interests of Existentialism in religious, metaphysical, moral and political
aspects of existence is rooted in the subjectivism of St. Augustine, Dionysian Romanticism of
Nietzsche, and the nihilism of Dostoyevsky.
5) The diversity of interests of Existentialism in religious, metaphysical, moral and political
aspects of existence is rooted in parts in the diversity of interests from which Existentialism
has drawn.
Book 2- Question of the Day

69

www.TestFunda.com

The LEADER in Online Test Prep

Question of the Day #125: (15-Aug-09)


A tank has eight inlet taps to let water into the tank and four outlet taps to let water out of the tank.
Each inlet tap can fill up the tank in 24 seconds, while each outlet tap takes 35 seconds to drain out
the entire tank. If all the taps are opened for 4 seconds when the tank is empty, approximately what
volume (in %) of the tank is filled?

OPTIONS
1) 87.5
2) 80
3) 95
4) It overflows
5) None of these

Question of the Day #126: (16-Aug-09)


The question below consists of a paragraph in which the first sentence is fixed and the sentences
following it are jumbled. Choose from among the options the most logical order of the sentences.
A. Z particle is a massive electrically neutral carrier particle of the weak force that acts upon all
known subatomic particles. It is the neutral partner of the electrically charged W particle.
B. For decades after that, it appeared that only charged weak messengers were necessary to
account for all observed weak interactions. However, the attempts to produce a gaugeinvariant theory of the weak force- i.e., a theory that is symmetrical with respect to
transformations in space and time- suggested unifying weak and electromagnetic
interactions.
C. According to the Standard Model of particle physics, the W and Z particles are the gauge
bosons that mediate the weak force responsible for some types of radioactive decay and for
the decay of other unstable, short-lived subatomic particles.
D. The concept that the weak force is transmitted by intermediary messenger particles arose
following the successful description of the electromagnetic force in terms of the emission
and absorption of photons.
E. The Z particle has a mass of 91.19 gigaelectron volts nearly 100 times that of the proton.
The W is slightly lighter, with a mass of 80.4 GeV. Both particles are very short-lived, having
lifetimes of only about 10-25 second.

OPTIONS
1) ECDB
2) EDCB
3) CDEB
4) CBDE
5) DBEC

Book 2- Question of the Day

70

www.TestFunda.com

The LEADER in Online Test Prep

Question of the Day #127: (17-Aug-09)


Which of the following numbers divides 234 + 182 + 722 + 21162?

OPTIONS
1) 2
2) 3
3) 5
4) 7
5) None of these

Question of the Day #128: (18-Aug-09)


The question below consists of a paragraph in which the first sentence is fixed and the sentences
following it are jumbled. Choose from among the options the most logical order of the sentences.
A. The initial decline in U.S. output in the summer of 1929 is widely believed to have stemmed
from tight U.S. monetary policy aimed at limiting stock market speculation.
B. As a result, when a variety of minor events led to gradual price declines in October 1929,
investors lost confidence and the stock market bubble burst. Panic selling began on Black
Thursday, October 24, 1929.
C. The 1920s had been a prosperous decade, but not an exceptional boom period; prices had
remained nearly constant throughout the decade, and there had been mild recessions in
both 1924 and 1927.
D. Some scholars believe that a boom in housing construction in the mid-1920s led to an excess
supply of housing and a particularly large drop in construction in 1928 and 1929. By the fall
of 1929, U.S. stock prices had reached levels that could not be justified by reasonable
anticipations of future earnings.
E. The one obvious area of excess was the stock market. Stock prices had risen more than
fourfold from the low in 1921 to the peak in 1929. In 1928 and 1929, the Federal Reserve
had raised interest rates in hopes of slowing the rapid rise in stock prices. These higher
interest rates depressed interest-sensitive spending in areas such as construction and
automobile purchases, which in turn reduced production.

OPTIONS
1) EDBC
2) DBEC
3) CEDB
4) DBCE
5) CEBD

Book 2- Question of the Day

71

www.TestFunda.com

The LEADER in Online Test Prep

Question of the Day #129: (19-Aug-09)


Let a, b be positive integers such that a2 2009b + 2b2 = 0. Find a + b.

OPTIONS
1) 1372
2) 588
3) 688
4) 784
5) 884

Question of the Day #130: (20-Aug-09)


The question below contains a paragraph with a missing sentence or part of a sentence. Choose the
option that most logically completes the paragraph.

The prophecies of Nostradamus have given birth to much speculation. The major reason appears to
be the coded, poetic format of his work. His work Century remains cryptic to this day, in spite of
many translations and research by different scholars. Now, hopes of demystifying the texts are
resurfacing. ______.

OPTIONS
1) A scientist has calculated the exact day and date of Nostradamus death.
2) The future is never fixed as Nostradamus had us to believe, instead it follows the whims of
the Lord.
3) A folio containing Nostradamus personal notes and calculations was apparently discovered
in a Monastery deep in the Swiss Alps.
4) Modern research is increasingly using the internet and other electronic media in its pursuit
of the truth.
5) Nostradamus mysteries may soon be unraveled.

Book 2- Question of the Day

72

www.TestFunda.com

The LEADER in Online Test Prep

Question of the Day #131: (21-Aug-09)


Akshay is a strange liar. He lies on six days of a week, but on the seventh day he always tells the
truth. He made the following statements on three consecutive days.
Day 1: "I lie on Monday and Tuesday."
Day 2: "Today, it's Thursday, Saturday, or Sunday."
Day 3: "I lie on Wednesday and Friday."
On which day does Akshay tell the truth?

OPTIONS
1) Monday
2) Tuesday
3) Wednesday
4) Thursday
5) Friday

Question of the Day #132: (22-Aug-09)


Given the current scares just as the tourism season is hotting up- from attacks on Indians in Australia
to the threat of swine flu in North America- there is even more reason for a staycation. And we are
lucky that India has such a diversity of destinations to offer, from cool climes to sultry sands. Its just
that we dont know how much our own country has to offer. Think about it- theres nothing we lackhorseshoe falls a la Niagara in Chhattisgarh, a crater like Tanzanias Ngorongoro near Aurangabad,
coral reefs in Lakshadweep, Francophone charm in Pondicherry, prehistoric paintings in Bhimbetka,
frescoes surpassing the Sistine Chapel in Ajanta, snowcaps higher than Switzerland in Himachal,
orchid-studded rainforests in Arunachal, cherry blossoms like Tokyos in Sikkim...the list is endless.
The writer assumes which of the following?

OPTIONS
1) There is no better place in the world to discover than India.
2) There is no reason to travel to other destinations for a vacation.
3) India offers world class tourist destinations.
4) Indians tourists are likely targets for racist attacks in Australia.
5) In India, people need not take a vacation.

Book 2- Question of the Day

73

www.TestFunda.com

The LEADER in Online Test Prep

Question of the Day #133: (23-Aug-09)


There are 64 identical cubes numbered 1 to 64. The number of a cube is written on all its faces i.e.
for a cube numbered 15, 15 is written on all its six faces. If all these 64 cubes are joined together to
form a single large cube, then the sum of all the numbers that appear on the faces of the larger cube
is at least how much?

OPTIONS
1) 2160
2) 2060
3) 2130
4) 2090
5) None of these

Question of the Day #134: (24-Aug-09)


The question below contains a paragraph followed by alternative summaries. Choose the option that
best captures the essence of the text.
Abatement in law is the interruption of a legal proceeding upon the pleading by a defendant of a
matter that prevents the plaintiff from going forward with the suit at that time or in that form. Pleas
in abatement raise such matters as objections to the place, mode, or time of the plaintiff's claim. At
one time, abatement of proceedings in equity differed from abatement in law in that the former
merely suspended the action, subject to revival when the defect was cured, whereas the latter
terminated it, though the plaintiff could start the action anew.

OPTIONS
1) Abatement in law is the termination of the proceedings until re-initiated by the plaintiff;
abatement in equity suspends the proceedings until the defect is cured.
2) Abatement in law suspends the proceedings until the defect is cured or re-initiated by the
plaintiff.
3) Abatement in law is the cessation of proceeding at the instance of the defendant;
abatement in equity suspends the proceedings until the defect is cured.
4) Abatement in law is the termination of proceeding at the instance of the plaintiff and may
be re-initiated by the defendant; abatement in equity suspends the proceedings until the
defect is cured.
5) Abatement is the legal termination of proceeding at the instance of the plaintiff and may be
re-initiated by the defendant; abatement in equity suspends the proceedings until the defect
is cured.

Book 2- Question of the Day

74

www.TestFunda.com

The LEADER in Online Test Prep

Question of the Day #135: (25-Aug-09)


Bini and Vini play a game of words using an unbiased die. They have defined a word simply as a
collection of letters. If the outcome of throwing the die is 1, 3 or 5, odd numbered letters of the
alphabet are selected, while if it is 2, 4 or 6, even numbered letters of the alphabet are selected. The
objective is to make words from the letters so selected. In two turns, Bini threw the die twice and
did not get a composite number in any of the throws. Every time she threw the die, she first formed
all the words possible, returned the letters of the alphabet that were selected, and only then threw
the die again. From the letters she could select, she decided to make a 3-letter word containing
three different letters including atleast one vowel. How many words of this kind did she make?

OPTIONS
1) 0
2) 1380
3) 2760
4) None of these
5) Cannot be determined

Question of the Day #136: (26-Aug-09)


The question below contains a paragraph followed by alternative summaries. Choose the option that
best captures the essence of the paragraph.
Avoiding deforestation means that many landowners must forgo the right to cut down their trees, so
that the world at large can benefit. As such, carbon emissions from deforestation are a classic
example of environmental externality. So long as this remains the case, forests will continue to be
cut down. To resolve the problem, it has been suggested that the people who forgo their rights are
compensated. There is already a market for what are called voluntary credits in avoided
deforestation.

OPTIONS
1) The existing market for voluntary credits in avoided deforestation is an effective way to
internalise the environmental externality in environmental economics.
2) The existing market for voluntary carbon credits in avoided deforestation compensates the
landowners and resolves the problem of deforestation by the landowners and reduces
carbon emissions.
3) In order to decrease carbon emissions from deforestation landowners must be compensated
for not cutting down trees, and the market for voluntary carbon credits must be
strengthened.
4) The existing market for voluntary carbon credits in avoided deforestation compensates
landowners and decreases carbon emissions from deforestation.
5) The existing market for voluntary carbon credits in avoided deforestation could be one of
the ways to resolve deforestation by the landowners, and decrease carbon emissions from
deforestation.

Book 2- Question of the Day

75

www.TestFunda.com

The LEADER in Online Test Prep

Question of the Day #137: (27-Aug-09)


If N is a seven digit natural number such that when it is added to the sum of its own digits the result
is the least eight digit number. What is the remainder when N is divided by 9?

OPTIONS
1) 0
2) 5
3) 9
4) 1
5) 3

Question of the Day #138: (28-Aug-09)


The question below contains a paragraph followed by alternative summaries. Choose the option that
best captures the essence of the text.
That people have personalities goes without saying. There are the shy, the cruel, the kind, and the
sceptical. And, it is hardly uncommon to hear a dog described as friendly or inquisitive. Scientific
research has confirmed that dogs do indeed have personality traits similar to those found in people.
In dogs, for instance, these are usually referred to as energy-level, affection-aggression, anxietycalmness and intelligence-stupidity; in people they are extroversion, agreeableness, neuroticism,
openness to experience and conscientiousness. Yet in spite of all this, rather little has been done to
find out if such characteristics exist in wild animals. Studies in Animal Behaviour show not only that
some do but also that the presence of such traits is skewing the way data is collected by researchers.

OPTIONS
1) People and animals have personalities, but animal personalities are not recognized by
scientific research.
2) Animals have personalities like humans. However scientific research so far has either
ignored them or is biased against them.
3) Animals have personalities the way people have. However, scientific research in animal
behaviour is not only inadequate but also biased.
4) Dogs have personalities just like people. However, scientific research ignores wild animals,
and the data collected about animal behaviour by scientific research is skewed.
5) Scientific research has established that dogs have personality traits like those of people.
Wild animal behaviour is not adequately studied and data collected is skewed.

Book 2- Question of the Day

76

www.TestFunda.com

The LEADER in Online Test Prep

Question of the Day #139: (29-Aug-09)


What is the sum of the first 46 prime numbers?

OPTIONS
1) 3266
2) 3087
3) 4226
4) 3936
5) 4227

Question of the Day #140: (30-Aug-09)


The question below contains a number of sentences. Each sentence has pairs of word(s)/phrases
that are highlighted. From the highlighted word(s)/phrase(s), select the most appropriate
word(s)/phrase(s) to form correct sentences. Then, from the options given, choose the best one.
1. In certain countries thievery is equal (A) / equivalent (B) to murder.
2. The jury would rather acquit than (A) / then (B) convict on the basis of flimsy evidence.
3. Maria is older (A) / elder (B) than Federico.
4. He has provided a veracious (A) / voracious (B) statement of facts as justification.
5. The police tried to elicit (A) / illicit (B) information from the suspect.

OPTIONS
1) AAAAA
2) ABAAB
3) BAAAB
4) BBBAA
5) BAAAA

Book 2- Question of the Day

77

www.TestFunda.com

The LEADER in Online Test Prep

Question of the Day #141: (31-Aug-09)


If log x 4 (2x2 14x + 21) log x 4 (x2 16), then what is the range of values that x can take?
OPTIONS
1)
2)
3)
4)
5) None of these

Question of the Day #142: (01-Sep-09)


Each of the questions below contains a number of sentences. Each sentence has pairs of
word(s)/phrases that are highlighted. From the highlighted word(s)/phrase(s), select the most
appropriate word(s)/phrase(s) to form correct sentences. Then, from the options given, choose the
best one.
1. He caste (A) / cast (B) his remarks to suit the occasion.
2. Cricket matches these days also employ the services of claques (A) / clacks (B) in the form of
cheerleaders.
3. Those obscure forces of nature were personified by the Greeks in shapes part human and
part beast, the satyr and the faun (A) / fawn (B).
4. They would have stripped off his stockings if his greaves (A) / grieves (B) had not prevented
them.
5. The child looks pretty in a skirt that flares (A) / flairs (B) from the waist.

OPTIONS
1) BABAA
2) ABABB
3) BAAAA
4) ABABA
5) BAAAB

Book 2- Question of the Day

78

www.TestFunda.com

The LEADER in Online Test Prep

Question of the Day #143: (02-Sep-09)


Is it possible to find the average of the first n 1 integers of a set of n integers?
A. All n integers are in AP with a known value of common difference.
B. The n integers are the n roots of the equation xn + Axn 1 + Bxn 2 + ... + K = 0. The value of A is
known.

OPTIONS
1) 1
2) 2
3) 3
4) 4
5) 5

Question of the Day #144: (03-Sep-09)


The question below contains a number of sentences. Each sentence has pairs of word(s)/phrases
that are highlighted. From the highlighted word(s)/phrase(s), select the most appropriate
word(s)/phrase(s) to form correct sentences. Then, from the options given, choose the best one.
1. The professor was too obtuse (A) / abstruse (B) to realize that his remarks were offensive.
2. Their precipitous (A) / precipitate (B) entry into the foreign markets led to disaster.
3. It was yellowish brown and seemed coated with a rough and half-dry mucus (A) / mucous (B)
4. They reached a settlement after months of tortulous (A) / tortuous (B) negotiations.
5. I sought out the few friends I knew who might be able to get me work; but they were either
uninterested (A) / disinterested (B) or unable to find anything for me.

OPTIONS
1) ABBAB
2) BABAA
3) ABABB
4) BABBA
5) ABABA

Book 2- Question of the Day

79

www.TestFunda.com

The LEADER in Online Test Prep

Question of the Day #145: (04-Sep-09)


The question below is followed by two statements, A and B. Answer the question using the following
instructions:
Mark (1) if the question can be answered by using statement A alone but not by using statement B
alone.
Mark (2) if the question can be answered by using statement B alone but not by using statement A
alone.
Mark (3) if the question can be answered by using either of the statements alone.
Mark (4) if the question can be answered by using both the statements together but not by either of
the statements alone.
Mark (5) if the question cannot be answered on the basis of the two statements.
Can we find a sequence of 1000 consecutive positive integers, in which there are exactly 5 prime
numbers?

A. The sequence A! + 2, A! + 3, , A!+ 1001 has no prime numbers, (A >200).


B. The sequence B!+1, B!+2, , B!+1000 has exactly 10 prime numbers, (B > 200).

OPTIONS
1) 1
2) 2
3) 3
4) 4
5) 5

Book 2- Question of the Day

80

www.TestFunda.com

The LEADER in Online Test Prep

Question of the Day #146: (05-Sep-09)


The question below contains a number of sentences. Each sentence has pairs of word(s)/phrases
that are highlighted. From the highlighted word(s)/phrase(s), select the most appropriate
word(s)/phrase(s) to form correct sentences. Then, from the options given, choose the best one.
1. The woman was at first a little brief and sullen in her answers, but care for her family soon
rendered her more complaisant (A) / complacent (B).
2. The Patents Act provides an exhaustive code for determining who is entitled to the grant of
a patent, the person entitled being the inventor or his/her successor in title, the inventor
being the actual devisor (A) / deviser (B).
3. Many of these holes are as round and as cleanly cut as if they had been made with an auger
(A) / augur (B).
4. No novelist has more magnificent power in dramatic scenes, as in the climatic (A) / climactic
(B) series in 'Vanity Fair.
5. The professor inserts a caret (A) / carat (B) to indicate that something needs to be inserted
in the text.

OPTIONS
1) BAABB
2) AABBA
3) AAABA
4) BABAA
5) ABABB

Question of the Day #147: (06-Sep-09)


Aarav, who lived on Earth, went to Mars and met Jadoo, Badoo, Kadoo and Ladoo there. All four of
them, who were residents of Mars, had some chocolates with them. On the arrival of Aarav, they
became so happy that all of them gave away all their chocolates to Aarav. The number of chocolates
with Jadoo, Badoo, Kadoo and Ladoo were 15, 40, k01 and 122 respectively as per the number
system prevalent on their planet. They also told Aarav that the number of chocolates they had was
in an arithmetic progression in the number system used on their planet. Aarav is an expert in
number systems. If Aarav counts the chocolates that he received, which of the following values
cannot be the number of chocolates counted by him?

OPTIONS
1) 122
2) 233
3) 442
4) 278
5) None of these

Book 2- Question of the Day

81

www.TestFunda.com

The LEADER in Online Test Prep

Question of the Day #148: (07-Sep-09)


The question below contains a paragraph followed by alternative summaries. Choose the option that
best captures the essence of the text.
Nietzsches early works The Birth of Tragedy and the four Unzeitgemsse Betrachtungen (1873;
Untimely Meditations), are dominated by a Romantic perspective influenced by Schopenhauer and
Wagner. The middle period, from Human, All-Too-Human up to The Gay Science, reflects the
tradition of French aphorists. It extols reason and science, experiments with literary genres, and
expresses Nietzsche's emancipation from his earlier Romanticism and from Schopenhauer and
Wagner. In his mature writings Nietzsche was preoccupied by the origin and function of values in
human life.

OPTIONS
1) Nietzsche's writings fall into three well-defined periods. Nietzsche's mature philosophy
emerged after The Gay Science dealing with the origin and function of values in human life.
2) Nietzsche's writings fall into two well-defined periods; the Romantic perspective influenced
by Schopenhauer and Wagner and the mature writings that followed The Gay Science.
3) Nietzsche's writings fall into three well-defined periods- the Romantic perspective of
Schopenhauer and Wagner; the one that reflects the tradition of French aphorists; and the
mature preoccupation with human values.
4) Nietzsche's writings fall into two well-defined periods- the Romantic perspective influenced
by Schopenhauer, Wagner and the tradition of French aphorists followed by the mature
inquiry into the origin and function of human values.
5) Nietzsche's writings fall into well-defined periods- the Romantic perspective followed by
Schopenhauer, Wagner and the tradition of French aphorists and the mature inquiry into the
origin and function of human values.

Question of the Day #149: (08-Sep-09)


How many 10-digit positive integers with distinct digits are multiples of 11111?

OPTIONS
1) 1234
2) 2345
3) 3456
4) 4567
5) None of these

Book 2- Question of the Day

82

www.TestFunda.com

The LEADER in Online Test Prep

Question of the Day #150: (09-Sep-09)


The question below consists of a set of labelled sentences. These sentences, when properly
sequenced, form a coherent paragraph. Choose the most logical order of sentences from among the
options.
A. I certainly hope they do for their own sakes.
B. Eventually, most were reduced to goongi gudiyas nothing more than decorative, token
creatures, whose presence was always noted by waiting journos for the sartorial statements
they were making, rather than statements on more vital issues relating to the welfare of
women at large.
C. We have always had a few truly extraordinary women parliamentarians but their voices have
rarely been heard as emphatically as needed.
D. Given the small numbers this time, I really dont know how strong or effective a body these
ladies will make in parliament or even if they will speak in a cohesive voice.
E. Whether it is Jayaprada, Priya or any other presentable M.P. references to their physical
appearance are still obligatory.

OPTIONS
1) CBEDA
2) DACBE
3) DABEC
4) ACBED
5) BEDAC

Question of the Day #151: (10-Sep-09)


A certain number k has 8 factors. Which among the following cannot be the possible number of
factors of k3?
OPTIONS
1)
2)
3)
4)
5)

22
27
40
64
None of these

Book 2- Question of the Day

83

www.TestFunda.com

The LEADER in Online Test Prep

Question of the Day #152: (11-Sep-09)


The question below consists of two capitalized words that have a certain relationship to each other,
followed by a certain pair of words. Choose the pair that is RELATED to each other in the same way
as the capitalized pair.
OPTIONS
1)
2)
3)
4)
5)

Evolution : Transformation
Inducible : Deducible
Analogous : Differentiated
Affect : Affectation
Transcendental : Metaphysical

Question of the Day #153: (12-Sep-09)


If the average of 30 positive and distinct integers is 12410, what is the maximum possible value any
one of the 30 integers can take given that 12381 is the least of the 30 integers?
OPTIONS
1)
2)
3)
4)
5)

12975
12445
12845
12753
None of these

Question of the Day #154: (13-Sep-09)


This question consists of two capitalized words that have a certain relationship to each other,
followed by a certain pair of words. Choose the pair that is RELATED to each other in the same way
as the capitalized pair.
PREVARICATE: EQUIVOCATE
OPTIONS
1)
2)
3)
4)
5)

Reschedule : Resume
Reorganize : Regulate
Procastinate : Protract
Obviate : Elucidate
Advocate : Abandon

Question of the Day #155: (14-Sep-09)


The number of rectangles with integral sides, whose area is 5n and 7n square units are 9 and 12,
respectively. How many rectangles can be formed with integral sides whose area is 35n square units
if n is a natural number and 35n is a perfect square but 5n and 7n are not perfect squares?
OPTIONS
1)
2)
3)
4)
5)

15
14
21
216
None of these

Book 2- Question of the Day

84

www.TestFunda.com

The LEADER in Online Test Prep

Question of the Day #156 (15-Sep-09)


The question below contains a paragraph followed by alternative summaries. Choose the option that
best captures the essence of the text.
Among Yeats's dominant images are Leda and the Swan; Helen and the burning of Troy; the Tower in
its many forms; the sun and moon; the burning house; cave, thorn tree, and well; eagle, heron, sea
gull, and hawk; blind man, lame man, and beggar; unicorn and phoenix; and horse, hound, and boar.
Yet these traditional images are continually validated by their alignment with Yeats's own personal
experience, and it is this that gives them their peculiarly vital quality. In Yeats's verse they are often
shaped into a strong and proud rhetoric and into the many poetic tones of which he was the master.
All are informed by the two qualities which Yeats valued and which he retained into old age-passion
and joy.
OPTIONS
1) The symbolism of Yeatss poetry arose from his personal experiences; his passion and joy
gave his poetry its peculiar vitality.
2) In Yeatss poetry, traditional symbols achieved peculiar vitality in alignment with his own
experience; his many poetic tones expressed his joy and passion.
3) Yeats gave traditional images a peculiar vital quality; the many poetic tones that he was the
master of demonstrated passion and joy the two qualities he retained into old age.
4) The symbolism of Yeatss poetry was based on traditional images with a peculiar vital
quality; his readers did not miss the passion and the joy that he valued and retained into old
his age.
5) Yeats infused traditional symbols with vitality by aligning them with his own experiences; his
strong and proud rhetoric revealed his own passion and joy.

Question of the Day #157: (16-Sep-09)


Which of the following numbers gives the largest remainder when divided by 101?
OPTIONS
1) 123456789123
2) 231456789231
3) 312456789312
4) 213456789213
5) All of the above give the same remainder.

Book 2- Question of the Day

85

www.TestFunda.com

The LEADER in Online Test Prep

Question of the Day #158: (17-Sep-09)


The question below consists of a paragraph in which the first and last sentences are identified.
Choose the option that has the most logical order of the intermediate sentences.
A. As the G20 summit showed, we typically regulate in the midst of a bust.
B. Ironically, faith in draconian regulation is strongest at the bottom of the cycle, when there is little
need for participants to be regulated.
C. By contrast, the misconception that markets will take care of themselves is most widespread at
the top of the cycle, at the point of most danger to the system.
D. But we reform under the delusion that the regulated, and the markets they operate in, are static
and passive, and that the regulatory environment will not vary with the cycle.
E. We need to acknowledge these differences and enact cycle-proof regulation.
F. That is when righteous politicians feel the need to do something, bankers frail balance-sheets and
vivid memories make them eschew risk, and regulators have their backbones stiffened by public
disapproval of past laxity.
G. If we dont, there are many dangers.
OPTIONS
1)
2)
3)
4)
5)

BCEDF
DBCEF
BCDEF
FDBCE
FECDB

Question of the Day #159: (18-Sep-09)


Let x be a positive integer less than 100, such that x2 gives the same remainder as 100 does on
division by 19. How many such values can x take?
OPTIONS
1)
2)
3)
4)
5)

7
8
9
10
Cannot be determined

Book 2- Question of the Day

86

www.TestFunda.com

The LEADER in Online Test Prep

Question of the Day #160: (19-Sep-09)


The following question has a paragraph from which the last sentence has been deleted. From the
given options, choose the one that completes the paragraph in the most appropriate way.
Central to Kallats work is the relationship between text, images, traditional and contemporary
symbols. The urban milieu of Mumbai remains his primary muse; its nurturing of high glamour,
human suffering, simmering aggression and swarming masses an ongoing preoccupation. One of his
most ambitious installations till date- Aquasaurus, a seven-metre long water-tanker, fashioned from
bones, that morphs to become a macabre prehistoric creature for instance, personifies the radical
transformation of Indian city life. _____.
OPTIONS
1) I am liberalisations child; my worldviews are different from artists born twenty years
before me, explains Kallat.
2) If youre a remotely interesting artist, easy classification should be difficult, Kallat says.
3) There was a certain shifting in some of the known territories of painting, Kallat says.
4) Rules were being dismantled and I knew I could make a value addition as a young artist,
Kallat says.
5) Your work is largely what you breathe in, explains Kallat.

Question of the Day #161: (20-Sep-09)


A rectangular floor with integer sides (not necessarily distinct) has to be covered with square tiles of
unit side length. The tiles at the edges are purple and tiles in the interior are blue in colour. If the
number of blue tiles used, is equal to the number of purple tiles used, which of the following cannot
be the side length of the rectangular floor?
OPTIONS
1)
2)
3)
4)
5)

12 units
8 units
6 units
5 units
16 units

Book 2- Question of the Day

87

www.TestFunda.com

The LEADER in Online Test Prep

Question of the Day #162: (21-Sep-09)


The question below contains a paragraph followed by alternative summaries. Choose the option that
best captures the essence of the text.
The Quantity theory of money is the foundation stone of Monetarism. The theory says that the
quantity of money available in an economy determines the value of money. Increases in the money
supply are the main cause of inflation. The theory is built on the Fisher equation, MV = PT, named
after Irving Fisher (18671947). M is the stock of money, V is the Velocity of circulation, P is the
average price level and T is the number of transactions in the economy. The quantity theory, in its
purest form, assumes that V and T are both constant, at least in the short-run. Thus any change in M
leads directly to a change in P. In other words, increase the money supply and you simply cause
inflation.
OPTIONS
1) Monetarism is based on the Fisher equation which assumes that the stock of money is equal
to the price level. Hence, an increase in money supply causes inflation.
2) The Fisher equation assumes that the stock of money directly affects the average price level;
hence any increase in money supply causes inflation.
3) The quantity theory assumes that circulation of money is equal to the number of
transactions; hence if money supply is increased inflation must result.
4) The quantity theory is based on the Fisher equation which assumes that the stock of money
is equal to the price level; hence, increase in money supply causes inflation.
5) The Fisher equation which assumes that the stock of money is equal to the price level;
hence, increase in money supply causes inflation.

Question of the Day #163: (22-Sep-09)

B = 364 1.

OPTIONS
1)
2)
3)
4)
5)

(364)1
(2 363)1
(363)1
(363 + 364)1
(2 363 + 2 364)1

Book 2- Question of the Day

88

www.TestFunda.com

The LEADER in Online Test Prep

Question of the Day #164: (23-Sep-09)


The question below contains a paragraph followed by alternative summaries. Choose the option that
best captures the essence of the paragraph.
Accounting provides information for several purposes through the maintenance of data, the analysis
and interpretation of these data, and the preparation of various kinds of reports. Most accounting
information is historical- that is, the accountant observes all activities that the organization
undertakes, records their effects, and prepares reports summarizing what has been recorded; the
rest consists of forecasts and plans for current and future periods. Accounting information can be
developed for any kind of organization, not just for privately owned, profit-seeking businesses. One
branch of accounting deals with the economic operations of entire countries.
OPTIONS
1) Accounting is the systematic development and analysis of information about the economic
affairs of an organization for the purpose of forecasts and planning.
2) Accounting is the systematic development and analysis of information about the economic
affairs of an organization and even nations for several purposes.
3) Accounting provides systematic analysis of information about the economic affairs of an
organization for the purpose of forecasts and planning.
4) Accounting is the systematic development and analysis of information about the economic
affairs of organizations including countries.
5) Accounting provides organizations and countries with information about the economic
affairs for the purpose of forecasts and planning.

Question of the Day #165: (24-Sep-09)


If X and Y are natural numbers with no common prime factor and Z is the greatest common divisor of
(X + Y) and (X2 + Y2) then how many values can Z take?
OPTIONS
1)
2)
3)
4)
5)

0
1
2
3
Cannot be determined

Book 2- Question of the Day

89

www.TestFunda.com

The LEADER in Online Test Prep

Question of the Day #166: (25-Sep-09)


The decades-old "hygiene hypothesis" holds that early exposure to microbes somehow challenges
the immune system and strengthens it against allergies. Studies have shown children exposed to
disease-causing microorganisms by older siblings or attending nursery cut their future allergy risk.
Which of the following, if true, would most seriously challenge the conclusion of the studies?
OPTIONS
1) Children in day care usually get more colds and other infections.
2) Children who grow up on farms are less likely to develop allergies like hay fever and asthma.
3) Children with frequent chest infections and wheezing had older siblings and had attended
nursery in early life.
4) Children who went to nursery and who had older siblings had greatly reduced the risk of
frequent chest infections and wheezing in early life.
5) In susceptible children, something in their environment triggers infections and allergies.

Question of the Day #167: (26-Sep-09)

then which of the following is equal to the value of A?


OPTIONS
1)
2)
3)
4)
5)

Book 2- Question of the Day

90

www.TestFunda.com

The LEADER in Online Test Prep

Question of the Day #168: (27-Sep-09)


Ahead of the forthcoming Copenhagen climate talks, the UK government asked the Climate Change
Committee to advise on what should be done about emissions from aviation in the UK. The
committee estimated that the aviation industry will have to cut emissions from planes back to their
2005 level by 2050. That is much more permissive than the overall UK target of cutting emissions
80% on 1990 levels by 2050.
Based on the information given in the paragraph above, which of the following options is most likely
to be true?
OPTIONS
1) It might be easier for the aviation industry to make the leap to zero carbon emissions rather
than trying to reduce it by 80%.
2) The overall UK target is to cut emissions by 2050 to less than 80% of the 1990 levels.
3) Some sectors may have to cut emissions by 90% by 2050 so the aviation sector can continue
to grow.
4) The aviation sector in the UK contributes 80% of the overall emissions.
5) An 80% reduction in overall emissions will impact the performance of the Aviation industry
in the UK.

Question of the Day #169: (28-Sep-09)


Amar has 4 cubes each with a distinct number from 1 to 4 on it.The cubes are arranged in a single
line in any order. He wants them placed in ascending order from his left to his right. To do this, he
takes the leftmost cube and places it rightmost. He then takes the third cube from the right and
places it rightmost.
What was the initial order in which the cubes were kept?
OPTIONS
1) 4-3-1-2
2) 4-1-3-2
3) 2-4-3-1
4) 3-1-4-2
5) None of these

Book 2- Question of the Day

91

www.TestFunda.com

The LEADER in Online Test Prep

Question of the Day #170: (29-Sep-09)


The question below contains a number of sentences. Each sentence has pairs of word(s)/phrases
that are highlighted. From the highlighted word(s)/phrase(s), select the most appropriate
word(s)/phrase(s) to form correct sentences. Then, from the options given, choose the best one.
1. How can you proscribe (A) / prescribe for (B) others, when you are unable to heal yourself?
2. His silence is tantamount to (A) / paramount with (B) an admission of guilt
3. The critics have degraded their (A) / denigrated our (B) efforts.
4. Our professor is very amiable and (A) / amenable and (B) does not get easily annoyed.
OPTIONS
1) ABBA
2) BABA
3) ABAA
4) BAAB
5) BBAA

Question of the Day #171: (30-Sep-09)


Quadratic functions f(x) = x2 + ax + b and g(x) = x2 + cx + d are defined for real values of x such that
f(x) < 0 for all values of x, except x = 2 and g(x) > 0 for all values of x except 2 x 3. If h(x) = max
[f(x), g(x)] then for what value of x is the value of the function h(x) minimum?
OPTIONS
1) 2
2) 0
3)
4)
5) None of these

Question of the Day #172: (01-Oct-09)


From the following words, identify the word that will make a relationship for the third word that is
the most similar to the relationship between the first pair.
Poltergeist: Apparition:: Balderdash:
OPTIONS
1)
2)
3)
4)
5)

Moonshine
Alacrity
Sunshine
Avidity
Riposte

Book 2- Question of the Day

92

www.TestFunda.com

The LEADER in Online Test Prep

Question of the Day #173: (02-Oct-09)


Functions f(x) and g(y) are defined for natural numbers x and y as follows.

If z is any real number, [z] denotes the largest integer less than or equal to z.
Which of the following can never be the value of the product f(x) g(y)?
OPTIONS
1) 0
2) 1
3) 6
4) 8
5) 12

Question of the Day #174: (03-Oct-09)


Each of the questions below contains a number of sentences. Each sentence has pairs of
word(s)/phrases that are highlighted. From the highlighted word(s)/phrase(s), select the most
appropriate word(s)/phrase(s) to form correct sentences. Then, from the options given, choose the
best one.
i.
I have a turgid (A) / turbid (B) limb.
ii.
The award winning actors performance on the screen was exceptional (A) / exceptionable
(B).
iii.
The animal looked like a young deer and was fawn (A) / faun (B) in colour.
iv.
For years, 'experts' have told cat owners that domestic cats are insocial (A) / unsociable (B)
creatures that dislike the company of other cats.
v.
The media has reported several times about the venial (A) / venal (B) arrangement that the
citys mafia has with the police.
OPTIONS
1)
2)
3)
4)
5)

AABBB
BAABB
ABAAB
AAABB
BABAB

Book 2- Question of the Day

93

www.TestFunda.com

The LEADER in Online Test Prep

Question of the Day #175: (04-Oct-09)


If a2 + b2 + c2 = p2 + q2 + r2 = 101, where a, b, c, p, q and r are all distinct real numbers, then which of
the following inequalities is true?
OPTIONS
1)
2)
3)
4)
5)

ap + bq + cr < 99
ap + bq + cr < 101
ap + bq + cr < 202
ap + bq + cr < 200
None of these

Question of the Day #176: (05-Oct-09)


Generally, income and education are positively linked to better lives. However, taller people live
better lives, according to US researchers. More than 454,000 adults aged 18 and over were asked by
phone for their height and their evaluation of their lives. Overall, taller individuals judged their lives
more favourably and were more likely to report positive emotions such as enjoyment and happiness.
Hence, there does appear to be a correlation between height and happiness.
Which of the following weakens the above argument the most?
OPTIONS
1)
2)
3)
4)
5)

Taller people also had higher incomes and education.


The people who are the happiest are not the very tallest.
There is a direct correlation between income and happiness.
There is a threshold of height tolerance; beyond that life becomes very difficult.
Taller men are more prone to experience irritation and anger.

Book 2- Question of the Day

94

www.TestFunda.com

The LEADER in Online Test Prep

Question of the Day #177: (06-Oct-09)

What is the range of x if A B C?


OPTIONS

1)
2)
3)
4)
5) None of these

Question of the Day #178: (07-Oct-09)


The law of supply and demand tells you that increasing the quantity of something tends to reduce its
price. But not, it seems, in management education. Every year with scores of new B-schools coming
up, people wonder if this will be the year that management education starts to lose its valueand
every year; there is no sign of it happening.
Which of the following helps resolve the apparent paradox in relation to management education and
its demand?
OPTIONS
1) There are over 750 B-Schools in India and their fees are on the increase every year.
2) Some of the premier B-schools fetches phenomenal salaries to students even though they
do not deserve that much.
3) Corporates reward management students well for the time and money they invest in
education.
4) The government has slashed the funding of most b-schools and this has caused the fees to
increase.
5) With many graduates to choose from, employers increasingly reject anyone who does not
sport a degree, no matter what the jobs requirements.

Book 2- Question of the Day

95

www.TestFunda.com

The LEADER in Online Test Prep

Question of the Day #179: (08-Oct-09)

and P < Q + R, which of the following gives the range of values that x can take?
OPTIONS
1)
2)
3) (3.5, 3) (3, 3.5)
4) (3.5, 3) (3, 6)
5) None of these

Question of the Day #180: (09-Oct-09)


Female MBAs tend to earn less than their male colleagues. Female MBA graduates not only earn
significantly less than their male colleagues, but the gap grows over time, according to a paper from
the National Bureau of Economic Research. The authors tracked the average earnings of 1,600 MBAs
who graduated from the University of Chicagos Graduate School of Business between 1990 and
2009.
Which of the following best supports the argument above?
OPTIONS
1) 13% of the women MBAs in the study were not working at all, ten years after graduating,
while the corresponding figure for men was a mere 1%.
2) The difference between the average salary of male and female MBAs is skewed by a small
number of extremely high earners among women.
3) The difference between the average salary of male and female MBAs is skewed by a large
number of extremely low earners among men.
4) Business school admissions are less competitive for women than it is for men.
5) Of the 1600 MBAs only 75% constituted men.

Book 2- Question of the Day

96

www.TestFunda.com

The LEADER in Online Test Prep

Question of the Day #181: (10-Oct-09)

Find the value of f2009(2009).


OPTIONS
1)
2) 2009
3) 2008
4) 1004
5)

Question of the Day #182: (11-Oct-09)


Studies indicate the need to screen cancer patients carefully for signs of psychological distress. The
study found death rates were up to 25% higher in patients showing symptoms of depression. In
patients actually diagnosed with major or minor depression, death rates were up to 39% higher.
The authenticity of the above study depends on which of the following (assumptions)?
OPTIONS
1)
2)
3)
4)
5)

Diagnosis of a depressive disorder can predict mortality in cancer patients.


Other clinical characteristics that might affect survival were taken into consideration.
Depression is more life threatening than cancer itself.
Depression impacted the progression of cancer.
More research will be needed to explain whether these observations are true and if so why.

Book 2- Question of the Day

97

www.TestFunda.com

The LEADER in Online Test Prep

Question of the Day #183: (12-Oct-09)


A is a number of the form PN, where P is a prime number and N is a natural number. X is the product
of all the positive integral factors of A. Which of the following could be a value of

OPTIONS
1)
2)
3)
4)
5)

14
28
112
56
None of these

Question of the Day #184: (13-Oct-09)


Critics argue that surveillance and data aggregation technologies -'dataveillance', as some
researchers have defined it- do not deter criminals or help solve crimes and puts privacy at stake. A
report into London's surveillance network found that in 2008 more than a million cameras helped to
solve just 1000 crimes. That is one crime for every thousand cameras. Civil Liberties Union said
innocent people should not be watched. "If you are a law-abiding citizen and no one has any reason
to believe that you are doing anything wrong, law enforcement should not be watching you".
Which of the following most seriously undermines the argument above?
OPTIONS
1)
2)
3)
4)
5)

The law enforcement agencies are just doing their job.


Criminals are caught a lot faster with the help of the surveillance cameras.
Surveillance and cases would take more personnel in the absence of surveillance cameras.
Police officers and surveillance cameras are placed in localities prone to crime.
Surveillance cameras monitor areas that are not patrolled because officers may be on other
assignments.

Question of the Day #185: (14-Oct-09)


X = (10000 10001 10002 10003) + 1 and X = Y2, where Y is a natural number.
What is the value of Y?
OPTIONS
1)
2)
3)
4)
5)

100000001
102030201
102000201
103000301
None of these

Book 2- Question of the Day

98

www.TestFunda.com

The LEADER in Online Test Prep

Question of the Day #186: (15-Oct-09)


Can a husband open his wife's letters? It has recently been decided in a Paris tribunal that the
husband has the right to open the letters addressed to his wife. The Paris decision must be based
upon the familiar formula that man and wife are one, and that that one is the husband. If a man has
the right to read all the letters written to his wife, being his property by reason of his ownership of
her, why may he not have a legal right to know all that is said to her? The question is not whether a
wife ought to receive letters that her husband may not read, or listen to talk that he may not hear,
but whether he has a sort of lordship that gives him privileges which she does not enjoy.
What can you conclude from the above passage?
OPTIONS
1)
2)
3)
4)
5)

The wife has no interest in her husbands personal life.


The husband can keep tabs on his wifes personal correspondence.
The wife does not have the right to have a private life.
Marriage is a male dominated relationship.
Letters are legal documents that must be opened only by the person it is addressed to.

Question of the Day #187: (16-Oct-09)


We have D = x3 + y3 (x y)(x2 + 4y2) for real, positive and unequal values of x and y. Which of these
is true?
OPTIONS
1) D > 0
2) D = 0 for three values of (x, y)
3) D < 0
4) D = 0 for two values of (x, y)
5) D may be greater than or less than 0 depending on the values of x and y

Book 2- Question of the Day

99

www.TestFunda.com

The LEADER in Online Test Prep

Question of the Day #188: (17-Oct-09)


Each of the questions below contains a number of sentences. Each sentence has pairs of
word(s)/phrases that are highlighted. From the highlighted word(s)/phrase(s), select the most
appropriate word(s)/phrase(s) to form correct sentences. Then, from the options given, choose the
best one.
1. In the absence of a queue, we could not find out whos (A) / whose (B) turn came next.
2. At the funeral, several wreathes (A) / wreaths (B) were placed on the grave of the deceased.
3. During his speech he made the ironic (A) / sardonic (B) observation that the politicians could
always be trusted.
4. The judges were uninterested (A) / disinterested (B) in the outcome of the case.
5. The administrative division of India is composed (A) / comprises (B) of 28 states and 7 union
territories.
OPTIONS
1)
2)
3)
4)
5)

ABABB
BBBAA
BBABA
BABAB
ABBAB

Question of the Day #189: (18-Oct-09)


If X = 26863 6863 15003 5003
then which of the following is not a factor of X?
OPTIONS
1)
2)
3)
4)
5)

3
2000
1186
2186
None of these

Book 2- Question of the Day

100

www.TestFunda.com

The LEADER in Online Test Prep

Question of the Day #190: (19-Oct-09)


Each of the questions below contains a number of sentences. Each sentence has pairs of
word(s)/phrases that are highlighted. From the highlighted word(s)/phrase(s), select the most
appropriate word(s)/phrase(s) to form correct sentences. Then, from the options given, choose the
best one.
1. The prerequisites (A) / perquisites (B) of this job include health insurance and a
performance bonus.
2. I am always ready to take the risk of being tedious in order to be sure that I am perspicuous
(A) / perspicacious (B).
3. My principle (A) / principal (B) sells steel forged items.
4. Marijuana is prescribed (A) / proscribed (B) in the U.S because it is a deadly narcotic.
5. It was regretful (A) / regrettable (B) that the teacher made the class retake the test when he
was wrong about the answers.
OPTIONS
1) BABAA
2) ABBBA
3) BBBBA
4) BABBB
5) AAABB

Question of the Day #191: (20-Oct-09)


X, Y and Z are three distinct natural numbers in Harmonic Progression. Which of the the following
values is a possible value of (X + Z)/Y?
OPTIONS
1) 0
2) 0.5
3) 1
4) 2
5) None of these

Book 2- Question of the Day

101

www.TestFunda.com

The LEADER in Online Test Prep

Question of the Day #192: (21-Oct-09)


The laws of Rome were very strict in regard to associations, which, formed on the pretense of
amusement, charity, or athletic sports, were apt to degenerate into political sects. Exception was
made in favor of the collegia funeraticia, which were societies formed to provide a decent funeral
and place of burial for their members. An inscription discovered at Civita Lavinia quotes the very
words of a decree of the Senate on this subject: "It is permitted to those who desire to make a
monthly contribution for funeral expenses to form an association." "These clubs or colleges collected
their subscriptions in a treasure-chest, and out of it provided for the obsequies of deceased
members. Funeral ceremonies did not cease when the body or the ashes was laid in the sepulchre. It
was the custom to celebrate on the occasion a feast, and to repeat that feast year by year on the
birthday of the dead, and on other stated days. For the holding of these feasts, as well as for other
meetings, special buildings were erected, named scholae and when the societies received gifts from
rich members or patrons, the benefaction frequently took the shape of a new lodge-room, or of a
ground for a new cemetery, with a building for meetings." The Christians took advantage of the
freedom accorded to funeral colleges, and associated themselves for the same purpose, following as
closely as possible their rules concerning contributions, the erection of lodges, the meetings, and the
love feasts; and it was largely through the adoption of these well-understood and respected customs
that they were enabled to hold their meetings and keep together as a corporate body through the
stormy times of the second and third centuries.
Based on the above passage, which of the following is most likely to be true?
OPTIONS
1) For the populace of Rome, getting involved in amusement, charity or athletic activities was
dangerous.
2) Funeral colleges were responsible for funeral ceremonies - the family was not.
3) The raison d'etre for funeral colleges was the lack of places where people could meet and
interact.
4) The emulation of funeral colleges by Christians allowed them to survive beyond the second
and third centuries.
5) Funeral colleges evolved from being socio-political associations to socio-religious bodies.

Book 2- Question of the Day

102

www.TestFunda.com

The LEADER in Online Test Prep

Question of the Day #193: (22-oct-09)


Ari, the king of Egypina, is very vain
Wishes all his soldiers be called by his own name!
Dexter, his minister, finds this ludicrous,
And to the relief of the soldiers,
Manages a fine bargain.
Your highness, all soldiers will have an A in their names, says he,
The initial letter of your name that A happens to be.
Amen, but no name will have more letters than mine has the king bellows!
Else, find yourself, Dexter, right at the gallows!
If all soldiers in Aris army have unique names, how many soldiers can the army have at the most?
OPTIONS
1)
2)
3)
4)
5)

2003
2002
1951
703
None of these

Question of the Day #194: (23-Oct-09)


A. The boy who cried wolf is an especially apposite (A)/ opposite (B) story, which we can use
to explain the importance of being truthful.
B. His receding (A)/ reducing (B) hairline made him uncomfortable.
C. Rigour (A)/ Rigor (B) mortis had set it almost five hours ago.
D. Even after all the tests we were unable to determine what ailed (A)/ weakened (B) the old
priest.
E. The poor clerk was inundated with words like lien (A)/ lean (B) on goods, mortgage and
inter-alia on his first day in court.
OPTIONS
1)
2)
3)
4)
5)

AAAAA
AAABA
BAABA
AABAA
BABBA

Book 2- Question of the Day

103

www.TestFunda.com

The LEADER in Online Test Prep

Question of the Day #195: (24-Oct-09)

Then what is the value of y1 + y2 + y3 + y4 + y5 + y6 + y7 + y8?


OPTIONS
1)
2)
3)
4)
5)

459
259
267
467
None of these

Question of the Day #196: (25-Oct-09)


The question below has five word pairs. Only one of the words in a word pair fits the context of the
sentence correctly. Find the option which enlists all the correct words.
The (A) adoptive/adopted (B) parents (A) preceded/proceeded (B) to file their application to adopt
another child. They had (A) born/borne (B) the taunts of their relatives for a long time before they
had come to the decision to go for adoption. Even then, this (A) alternate/alternative (B) was seen
by the society as an act of (A) bravado/bravery (B) rather than a natural desire to bring up an
individual.
OPTIONS
1)
2)
3)
4)
5)

ABBBA
BBBBA
ABABB
BBAAB
ABABA

Book 2- Question of the Day

104

www.TestFunda.com

The LEADER in Online Test Prep

Question of the Day #197: (26-Oct-09)

which of the following intervals best describes the range of K?


OPTIONS
1)
2) (1, )
3)
4) *1, )
5)

Question of the Day #198: (27-Oct-09)


The question below has five word pairs. Only one of the words in a word pair fits the context of the
sentence correctly. Find the option which enlists all the correct words for a question.
She always felt that she was (A) inflicted/afflicted (B) by some malady or the other. She would often
(A) elude/allude (B) to her so-called insufferable imagined illnesses to her visitors. She was such a
good storyteller that her visitors were horrified listening to her endless (A) forebodings/forbidding
(B) about future health concerns. She also had (A) excess/access (B) to all kinds of medicine
information that she would want to discuss with her doctor daily.
OPTIONS
1)
2)
3)
4)
5)

ABAB
BBAB
BBAA
AAAB
BBBA

Question of the Day #199: (28-Oct-09)


How many divisors of the number 1! 2! 3! 9! 10! are perfect squares?
OPTIONS
1)
2)
3)
4)
5)

18088
2160
14592
912
None of these

Book 2- Question of the Day

105

www.TestFunda.com

The LEADER in Online Test Prep

Question of the Day #200: (29-Oct-09)


The question below has five word pairs. Only one of the words in a word pair fits the context of the
sentence correctly. Find the option which enlists all the correct words.
One good thing about his jokes was that they were not (A) backwards/backward (B). They were
spontaneous and witty (A) antidotes/anecdotes (B) superbly created from real-life incidents.
Whether you were (A) aggravated/annoyed (B) or feeling (A) bad/badly (B) about something, his
jokes could blow away your gloom. How he could so easily (A) elicit/illicit (B) laughter from people
still remains a mystery for me.
OPTIONS
1)
2)
3)
4)
5)

ABBAA
ABABA
BAABA
BBBAA
BBBAB

Question of the Day #201: (30-Oct-09)


How many ordered pairs (x, y), such that x and y are positive integers, satisfy the equation x2 18x
1 = 2y 34?
OPTIONS
1)
2)
3)
4)
5)

0
1
2
3
More than 3

Question of the Day #202: (31-Oct-09)


Identify the incorrect sentence or sentences.
A. All the facts thus predicted by the theory is confirmed by experiment.
B. There is no known process which permit the direct measurement of the mass of an electron.
C. In the case of the cathode rays emitted by radium, these measurements are particularly
interesting, for the reason that the rays which compose a pencil of cathode rays are
animated by very different speeds.
D. Professor Kaufmann has effected some very careful experiments by the method of crossed
spectra.
E. He has thus been enabled by working in vacuo to register the very different velocities which,
starting in the case of certain rays from about seven-tenths of the velocity of light, attain in
other cases in ninety-five hundredths of it.
OPTIONS
1)
2)
3)
4)
5)

A, B, D and E
A, B and E
Only C
A, B and C
D and E

Book 2- Question of the Day

106

www.TestFunda.com

The LEADER in Online Test Prep

Question of the Day #203: (01-Nov-09)


A circle with radius r and centre O is drawn such that a circle with center O and radius r (< r) is
tangent to it at a point P. There is a point A on the larger circle (distinct from P) such that chords AX
and AY are tangent to the smaller circle at B and C respectively. If BPC = 72, what is the measure
of XPY?
OPTIONS
1)
2)
3)
4)
5)

108
72
112
144
120

Question of the Day #204: (02-Nov-09)


Identify the correct sentence/ sentences
A. I was very sleepy and frightened on that damp chilly morning.
B. I wasn't quite sure I liked the idea.
C. But the sharp morning air, interest in training a new motor-cycle in the way it should go, the
unexpected popping-up and grotesque salutes of Boy Scouts soon made me forget the war.
D. The little breakdowns delayed us considerably.
E. My friends rode joyfully from one end of the platform to the other, much to the agitation of
the guard.
OPTIONS
1)
2)
3)
4)
5)

A, B and E
B, C, D and E
A, B, D and E
A, B and C
Only C

Question of the Day #205: (03-Nov-09)

What is the total number of integral values of x that satisfies the given inequality?
OPTIONS
1)
2)
3)
4)
5)

40
52
54
45
51

Book 2- Question of the Day

107

www.TestFunda.com

The LEADER in Online Test Prep

Question of the Day #206: (04-Nov-09)


This book comprises (A) / consists (B) of five chapters. I was confidant (A) / confident (B) that I
would be able to complete them in five days. Despite (A) / In spite (B) of my will I could manage only
three. This was because of the dinning (A) / dining (B) noise that the machines used for construction
were making. Presently (A) / Currently (B) I am on my fourth chapter.
OPTIONS
1) ABBAB
2) BBBAB
3) ABAAB
4) BBAAA
5) ABBAA

Question of the Day #207: (05-Nov-09)


P is a point in the ABC such that PAC = PCB = PBA = .
If AB = 12 cm, BC = 10 cm and AC = 14 cm, then tan =
OPTIONS

1)
2)
3)
4)
5) Cannot be determined

Question of the Day #208: (06-Nov-09)


The question below contains a paragraph with a missing sentence or part of a sentence. Choose the
option that most logically completes the paragraph.
The story of Georges life is a series of failures. Being unable to continue at the University, George
began work at a stock brokers in the City. These were the days prior to the great depression. While
George struggled at learning the ropes of his trade, dark clouds began forming on the financial
horizon. ______.
OPTIONS
1)
2)
3)
4)
5)

And in mid 1929, the market bottomed out leaving George and millions others jobless.
Suddenly, there was a shower of troubles for George.
Around the same time George was happily married to Celia.
As a consequence, George began worrying about his career.
The future was certainly not bright.

Book 2- Question of the Day

108

www.TestFunda.com

The LEADER in Online Test Prep

Question of the Day #209: (07-Nov-09)


Let n be an integer such that

Compute the remainder when n is divided by 17.


OPTIONS
1)
2)
3)
4)
5)

16
7
8
9
None of these

Question of the Day #210: (08-Nov-09)


Each of the questions below contains a number of sentences. Each sentence has pairs of
word(s)/phrases that are highlighted. From the highlighted word(s)/phrase(s), select the most
appropriate word(s)/phrase(s) to form correct sentences. Then, from the options given, choose the
best one.
1. His was a life devoted to and wasted in mere sensual (A) / sensuous (B) pleasures.
2. Spent in enjoyments of extravagance, pleasure, or sensual gratifications he had a long and
luxurious (A) / voluptuous (B) holiday.
3. Teenagers are often veracious (A) / voracious (B) eaters.
4. They have a tertiary (A) / ternary (B) number system, because of the importance they give
to the number three.
5. He is one of Indias premier (A) / premiere (B) industrialists.
OPTIONS
1)
2)
3)
4)
5)

AABBA
BABAB
ABBAB
ABBBA
BABBB

Book 2- Question of the Day

109

www.TestFunda.com

The LEADER in Online Test Prep

Question of the Day #211: (09-Nov-09)


What is the radius of a circle which passes through (0, 0) and (5/2, 1/2) and has the line 3x 2y = 0
as one of its diameters?
OPTIONS
1) 13/2
2) 13/4
3)
4)

5)

Question of the Day #212: (10-Nov-09)


The question below contains a number of sentences. Each sentence has pairs of word(s)/phrases
that are highlighted. From the highlighted word(s)/phrase(s), select the most appropriate
word(s)/phrase(s) to form correct sentences. Then, from the options given, choose the best one.
1. Discreet (A) / discrete (B) followers and servants, help much to reputation.
2. She finds it necessary and has sometimes felt it expedient (A) / expeditious (B) to excuse
her husband for lacking in sense.
3. The professors remark that the questions were eminently (A) / imminently (B) solvable left
the students mocking at him.
4. It is the Prime Ministers prerogative to elect (A) / select (B) members of the cabinet.
OPTIONS
1)
2)
3)
4)
5)

ABBA
BBBA
AAAB
BAAB
AABB

Book 2- Question of the Day

110

www.TestFunda.com

The LEADER in Online Test Prep

Question of the Day #213: (11-Nov-09)


The question below contains a paragraph followed by alternative summaries. Choose the option that
best captures the essence of the paragraph.
Accounting provides information for several purposes through the maintenance of data, the analysis
and interpretation of these data, and the preparation of various kinds of reports. Most accounting
information is historical- that is, the accountant observes all activities that the organization
undertakes, records their effects, and prepares reports summarizing what has been recorded; the
rest consists of forecasts and plans for current and future periods. Accounting information can be
developed for any kind of organization, not just for privately owned, profit-seeking businesses. One
branch of accounting deals with the economic operations of entire countries.
OPTIONS
1) Accounting is the systematic development and analysis of information about the economic
affairs of an organization for the purpose of forecasts and planning.
2) Accounting is the systematic development and analysis of information about the economic
affairs of an organization and even nations for several purposes.
3) Accounting provides systematic analysis of information about the economic affairs of an
organization for the purpose of forecasts and planning.
4) Accounting is the systematic development and analysis of information about the economic
affairs of organizations including countries.
5) Accounting provides organizations and countries with information about the economic
affairs for the purpose of forecasts and planning.

Question of the Day #214: (12-Nov-09)


The question below has five word pairs. Only one of the words in a word pair would fit the context of
the sentence correctly. Find the option which enlists all the correct words for a question.
Archeologists have found books embedded in an (A) implicated/ imbricated (B) fashion under the
broken castle. There were earlier no (A) extant/extent (B) copies of these books. The books narrate
tales about life in that (A) diffident/difficult (B) era. People were being (A) hanged/hung (B) for no
reason at all. They also speak about (A) hardly/hardy (B) people who fought the injustices of their
time.
OPTIONS
1)
2)
3)
4)
5)

BABAB
AABAB
BAAAB
BABBB
BABAA

Book 2- Question of the Day

111

www.TestFunda.com

The LEADER in Online Test Prep

Question of the Day #215: (13-Nov-09)

What is the largest value that x can take?


OPTIONS
1)
2)
3)
4)
5)

Question of the Day #216: (14-Nov-09)


The question below has five word pairs. Only one of the words in a word pair would fit the context of
the sentence correctly. Find the option which enlists all the correct words for a question.
The man who (A) abetted/betted (B) the bank robber has been caught. Last he was (A)
interrogated/interviewed (B) he appeared quite (A) reticent/reluctant (B) and kept to himself. The
(A) taut/tout (B) -lipped man only (A) sited/cited (B) ignorance of the robbery when pushed.
OPTIONS
1)
2)
3)
4)
5)

AAABB
BAABB
AAAAB
AAAAA
BBBAA

Book 2- Question of the Day

112

www.TestFunda.com

The LEADER in Online Test Prep

Question of the Day #217: (15-Nov-09)


For how many positive integers m, is the following inequality true?

OPTIONS
1)
2)
3)
4)
5)

69
68
34
35
None of these

Question of the Day #218: (16-Nov-09)


The question below has five word pairs. Only one of the words in a word pair would fit the context
of the sentence correctly. Find the option which enlists all the correct words for a question.
The psychopath read the thriller with (A) moribund/morbid (B) interest. It thrilled him when he
read about the murderer puncturing the (A) palate/palette (B) of the (A) peasant/pheasant (B). He
read with more interest as he read about more homicides. But as he (A) pored/poured (B) over the
book, he came to a part where the murderer was caught and was (A) persecuted/prosecuted (B) in
a court of law. That part he did not like and he put the book down.
OPTIONS
1) BAAAB
2) BBBBB
3) AAAAB
4) ABAAB
5) AAABA

Question of the Day #219: (17-Nov-09)


Two friends A and B can finish a certain piece of work in 12 and 18 days respectively, while working
alone. But, when they work together, they start talking and their efficiency comes down by 20%. If
one of them works alone for a few days, then the second works alone for a few days and then the
both work together, the work gets over in 13 days. If the number of days for which A and B work
alone is a prime number then which of the following can be determined?
OPTIONS
1) The number of days for which A worked alone
2) The number of days for which B worked alone
3) The number of days for which A and B worked together
4) All of these
5) None of these

Book 2- Question of the Day

113

www.TestFunda.com

The LEADER in Online Test Prep

Question of the Day #220: (18-Nov-09)


Striving to revive the world economy while simultaneously responding to the global climate crisis has
raised a knotty question: are statistics giving us the right signals about what to do? In our
performance-oriented world, measurement issues have taken on increased importance: what we
measure affects what we do. If we have poor measures, what we strive to do, say, increase GDP,
may actually contribute to a worsening of living standards. We may also be confronted with false
choices, seeing trade-offs between output and environmental protection that dont exist. By
contrast, a better measure of economic performance might show that steps taken to improve the
environment are good for the economy.
The above argument leads to which of the following conclusions?
OPTIONS
1) We need a set of indicators that more accurately capture both well-being and sustainability.
2) GDP is a poor measure of well-being.
3) The demand to maximise GDP should be coupled with a demand to reduce air, water, and
noise pollution.
4) What is good for the economy is not always good for the environment.
5) Measure of how well we are doing takes account of sustainability.

Question of the Day #221: (19-Nov-09)


The question below is followed by two statements, A and B. Answer each question using the
following instructions:
Mark (1) if the question can be answered by using statement A alone but not by using statement B
alone.
Mark (2) if the question can be answered by using statement B alone but not by using statement A
alone.
Mark (3) if the question can be answered by using either of the statements alone.
Mark (4) if the question can be answered by using both the statements together but not by either of
the statements alone.
Mark (5) if the question cannot be answered on the basis of the two statements.
Is the integer x prime?
A. p! < x < (p + 1)!, where p is not a prime number.
B. p! + 2 x p! + p, where p is a prime number.
OPTIONS
1)
2)
3)
4)
5)

1
2
3
4
5

Book 2- Question of the Day

114

www.TestFunda.com

The LEADER in Online Test Prep

Question of the Day #222: (20-Nov-09)


It was found that puppy fetuses would react to touch and/or pressure from the outside of the
mothers abdomen. In addition, it is theorized that since puppies have such a well-developed sense
of touch at birth, the sense of touch would also be well-developed before birth. Studies have found
that "when a pregnant animal is petted, her litter is more docile". According to the study, this
facilitates relaxation, emotional attachment, and socialization. Other studies have indicated that
puppies that receive outside contact (petting of the mother) while in utero have a higher tolerance
for touching than puppies that receive no contact at all.
Based on the above information, what can be concluded about the state of a puppys sense of
touch?
OPTIONS
1) One could theorise that gentle petting of the mothers abdomen could help to facilitate
positive, beneficial puppy socialization with people.
2) One could theorise that puppies that are intolerant of touching have probably had a very
cruel time in utero
3) One could theorise that for puppies, there is an exponential relationship between tolerance
towards touching and socializing.
4) One could safely assume that the sense of touch is more keen in puppies than any other
sense
5) One could conclude from the given information that the mothers abdomen is especially
sensitive to touch and puppies shape their behaviour based on their mothers lifestyle.

Question of the Day #223: (21-Nov-09)

What is the value of X?


OPTIONS
1)
2)
3)
4)
5)

10
18
26
28
None of these

Book 2- Question of the Day

115

www.TestFunda.com

The LEADER in Online Test Prep

Question of the Day #224: (22-Nov-09)


UK researchers have found that a diet low in calcium can lead to an early onset of osteoporosis in
women. Which is why, men can get osteoporosis and should watch the level of calcium in their food.
The author's conclusion about men is based on the assumption that:
OPTIONS
1)
2)
3)
4)
5)

Low calcium may cause osteoporosis in men


Osteoporosis affects many people
Osteoporosis is caused by low calcium which is common among women
Since low calcium causes osteoporosis in women, it will have a similar effect on men too
Osteoposoris is a direct effect of low calcium

Question of the Day #225: (23-Nov-09)


An is the nth term of the series which satisfies the following condition,
An + 1 = An2 + 3An + 1

and

what is the least integer greater than S?


OPTIONS
1)
2)
3)
4)
5)

1
2
3
4
5

Book 2- Question of the Day

116

www.TestFunda.com

The LEADER in Online Test Prep

Question of the Day #226: (24-Nov-09)


Members of the fathers' rights movement state that feminist organizations invoke the spectre of
domestic violence as propaganda directed against fathers and fathers' rights groups. They point to
domestic violence studies based on the Conflict Tactics Scale (CTS), which show that men and
women act violently toward their partners in about equal percentages. They argue that men
comprise a "significant portion" of the victims of domestic violence, and they call for more services
to be provided for male victims of domestic violence.
Which of the following statements strengthens the argument presented in the passage?
OPTIONS
1) Researchers argue that the real numbers for violence against men are likely to be higher,
since male victims may be less likely to report abuse than female victims due to social
stigmatization.
2) The percentage of shelters for battered men is much more than the percentage of shelters
for women and children.
3) Many women's shelters assist male victims of domestic abuse but do not house men,
instead offering hotel vouchers, counselling, case management, legal services and other
support services.
4) Fathers rights groups feel that the feminist rights movement ignores the violence
perpetrated against men
5) Advocates cite government statistics that show that in 15% to 38% of the cases of violence,
the victim is a spouse.

Question of the Day #227: (25-Nov-09)


In ABC, AM and CN are medians to the sides BC and AB, respectively. If AM and CN are
perpendicular to each other, AM = 9 cm and CN = 6 cm, then BAC =
OPTIONS
1)
2)
3)
4)
5)

Book 2- Question of the Day

117

www.TestFunda.com

The LEADER in Online Test Prep

Question of the Day #228: (26-Nov-09)


All social animals have hierarchical societies in which each member knows its own place. Social order
is maintained by certain rules of expected behaviour and dominant group members enforce order
through punishment. However, higher order primates also have a sense of reciprocity and fairness.
Chimpanzees remember who did them favours and who did them wrong.
Which of the following statements strengthens the argument presented in the passage?
OPTIONS
1) Chimpanzees are more likely to share food with individuals who have previously groomed
them.
2) Humanitys closest living relatives are common chimpanzees and bonobos - and humans,
being the most intelligent animals on the planet, of course, have a highly developed sense of
reciprocity and fairness.
3) Though morality may be a unique human trait, many social animals, such as primates,
dolphins and whales, have been known to exhibit pre-moral sentiments.
4) All social animals have had to restrain or alter their behaviour for group living to be
worthwhile.
5) Like most behaviours that are found in societies throughout the world, social order must
have been present in the ancestors of the human population before the dispersal from
Africa.

Question of the Day #229: (27-Nov-09)


If (a + b + c) = d (> 0), a3 + b3 + c3 = 3abc,

Then X =
OPTIONS
1)
2)
3) 1
4) 3
5) 9

Book 2- Question of the Day

118

www.TestFunda.com

The LEADER in Online Test Prep

Question of the Day #230: (28-Nov-09)


The anthropologist James Frazer saw myths as a misinterpretation of magical rituals, which were
themselves based on a mistaken idea of natural law. According to Frazer, man begins with an
unfounded belief in impersonal magical laws. When he realizes that his applications of these laws
don't work, he gives up his belief in natural law, in favour of a belief in personal gods controlling
nature thus giving rise to religious myths. Meanwhile, man continues practising formerly magical
rituals through force of habit, reinterpreting them as reenactments of mythical events. Finally,
Frazer contends, man realizes that nature does follow natural laws, but now he discovers their true
nature through science.
Which of the following statements best sums up the the main argument presented in the passage?
OPTIONS
1) Man progresses "from magic through religion to science.
2) By pitting mythical thought against modern scientific thought, such theories implied that
modern man must abandon myth
3) 19th-century theories framed myth as a failed or obsolete mode of thought, often by
interpreting myth as the primitive counterpart of modern science
4) The primitive mentality is a condition of the human mind, and not a stage in its historical
development
5) Myths arose due to the lack of abstract nouns and neuter gender in ancient languages

Question of the Day #231: (29-Nov-09)


Let N = aabb be a 4 digit natural number, such that the two three-digit numbers aab and abb are
both prime. What is the sum of the digits of the smallest such N?
OPTIONS
1)
2)
3)
4)
5)

8
10
12
14
None of these

Book 2- Question of the Day

119

www.TestFunda.com

The LEADER in Online Test Prep

Question of the Day #232: (30-Nov-09)


The term Male Abortion was coined by Melanie McCulley, a South Carolina attorney, in her 1998
article, "The Male Abortion: The Putative Father's Right to Terminate His Interests in and Obligations
to the Unborn Child". The theory is that when an unwed female becomes pregnant she has the
option of abortion, adoption, or parenthood; and argues, in the context of legally recognized gender
equality, that in the earliest stages of pregnancy the father should have the right to relinquish all
future parental rights and financial responsibility leaving the informed mother with the same three
options. This would allow the woman time to make an informed decision and give men the same
reproductive rights as women.
Which of the following statements is the author of the passage most likely to agree with?
OPTIONS
1) The availability of abortion should not relieve men of equal financial responsibility to their
child.
2) An unwed female does not easily get support from the father of the child.
3) Fathers should be given equal power over a decision about their offspring
4) The law recognises abortion as the exclusive right of men
5) Men do not want to carry the responsibility of parenthood and "Male Abortion" recognises
it.

Question of the Day #233: (01-Dec-09)


For how many values of p do the equations x2 + 3px + 36 = 0 and x2 4x + p = 0 have real roots if p >
0?
OPTIONS
1)
2)
3)
4)
5)

3
1
4
Infinite
Cannot be determined

Book 2- Question of the Day

120

www.TestFunda.com

The LEADER in Online Test Prep

Question of the Day #234: (02-Dec-09)


Computer viruses are known to everyone and their ability to spread is also well known. But it is
equally helpful to portray the biosphere of real, living microbes as a world wide web of informational
exchange. Microbes exchange information with each other and their environment, with DNA serving
as the packets of data going every which way. Microbes differ from computer viruses because they
not only spread but evolve, and do so at a faster pace than their hosts. Microbes are in fact well
designed to exploit this difference to their advantage in the war that occasionally erupts between
them and other species - a war we see as disease and death.
The world wide web of microbes as presented in the argument rests most accurately on which of the
following assumptions?
OPTIONS
1) The capacity of computer viruses to transfer information to other computers.
2) Living viruses can integrate their own DNA into their hosts genetic material, and this can be
copied and passed on.
3) Many segments of human DNA originated from encounters with viruses which
downloaded their information into human cells.
4) The sheer number of microbes, their ability to exchange information, and the speed of
transmission are akin to the World Wide Web.
5) The microbes capacity to transfer information to other organisms.

Question of the Day #235: (03-Dec-09)


Mahesh, a mathematics teacher, invested a certain amount for 15 years in a term deposit scheme
with a bank. He sat down to find how much interest he would earn for his money each year. He
found out that he would get Rs. 2,000 as interest for the 5th year and Rs. 8,000 as interest for the
11th year. If the interest is compounded annually, the total amount that Rajesh, a friend of Mahesh,
would get if he invested Rs. 5,000 for 15 years in the same scheme is:
OPTIONS
1)
2)
3)
4)
5)

Rs. 32,000
Rs. 16,000
Rs. 1,60,000
Rs. 3,20,000
None of these

Book 2- Question of the Day

121

www.TestFunda.com

The LEADER in Online Test Prep

Question of the Day #236: (04-Dec-09)


European governments are adamantly opposed to capital punishment the European Union bans it
outright and some Europeans consider its use in America barbaric. Indeed, many European
intellectuals argue that not just capital punishment, but punishment in general, does not deter
criminals.
Which of the following, if true, most seriously challenges the European intellectuals argument?
OPTIONS
1) Highly unpleasant actions are unnecessary to prevent even worse behavior that takes the
lives of innocent victims.
2) Europeans have seen their crime rates increase during the past twenty years, whereas
American rates have fallen.
3) Europeans have had crime rates well below American rates for the past half-century.
4) The available data on the impact of capital punishment on crime rate are quite limited.
5) Capital punishment makes a murderer fight harder to avoid being captured.

Question of the Day #237: (05-Dec-09)

If A B = 0, then what is the sum of all the possible values of x?


OPTIONS
1)
2)
3) 0
4) 1
5) None of these

Book 2- Question of the Day

122

www.TestFunda.com

The LEADER in Online Test Prep

Question of the Day #238: (06-Dec-09)


The only way to reduce corruption permanently is to drastically cut back government's role in the
economy. High priority should go to eliminating the thousands of petty, nuisance regulations and
laws on the books in most countries which do more harm than good, and that also encourage
bribery
and
other
efforts
to
unfairly
influence
government
officials
Which of the following is the underlying assumption in the above argument?
OPTIONS
1)
2)
3)
4)
5)

Deregulation increases corruption.


Regulation harms the government officials.
Without regulation, corruption cannot be reduced.
Corrupt actions involve illicit payoffs to government officials.
Economy of a country depends on government officials.

Question of the Day #239: (07-Dec-09)


If 15x2 214x + 455 0 and x 2 > 0, then what is the minimum value of the following expression

OPTIONS
1) 3
2)
3)
4)
5)

Book 2- Question of the Day

123

www.TestFunda.com

The LEADER in Online Test Prep

Question of the Day #240: (08-Dec-09)


Many intellectuals in the United States and Eastern Europe believe that West European social
welfare policies should be a blueprint for action in their own countries. But those policies are
financed by high taxes and costly mandates on business that are mainly responsible for the
enormous increase in unemployment during the past decade and a half.
Which of the following can be validly concluded from the above?
OPTIONS
1) West European social welfare policies will not work in the United States and Eastern Europe.
2) West European social welfare policies are ineffective.
3) The degree of taxation and regulation prevailing in Eastern European nations is very high.
4) People in Western Europe are healthier than those in the United and Eastern Europe.
5) None of the above.

Question of the Day #241: (09-Dec-09)


The total cost of production of n tube lights per day by Glolight is given by the following function.
C(n) = n2 + 10n + 363
Selling price per unit of n units produced in a day is given by the following function.
S(n) = 2(80 n)
If the number of units of tube lights sold per day is equal to number of units of tube lights produced
per day then how many units should the factory produce per week to maximize its weekly profit?
Production is done on all seven days of the week.
OPTIONS
1) 70
2) 175
3) 77
4) None of these

Book 2- Question of the Day

124

www.TestFunda.com

The LEADER in Online Test Prep

Question of the Day #242: (10-Dec-09)


This question consists of two capitalized words that have a certain relationship to each other,
followed by pairs of words. Choose the pair that is RELATED to each other in the same way as the
capitalized pair.
Nun : Habit
OPTIONS
1)
2)
3)
4)
5)

Church : Religion
Monk : Mendacious
Soldier : Uniform
Pilot : Plane
Pope : Papal

Question of the Day #243: (11-Dec-09)

OPTIONS
1)
2)
3)
4)

0
0.5
1
1

Question of the Day #244: (12-Dec-09)


This question consists of five groups of jumbled phrases, of which only one is grammatically
INCORRECT. Identify the INCORRECT option.
OPTIONS
1)
2)
3)
4)
5)

as some are theorizing / presently / in quantum thermodynamics


or air in a steam engine / or it can be the body / of a tropical cyclone
surround a single atom / resonating energy or it can be / a body of steam
that / something can be / the volumetric region
or it could also be / just one nuclide / (i.e. a system of quarks)

Book 2- Question of the Day

125

www.TestFunda.com

The LEADER in Online Test Prep

Question of the Day #245: (13-Dec-09)


In a certain country, all citizens are either of type T (who always speak the truth) or of type F (who
always lie). You meet two citizens A and B. A says, B is of type F. B says, A and I are not the same
type of citizens. Which of the following is true?
OPTIONS
1)
2)
3)
4)

Both A and B are of type T


Both A and B are of type F
A is of type T and B is of type F
A is of type F and B is of type T

Question of the Day #246: (14-Dec-09)


Items 1 to 5 (First Set) are parts of the human anatomy. Items 6 to 10 (Second Set) are meanings of
idiomatic phrases. Match each item in the second set with an item in the first set in order to identify
the idiomatic phrases.
1)
2)
3)
4)
5)
6)

Wrist
Shoulder
Eye
Tongue
Leg
to be troublesome to
deal with
7) a superstitious way to
say "Good Luck"
8) to avoid talking
9) to be mildly punisheds
10) cherished above all others
OPTIONS
1)
2)
3)
4)
5)

1-8, 2-6, 3-9, 4-7, 5-10


1-9, 2-7, 3-6, 4-8, 5-10
1-7, 2-10, 3-6, 4-9, 5-8
1-9, 2-6, 3-10, 4-8, 5-7
1-10, 2-8, 3-9, 4-7, 5-6

Book 2- Question of the Day

126

www.TestFunda.com

The LEADER in Online Test Prep

Question of the Day #247: (15-Dec-09)


There are three friends Amar, Akbar and Anthony. Each of them plays one or more sports out of
cricket, football and hockey for their school. The following facts are known:
At least two of them play football. Each sport is played by at least one of them. Each sport is played
by a different number of people. Amar cannot play hockey due to a childhood injury. Akbar plays
only one sport. Not all play cricket.
Which of the following is not true?
OPTIONS
1)
2)
3)
4)

Amar plays Cricket


Anthony plays Hockey
Akbar plays Cricket
Anthony plays Cricket

Question of the Day #248: (16-Dec-09)


Which two sentences in the following convey the same idea? Choose from the combinations listed
below:
1. He was told that
mum's the word.
2. He answered saying,
Elvis has left the building.
3. He was asked to pipe down.
4. He replied that the employee
could not cut the mustard.
5. He told them to let
bygones be bygones.
OPTIONS
1)
2)
3)
4)
5)

1, 5
2, 4
3, 5
2, 1
1, 3

Book 2- Question of the Day

127

www.TestFunda.com

The LEADER in Online Test Prep

Question of the Day #249: (17-Dec-09)


Ram is sitting on the ground. Shyam is standing on a balcony with his hand exactly 10 metres
vertically above Ram. Each of them has a ball in his hand. Ram throws up his ball and Shyam lets go
of his at the same instant. If the balls collide midway between Ram and Shyam, with what speed did
Ram throw up his ball?
Acceleration due to gravity = 10 m/s2.
OPTIONS
1)
2)
3)
4)

2.5 m/s
5 m/s
10 m/s
Cannot be determined

Question of the Day #250: (18-Dec-09)


Items 1 to 5 (First Set) are several meanings of the word COME. Items 6 to 10 (Second Set) are their
appropriate usages. Match each item in the second set with an item in the first set in order to
identify their correct usages.
1)
2)
3)
4)
5)
6)

to arrive by movement
to occur at a certain point
to befall
to give an impression
to have priority
The police assured the tourists
that no harm would come to them.
7) She comes across as quite
an intelligent girl.
8) The students are coming
home by bus.
9) My family comes first.
10) May comes after April.
OPTIONS
1)
2)
3)
4)
5)

1-10, 2-8, 3-7, 4-9, 5-6


1-8, 2-10, 3-6, 4-7, 5-9
1-6, 2-9, 3-10, 4-8, 5-7
1-8, 2-6, 3-9, 4-7, 5-10
1-9, 2-7, 3-6, 4-10, 5-8

Book 2- Question of the Day

128

www.TestFunda.com

The LEADER in Online Test Prep

Question of the Day #251: (19-Dec-09)


The total internal surface area of a cuboid shaped box without a lid is 12 cm2.
What is the maximum possible volume of such a box?

OPTIONS
1)
2)
3)
4)
5)

4 cm3
3.7 cm3
8.2 cm3
10 cm3
12 cm3

Question of the Day #252: (20-Dec-09)


Which of the following sentences draws a metaphor?
OPTIONS
1) The speech was as good as a reflection of King's evolving political advocacy in his later years,
which paralleled the teachings of the progressive Highlander Research and Education
Center, with whom King was affiliated.
2) The trip to India affected King in a profound way, deepening his understanding of nonviolent resistance and his commitment to America's struggle for civil rights.
3) In 1964, King became the youngest person to receive the Nobel Peace Prize for his work to
end racial segregation and racial discrimination through civil disobedience and other nonviolent means.
4) In his speech, "I have a dream" the message put forth by Martin Luther King is that
opportunity is the currency of the American dream.
5) King believed that organized, nonviolent protest against the system of southern segregation
known as Jim Crow laws would lead to extensive media coverage of the struggle for black
equality and voting rights.

Question of the Day #253: (21-Dec-09)


If f(x) = |x 1| + |x + 1|, for real values of x then which of the following is true?
OPTIONS
1)
2)
3)
4)

The function is continuous at both x = 0 and x = 1


The function is continuous at x = 0 and discontinuous at x = 1
The function is continuous at x = 1 and discontinuous at x = 0
The function is discontinuous at both x = 0 and x = 1

Book 2- Question of the Day

129

www.TestFunda.com

The LEADER in Online Test Prep

Question of the Day #254: (22-Dec-09)


Which sentence includes an example of metonymy?
OPTIONS
1) A military chaplain can be an army-trained soldier with additional theological training or a
priest nominated to the army by religious authorities.
2) Division 5 Australian Navy chaplains are "Principal Chaplains," and these three chaplains,
representing the three major Christian denominations: Catholic, Anglican and Protestant,
are accorded the rank and status of Commodore.
3) In the United Kingdom, the Ministry of Defence employs chaplains but their authority comes
from their sending church.
4) Chaplains serving in the U.S. Armed Forces wear the uniform of their respective branch of
service, and only wear clerical garb during the performance of a religious service.
5) The chaplain is a man of cloth who ministers to soldiers, sailors, marines, airmen, and other
members of the military.

Question of the Day #255: (23-Dec-09)


Five standard samples of wires have length 0.9, 0.9, 1.2, 1.0 and 0.8 metres. According to the six
sigma methodology, all manufactured wires but the ones that are within 6 standard deviations from
the mean of the standard samples are to be rejected.
Two new wires A and B come in. Their lengths are 0.7 and 1.7 metres respectively.
Which of the following is true?
OPTIONS
1)
2)
3)
4)

Both A and B will be accepted.


A will be accepted and B will be rejected.
B will be accepted and A will be rejected.
Both A and B will be rejected.

Book 2- Question of the Day

130

www.TestFunda.com

The LEADER in Online Test Prep

Question of the Day #256: (24-Dec-09)


Which sentence suffers from a tautology?
OPTIONS
1) Bush subsequently returned to San Jacinto in November 1944 and participated in operations
in the Philippines until his squadron was replaced and sent home to the United States.
2) In a 1988 campaign speech in Ohio, George H. W. Bush said, "It's no exaggeration to say the
undecideds could go one way or another."
3) Senator Barry Goldwater of Arizona declined to be considered and endorsed Bush, who,
along with his supporters, reportedly mounted an internal campaign to get a nomination.
4) He became involved in politics soon after founding his own oil company, serving as a
member of the House of Representatives, among other positions.
5) Bush signed a number of major laws in his presidency, including the Americans with
Disabilities Act of 1990; this was one of the most pro-civil rights bills in decades

Question of the Day #257: (25-Dec-09)


It is given that A is a 3 3 symmetric matrix and that B is a 3 3 skew-symmetric matrix. Also, C = A +
B. Neither A nor B can have more than three elements which are zero. Let x be the number of
elements of C which are zero. What is the maximum possible value of x?
OPTIONS
1)
2)
3)
4)
5)

3
5
6
9
None of these

Book 2- Question of the Day

131

www.TestFunda.com

The LEADER in Online Test Prep

Question of the Day #258: (26-Dec-09)


Which sentence includes a transferred epithet?
OPTIONS
1) Although people have been imprisoned throughout history, they have also regularly been
able to perform prison escapes.
2) A medium custody level is one where design and construction as well as inmate classification
reflect the need to provide maximum external and internal control and supervision of
inmates.
3) Prisons are normally surrounded by fencing, walls, earthworks, geographical features, or
other barriers to prevent escape.
4) In prison or residences inmates spend a great deal of time with other people immersed in
criminal pursuits and beliefs, whereas in community-based programs there is more
opportunity to mix with people involved in constructive, law-abiding activities.
5) Inmates are often heard referring to the cruel bars of prison while discussing their stay at
penitentiaries where the goal of the facilities is penance by the prisoners, through a
regimen of strict disciplines, silent reflections, and perhaps forced and deliberately pointless
labor on tread-wheels and the like.

Question of the Day #259: (27-Dec-09)


There are two mutually exclusive events whose probabilities are
and
in terms of some variable P.
What is the difference between the maximum and minimum possible values of P?
OPTIONS
1)
2)
3)
4)

1
9/10
1/2
6/5

Book 2- Question of the Day

132

www.TestFunda.com

The LEADER in Online Test Prep

Question of the Day #260: (28-Dec-09)


Fill in the blanks with the most appropriate pair of words from among the given options.
His ___ attitude when ____ about the change in policy does not mean that he would remain ____
when it was his turn to be one who _____ the performance of others.

OPTIONS
1) disinterested , apprised, uninterested , appraised
2) disinterested , appraised, uninterested, apprised
3) uninterested , appraised, disinterested , apprised
4) uninterested, apprised, disinterested, appraised

Question of the Day #261: (29-Dec-09)


We have a radioactive substance which decays according to the rule that the decay rate at any
instant of time is proportional to the amount of substance left. If 1/4th of the substance disappears
after 10 days, then approximately how much time (in days) will it take for half the substance to
disappear?
Use: ln (3/4) = 0.29 and ln (1/2) = 0.693

OPTIONS
1) 20 days
2) 24 days
3) 18 days
4) 30 days

Book 2- Question of the Day

133

www.TestFunda.com

The LEADER in Online Test Prep

Question of the Day #262: (30-Dec-09)


Fill in the blank with the most appropriate word/set of words from among the given options.
He _____ prosecution as the _____ of witnesses dropped dramatically. It was _____ to by the
Defence that the _____ of files generated during his court case led to complete chaos.

OPTIONS
1)
2)
3)
4)

eluded, number, alluded, amount.


eluded, amount, alluded, number.
alluded, amount, eluded, number.
alluded, number, eluded, amount.

Question of the Day #263: (31-Dec-09)


In the crime-ridden district of Hells Kitchen, 100 people come to the police station to report various
crimes. It turns out that 93 report being robbed, 76 report being assaulted, 62 report being
threatened and 81 report being cheated. The police head needs to pay a 100 rupee compensation to
all those suffering all four crimes. What is the minimum money (in rupees) he needs to pay?
OPTIONS
1) 1200
2) 1600
3) 700
4) 2400

Question of the Day #264: (01-Jan-10)


Fill in the blanks with the most appropriate pair of words from among the given options.
The ________ end of the movie was _________ by one and all. The last scene _________ the social
message regarding the _________ changes to be faced in the future.

OPTIONS
1)
2)
3)
4)

climactic, climatic, complimented, complemented


complemented, climactic, complimented, climatic
climactic, complimented, complemented, climatic
climatic, complimented, climactic, complemented

Book 2- Question of the Day

134

www.TestFunda.com

The LEADER in Online Test Prep

Question of the Day #265: (02-Jan-10)


If angles A, B and C are in AP, and B = 0 then X = cos A cos B cos C =
OPTIONS
1) 1
2) sin A cos A
3) sin2 A
4) cos2 A

Question of the Day #266: (03-Jan-10)


Read each statement, and from the options select the idiom that best describes what is being said in
the statement.
The anonymous donor is definitely a very wealthy person as his donation amounts to millions of
rupees.
OPTIONS
1)
2)
3)
4)

A cash cow
Cat-and-dog life
A fat cat
For the birds

Question of the Day #267: (04-Jan-10)


Consider the two arithmetic progressions 2, 6, 10, ... and 5, 12, 19, ...
If S is a set containing the first 100 members of each progression then how many distinct elements
are there in S?
OPTIONS
1)
2)
3)
4)
5)

190
188
186
184
182

Book 2- Question of the Day

135

www.TestFunda.com

The LEADER in Online Test Prep

Question of the Day #268: (05-Jan-10)


"Always do right. This will gratify some people and astonish the rest.
Mark Twain (1835-1910)
In this quotation the author is referring to:
OPTIONS
1)
2)
3)
4)

a ground rule for tackling intractable people


a mantra to succeed in life
bringing attention to things that can go wrong
choices we make in life

Question of the Day #269: (06-Jan-10)


For a sequence of positive integers, a1, a2, a3, ... we have that, a1 = 1
and, an + 1 = 2a12 + a22 + a32 + ... + an2 , n 1.
How many perfect squares can be found in the sequence an?
OPTIONS
1) 0
2) 1
3) 2
4) 3
5) 4

Question of the Day #270: (07-Jan-10)


Which set of words does not contain only collective nouns:
OPTIONS
1)
2)
3)
4)

crew, troupe, battery


team, band, clique
coterie, skein, wad
bevy, gang, light

Book 2- Question of the Day

136

www.TestFunda.com

The LEADER in Online Test Prep

Question of the Day #271: (08-Jan-10)


Consider the inequality

What is the maximum value of P for which this inequality is always true for all real, positive x, y and
z?

OPTIONS
1)
2)
3)
4)
5)

0.25
0.5
0.75
1
1.5

Question of the Day #272: (09-Jan-10)


Animals have these advantages over man: they never hear the clock strike, they die without any
idea of death, they have no theologians to instruct them, their last moments are not disturbed by
unwelcome and unpleasant ceremonies, their funerals cost them nothing, and no one starts lawsuits
over their wills.
- Voltaire (1694 - 1778)
In this quotation the author is referring to:
OPTIONS
1)
2)
3)
4)

the fact that he would rather be born as an animal than a human


animals not having any attachments to the material world
a simple comparison between two species
people suffering because they know they will eventually die

Question of the Day #273: (10-Jan-10)


How many solution sets (n, k) exist for the equation 1! + 2! + 3! + ... + n! = k2, where both n and k are
natural numbers?
OPTIONS
1) 0
2) 1
3) 2
4) 3
5) Infinite

Book 2- Question of the Day

137

www.TestFunda.com

The LEADER in Online Test Prep

Question of the Day #274: (11-Jan-10)


The two words below have a certain relationship to each other, followed by a four pairs of words.
Choose the pair that is RELATED to each other in the same way as the capitalized pair.
Apartheid: Prejudice
OPTIONS
1)
2)
3)
4)

Antipathy: Calm
Exacerbate: Soothe
Hadephobia: Death
Guerilla: Warfare

Question of the Day #275: (12-Jan-10)


The sequence x, y, z, w is a strictly decreasing sequence of positive numbers. If x + w = y + z then
which of the following is necessarily true?
OPTIONS
1)
2)
3)
4)
5)

xw > yz
xw < yz
xy > wz
xy < wz
More than one of the above.

Question of the Day #276: (13-Jan-10)


This question consists of two capitalized words that have a certain relationship to each other,
followed by certain pairs of words. Choose the pair that is RELATED to each other in the same way as
the capitalized pair.
Umbrage: Bliss
OPTIONS
1)
2)
3)
4)

Chagrin: Peeve
Pique: Vexation
Perturb: Disquiet
Nettle: Conciliate

Question of the Day #277: (14-Jan-10)


The number of positive integers n such that (n6 + 206) is divisible by (n2 + 2) is:
OPTIONS
1)
2)
3)
4)
5)

14
6
8
4
None of these

Book 2- Question of the Day

138

www.TestFunda.com

The LEADER in Online Test Prep

Question of the Day #278: (15-Jan-10)


Fill in the blanks in the passage with the most appropriate set of words from the options for each
blank.
Astrologers believe that the movements and positions of celestial bodies either ____ influence life
on Earth or correspond to events experienced on a human scale. Modern astrologers define
astrology as a ___ language, an art form, or a form of ____. Despite differences in definitions, a
common ____ of astrologers is that celestial placements can aid in the _____ of past and present
events, and in the prediction of the future.
OPTIONS
1)
2)
3)
4)

indirectly, symbiotic, divinity, interpretation, assumption


directly, symbiotic, divinity, interpretation, assumption
directly, symbolic, divination, assumption, interpretation
indirectly, symbiolic, divination, assumption, interpretation

Question of the Day #279: (16-Jan-10)


The sum of digits of the smallest positive integer that has exactly 18 positive integer factors and
leaves a remainder of 9 when divided by 31 is:
OPTIONS
1) 18
2) 16
3) 9
4) 8
5) None

Question of the Day #280: (17-Jan-10)


This question consists of two capitalized words that have a certain relationship to each other,
followed by certain pairs of words. Choose the pair that is RELATED to each other in the same way as
the capitalized pair.
Hydrocephalus: Cholecystectomy
OPTIONS
1)
2)
3)
4)

Atherosclerosis : Angioplasty
Hepatic neoplasms: Hepatectomy
Rheumatoid arthritis: Arthroplasty
Hernia: Tracheotomy

Book 2- Question of the Day

139

www.TestFunda.com

The LEADER in Online Test Prep

Question of the Day #281: (18-Jan-10)


There are 7 coins on a table, all tails up. If you can flip over exactly two at a time, the number of flips
to change them all to heads is

OPTIONS
1) 8
2) 3
3) 7
4) 5
5) None of these

Question of the Day #282: (19-Jan-10)


A sentence is given below with a part of it printed in bold type. That part may contain a grammatical
error. Each sentence is followed by phrases (1), (2), (3) and (4). Find out which phrase should replace
the phrase given in bold to correct the error, if there is any, and to make the sentence grammatically
meaningful and correct. If the sentence is correct as it is and no correction is required, mark (5) as
the answer.
The primary feature of Mental Ray is the achievement of high performance yet parallelism on both
multiprocessor machines and across rendering farms.

OPTIONS
1)
2)
3)
4)
5)

high performance though parallelism


high performance within parallelism
high performance as parallelism
high performance through parallelism
No correction required

Book 2- Question of the Day

140

www.TestFunda.com

The LEADER in Online Test Prep

Question of the Day #283: (20-Jan-10)

OPTIONS
1)
2)
3)
4)
5)

1
11
22
33
44

Question of the Day #284: (21-Jan-10)


Fill in the blanks in the passage with the most appropriate set of words from the options for each
blank.
In an enterprise whose __________ technology concerns are addressable by ready-made
technologies, a CIO might be the primary _________ of technology issues at the executive level. In
an enterprise whose primary technology concerns are addressed by developing new technologies, or
the general __________ exploitation of intellectual ___________ held by the company, a CTO might
be the primary representative of these concerns at the ___________ level.
OPTIONS
1)
2)
3)
4)

primary, representative, strategic, property, executive


secondary, representative, strategic, property, executive
secondary, representative, extempore, property, tertiary
primary, indicative, extempore, goods, executive

Book 2- Question of the Day

141

www.TestFunda.com

The LEADER in Online Test Prep

Question of the Day #285: (22-Jan-10)


There are four rooms in a small office building. A tea seller sells tea to the people in these rooms at
the rate of Rs. 5 per glass. Assume that he makes only 3 glasses of tea on any given day for this
building, and that any given room orders at most one glass of tea. If the probability of any of the four
rooms ordering a glass of tea is 3/4, what is the probability of the tea seller making Rs.15 a day from
this building?
OPTIONS
1)
2)
3)
4)
5) None of these

Question of the Day #286: (23-Jan-10)


Fill in the blanks in the passage with the most appropriate set of words from the options for each
blank.
In cold deserts, hypothermia and frostbite are the chief hazards, as well as dehydration in the
___________ of a source of heat to melt ice for drinking. Falling through pack-ice or surface ice
layers into freezing water is a particular danger requiring emergency action to prevent
_____________ hypothermia. Starvation is also a ___________; in low temperatures the body
requires much more food energy to __________ body heat and to move. As with hot deserts, some
people such as the Inuit have adapted to the ____________ conditions of cold deserts.
OPTIONS
1)
2)
3)
4)

presence, immediate, danger, maintain, hard


absence, rapid, hazard, maintain, harsh
absence, quick, problem, induce, ongoing
presence, rapid, danger, control, unforgiiving

Question of the Day #287: (24-Jan-10)


If the function f(x) is defined for real x as f(x) = x2 (for x2 1) and f(x) = x (for x2 < 1), at how many
points is f(x) discontinuous?
OPTIONS
1) 0
2) 1
3) 2
4) 3
5) More than 3
Book 2- Question of the Day

142

www.TestFunda.com

The LEADER in Online Test Prep

Question of the Day #288: (25-Jan-10)


A sentence is given below with a part of it printed in bold type. That part may contain a grammatical
error. Each sentence is followed by phrases (1), (2), (3) and (4). Find out which phrase should replace
the phrase given in bold to correct the error, if there is any, and to make the sentence grammatically
meaningful and correct. If the sentence is correct as it is and no correction is required, mark (5) as
the answer.
Homer Simpson is a fictional main characteristic in the animated television series "The Simpsons".
OPTIONS
1)
2)
3)
4)
5)

fiction main character


fictional main character
fictional mainly character
fictional main characters
No correction required

Question of the Day #289: (26-Jan-10)


An ant is sitting at the centre of one of the faces of a solid cube (kept in the space) of side 4
m. Poisonous material is kept at all the vertices of the cube and the effect of poison spreads to 2 m
in all the directions. If the ant wants to visit all the faces of the cube, then what is the minimum
distance (in m) that she must travel to get back to the original position?

OPTIONS
1)
2)
3)
4)
5)

41
42
40
37
33

Book 2- Question of the Day

143

www.TestFunda.com

The LEADER in Online Test Prep

Question of the Day #290: (27-Jan-10)


A sentence is given below with a part of it printed in bold type. That part may contain a grammatical
error. Each sentence is followed by phrases (1), (2), (3) and (4). Find out which phrase should replace
the phrase given in bold to correct the error, if there is any, and to make the sentence grammatically
meaningful and correct. If the sentence is correct as it is and no correction is required, mark (5) as
the answer.
Physical exercise helps people improve their health both physically and mentally.
OPTIONS
1)
2)
3)
4)
5)

both physically and mental


physically and mentally both
both physical and mentally
both physique and mentally
No correction required

Question of the Day #291: (28-Jan-10)

OPTIONS
1) -1
2) 0
3)
4) None of the above
5) This equation has no solution

Question of the Day #292: (29-Jan-10)


This question consists of two capitalized words that have a certain relationship to each other,
followed by a single word. Choose the option that creates a pair with the single word such that the
pair RELATED to each other in the same way as the capitalized pair.
Pride: Lions :: Unkindness: ?
OPTIONS
1) Pigs
2) Jackals
3) Ravens
4) Weasels

Book 2- Question of the Day

144

www.TestFunda.com

The LEADER in Online Test Prep

Question of the Day #293: (30-Jan-10)


If the two real functions f(x) and g(x) satisfy the functional equation f(x + 1) + g(x 1) = x2, what is
the value of

OPTIONS
1)
2)
3)
4)
5)

-2
-1
0
1
2

Question of the Day #294: (31-Jan-10)


From the following pair of words, identify the pair that shares the same relationship as the given
pair.
Abscond: Absquatulate
OPTIONS
1)
2)
3)
4)

Contrive: Stop.
Infernal: Blessed
Pilfer: Purloin
Condemn: Exonerate

Book 2- Question of the Day

145

www.TestFunda.com

The LEADER in Online Test Prep

Question of the Day #295: (01-Feb-10)


Find the sum of all real values of x such that

OPTIONS
1) 0
2) 7
3) -7
4)
5)

Question of the Day #296: (02-Feb-10)


Fill in the blanks in the passage with the most appropriate set of words from the options for each
blank.
The ____1_____ value of a book depends not so much upon the information _____2_____ as upon
the permanent interest stimulated and the initiative aroused. The youthful mind, and ____3____ the
average adult mind as well, is singularly non-logical and incapable of _____4______ concentration,
and loses interest under too consecutive thought and sustained ____5_____.

OPTIONS
1)
2)
3)
4)

real, sustained, indeed, non-descript, style


nominal, given, indeed, continued, push
real, given, indeed, continued, style
nominal, passing, with that, non-descript, pull

Question of the Day #297: (03-Feb-10)


In a trapezium ABCD, AB||CD, AB = 2010 cm, CD = 1000 cm, DAB = 40 and ABC = 50.
What is the length of the line joining the midpoints of AB and CD?

OPTIONS
1)
2)
3)
4)
5)

1000 cm
1010 cm
504 cm
505 cm
None of these

Book 2- Question of the Day

146

www.TestFunda.com

The LEADER in Online Test Prep

Question of the Day #298: (04-Feb-10)


From the following pair of words, identify the pair that shares the same relationship as the given
pair.
Opera : Interlude

OPTIONS
1) Farce : Burlesque
2) Ditty: Cadence
3) Vespers: Canticle
4) Vaudeville: Revue

Question of the Day #299: (05-Feb-10)


2010 coins, each of value 1, 2 or 3 are arranged in a row such that between any two coins of value 1
there is at least one coin, between any two of value 2 there are at least two coins, and between any
two of value 3 there are at least three coins. The maximum number of coins of value 3 that can be in
the row?

OPTIONS
1)
2)
3)
4)
5)

502
670
503
669
501

Question of the Day #300: (06-Feb-10)


From the following pair of words, identify the pair that shares the same relationship as the given
pair.
Urbane: Gaucherie

OPTIONS
1)
2)
3)
4)

Confident: Clique
Astucious: Shenanigan
Narcissistic: Altruism
Aghast: Annihilation

Book 2- Question of the Day

147

www.TestFunda.com

The LEADER in Online Test Prep

SOLUTIONS

Book 2- Question of the Day

148

www.TestFunda.com

The LEADER in Online Test Prep

Solution #01: (13-Apr-09)

Refer the diagram, here AB is the bridge, point C denotes the cat and point T denotes the train.
Let the length of the bridge be L m and the speed of the cat be S m/s.
The speed of the train = 72 km/hr = 20 m/s

If the cat runs towards the train with constant speed, then it will reach the end of the bridge when
the train is still 50 m away.
The cat has reached the end of the bridge in the same time in which the train has

(10L 400) = S (2L 50)

...(i)

If the cat runs away from the train with constant speed, the train will hit it while it is 12.5 m from B.
The distance travelled by the cat in this time

And, the distance travelled by the train in the same time is (2L + L 12.5) = (3L 12.5) m

(10L + 150) = S (3L 12.5)

...(ii)

Solving equations (i) and (ii) simultaneously, we get,


Book 2- Question of the Day

149

www.TestFunda.com

The LEADER in Online Test Prep

L = 100 m and S = 4 m/s


Now, we need to find the range of values of x, the initial position of the man for which he will be
safe if he runs towards the train. The lower limit of x is obviously 0 and for the upper limit, we note
that it will occur only when the man and the train reach the point A at the same time.

Range of the values of x is from 0 to 40 m.


Hence, option 3.
Discuss the solution with Testfunda users.

Solution #02: (14-Apr-09)


The essence of the paragraph is:
In The Birth of Tragedy Nietzsche presented his theory of dualism in art. Any work of art contained
identity and the dissolution of identity (art of sculpture and art of music). This dualism can be
compared to the two Greek Gods Apollo and Dionysius.
Option 1 does not make this dualism clear.
Options 3 and 5 do not mention the Apollonian and the Dionysian experiences.
Option 4 omits the reference to art.
Option 2 gives the essence of the paragraph in a clear and succinct manner.
Hence, the correct answer is option 2.
Discuss the solution with Testfunda users.

Book 2- Question of the Day

150

www.TestFunda.com

The LEADER in Online Test Prep

Solution #03: (15-Apr-09)


Consider,

Now, consider

Substituting x from equation (i) we get,

log10 y 3 0.7 = 2.1


Hence, option 2.
Discuss the solution with Testfunda users.

Book 2- Question of the Day

151

www.TestFunda.com

The LEADER in Online Test Prep

Solution #04: (16-Apr-09)


Statement A is implied in the passage because competition threatens Alitalias core part of business,
which is its services between Rome and Milan.
Statement B is virtually contrary to the passage. First, because fuel prices have fallen. Second,
because it is dwindling demand that is making the outlook bleak.
Statement C is not conclusive - it may be a possibility. Though the businessmen and bankers are at
ease with risk, they hadn't anticipated the weakening demand- is not conclusive and cannot be
implied with any degree of certainty.
Statement D is supported by investing in the phoenix-like rebirth- the extract in quotes suggests
that without them the airline would have been fully out of business.
Therefore statements A and D are implied.
Hence, the correct answer is option 2.
Discuss the solution with Testfunda users.

Solution #05: (17-Apr-09)


If p is a 3 digit prime then p2 is always greater than 2000.

Where [x] represents greatest integer less than or equal to x.

Highest power of p dividing 2000C1000


= highest power of p dividing 2000! 2 (highest power of p dividing 100!)

If p > 666 then,

i.e. p does not divide 2000C1000.


Thus the required largest prime is such that it is less than 666.
The largest 3 digit prime less than 666 is 661.
If p = 661 then,

661 is the largest 3 digit prime dividing 2000C1000.


Hence, option 1.
Discuss the solution with Testfunda users.
Book 2- Question of the Day

152

www.TestFunda.com

The LEADER in Online Test Prep

Solution #06: (18-Apr-09)


Statement A is implied in the first statement of the passage, in which the two groups of
campaigners- namely the lobby to save animals or trees and the lobby who try to preserve
languages- are compared. The comparison establishes similarity.
B is implied in the first statement, as it states these campaigners are a rare breed.
C is not implied. If at all we can conclude more African languages are endangered and not African
languages are more endangered.
D is implied as the passage states that 50 to 90 percent of languages are going to die by the end of
the century.
Therefore, statements A, B and D can be implied.
Hence, the correct answer is option 3.
Discuss the solution with Testfunda users.

Solution #07: (19-Apr-09)


N = 101 10001 100000001 (10000 0001)

Multiply and divide N by (102 1), we get,

Continuing in this way, we get,

Using the formula for sum of n terms of a G.P., we get,

N = 10101010 01
This number N is the addition of 128 terms and each term corresponds to one '1' digit.
The final product has 128 one's.
Hence, option 3.
Discuss the solution with Testfunda users.
Book 2- Question of the Day

153

www.TestFunda.com

The LEADER in Online Test Prep

Solution #08: (20-Apr-09)


Castor is a type of oil. Caster, comes from cast and means one who casts. Since, the paragraph
refers to a person, the word is caster.
One munches on food and mincing is a process of cutting or chopping something. Hence, the
correct word is 'munching'.
Baneful means causing harm or destruction. Since the casters wife only wants to only threaten
and not cause any harm, the third word is baleful, which means a menacing influence.
Awhile, an adverb is never preceded by a preposition like for. Therefore, a while is suitable.
Cashew is an edible nut whereas, cachou, our last word refers to a lozenge used for
sweetening breath. The second word is more suitable in this context.
Hence, the correct answer is option 3.
Discuss the solution with Testfunda users.

Solution #09: (21-Apr-09)

Refer to the diagram for all distances travelled.


Let Vi=speed of Intezam in ft/s and Vs=speed of Sachit in ft/s.

Book 2- Question of the Day

154

www.TestFunda.com

The LEADER in Online Test Prep

Also, D be the distance between the stumps (in ft).

The time taken by Intezam and Sachit to reach their first crossing, beginning from the start of the
first run, will be equal.

Hence, we have,

The time taken by Intezam and Sachit to reach their second crossing, beginning from the start of the
first run, will be equal.
Hence, we have,

To find D, we equate the two values of Vs/Vi obtained in the two steps above.

D=24 ft

Now, we will use the values we have determined so far to find out where the third meeting will
occur.

Book 2- Question of the Day

155

www.TestFunda.com

The LEADER in Online Test Prep

When Intezam and Sachit meet for the second time, they are both running their second run. After
the second crossing, Intezam is running towards the strikers end and Sachit is running towards the
non-strikers end.
Since the ratio of speeds of Sachit and Intezam is 5:3, in the above diagram when Intezan moves 3
units, Sachit will move 5 units. Using this logic, we can determine where Intezam and Sachit will
meet for the third time:

They meet for the third time before Intezam has completed his second run and when Sachit is
exactly in the middle of his third run.
The total distance travelled by Sachit up to this point is (2D + 12) = (48 + 12) = 60 ft
Hence, option 3.
Discuss the solution with Testfunda users.

Solution #10: (22-Apr-09)


Assent is saying yes, or agreeing to some point to view, whereas, ascent is climbing up or
climb. Our first word is ascent, as the author speaks about the upward movement or rise of the
country.
Autarchy is similar to autocracy or despotism. When we speak about good governance and more
importantly economic self-sustainability the word is autarky.
Disassembled- taken apart is the opposite of assembled. The country leaders want the taken
apart pieces of the country to be put together. Dissemble is to disguise or conceal under a false
appearance.
Depositary is our next word which means a trustee. Although, depository is sometimes used for
this, a trustee is specifically a depositary while depository also means a place for safe keeping.
Detrain is to leave or cause to leave a railroad train, whereas our word is detract- divert ones
attention or focus from.
Hence, the correct answer is option 1.
Discuss the solution with Testfunda users.
Book 2- Question of the Day

156

www.TestFunda.com

The LEADER in Online Test Prep

Solution #11: (23-Apr-09)


The roots of the equation x2 32x + 240 = 0 are 12 and 20.
12 is not a possible value for the semiperimeter as then the sides of the triangle will be 6, 6, 12
which is not possible.
s = 20 and the length of equal sides is 14.

Hence, option 4.
Discuss the solution with Testfunda users.

Solution #12: (24-Apr-09)


Option 1 is not conclusive because it has also infected civilian networks, as mentioned in the
passage.
Option 2 is also inconclusive- the passage does not imply that it infects only portable devices, but it
may spread through infected portable devices. If it infects only portable devices there is no danger
to vaster networks and therefore does not pose much danger which is contrary to the tone of the
passage.
Option 3 is supported by the passage. Every bit of portable memory in the sprawling American
military establishment now needs to be scrubbed clean before it can be used again.
Options 4 and 5 are nonsensical options as glue cannot do anything to prevent malicious software
programs from spreading. The bank merely used glue to block access to USB ports.
Hence, the correct answer is option 3.
Discuss the solution with Testfunda users.

Solution #13: (25-Apr-09)


Lagranges four square theorem states that, Every positive integer can be expressed as the sum of
four squares of integers.
Consider the number 310, since we have to express it as the sum of squares of four integers we will
first subtract the square of the number closest to it i.e. 172.
310 289 = 21
Now we will subtract the square of the number closest to 21 i.e. 42
21 16 = 5
Again we now subtract 22 and then 12 and we get a zero.
Hence, 310 = 172 + 42 + 22 + 12
Similarly,
123 = 112 + 12 + 12 + 02
59 = 72 + 32 + 12 + 02
67 = 82 + 12 + 12 + 12
Hence, option 5.
Discuss the solution with Testfunda users.
Book 2- Question of the Day

157

www.TestFunda.com

The LEADER in Online Test Prep

Solution #14: (26-Apr-09)


Option 1 is contrary to the data. The writer shows through the example of areca plantations that
farming and wildlife can be easy bedfellows contrary to the general belief that they are not.
Option 2 cannot be inferred because the passage does not support the inference about
widespread.
Option 3 can be considered but this inference is partial- only about birds- whereas the passage
mentions that the Western Ghats still remains a hotspot for biodiversity.
Option 4 which talks about areca plantations being biodiversity friendly is more inclusive as an
inference than option 3.
Option 5 cannot be inferred because of the generalisation about the entire Western Ghats. The
passage mentions that areca palms are hospitable to birds, not necessarily the entire Western ghats.
Hence, the correct answer is option 4.
Discuss the solution with Testfunda users.

Solution #15: (27-Apr-09)


As AC is the middle term, AC = 4.5
Because the length of sides is in arithmetic progression, 2AC = AB + BC
From BC = 2AB and AC = 4.5, we get AB = 3 and BC = 6
BD = 3, as AD bisects side BC.
Using Apollonius theorem, AB2 + AC2 = 2(AD2 + BD2)
32 + 4.52 = 2(AD2 + 32)
AD 2.35
Hence, option 2.
Discuss the solution with Testfunda users.

Solution #16: (28-Apr-09)


Option 5 appears close but it speaks of a certain disposition of action- not an idea presented in the
paragraph.
Option 3 has an issue with the tone which suddenly becomes very harsh. Beliefs cannot be
hammered into a person nor can attitudes be plastered.
Option 2 limits itself to physical aspects and is contradictory to the paragraph which cites education
as the most important factor for inculcating beliefs and aspirations.
Option 1 is close but it does not link well with the penultimate statement in the paragraph.
Option 4 links well with the penultimate statement of the paragraph which mentions the shaping
and molding of young people into a standard form of social activity. The paragraph begins with the
definition and process of self-renewal of a community or social group which is also addressed by
option 4, standard form of social activity.
Hence, the correct answer is option 4.
Discuss the solution with Testfunda users.
Book 2- Question of the Day

158

www.TestFunda.com

The LEADER in Online Test Prep

Solution #17: (29-Apr-09)


(a b)2 0
a2 + b2 2ab 0
a2 + b2 2ab
Similarly, b2 + c2 2bc and c2 + a2 2ac
Now, (a + b)(b + c)(c + a) = 2abc + c(a2 + b2) + b(a2 + c2) + a(b2 + c2)
(a + b)(b + c)(c + a) 2abc + c(2ab) + b(2ac) + a(2bc)
(a + b)(b + c)(c + a) 8abc

Let a = sinx, b = siny and c = sinz


a, b and c are positive real numbers (as x, y, z lie in the first quadrant),

Hence, the value of the expression has to be greater than 8.


Hence, option 3.
Discuss the solution with Testfunda users.

Solution #18: (30-Apr-09)


Option 4 can be eliminated as the paragraph does not mention anything on victory or defeat.
Option 3 adds data - but does not complete the passage. It also brings in the new elements of love
and co-habitation.
Option 5 speaks of destitute. The paragraph is silent on the economic condition of criminals and
malefactors- central characters of the paragraph.
Option 1 is a possible answer but it brings in a new dimension which is not indicated in the
paragraph.
Option 2 continues the idea of the penultimate statement beautifully and connects clearly with the
central theme of the passage - rehabilitation or reformation of criminals and malefactors.
Hence, the correct answer is option 2.
Discuss the solution with Testfunda users.

Book 2- Question of the Day

159

www.TestFunda.com

The LEADER in Online Test Prep

Solution #19: (01-May-09)


Consider the diagram,

Apollonius Theorem states that given a ABC, if D is any point on BC such that it divides BC in the
ratio n : m then
mAB2 + nAC2 = mBD2 + nDC2 + (m + n)AD2 (i)
Here, n = 2 and m = 3

Substituting the values in (i), we get,


3 (64) + 2 (36) = 3 (16) + 2 (36) + (2 + 3) AD2
192 + 72 = 48 + 72 + 5 AD2
144 = 5 AD2
AD 5.4 cm
Hence, option 1.
Discuss the solution with Testfunda users.

Book 2- Question of the Day

160

www.TestFunda.com

The LEADER in Online Test Prep

Solution #20: (02-May-09)


Regretting is a better word for the first blank in place of implying. One regrets ones weakness
especially if one knows about them. Hence, we can eliminate options 2 and 4.
Fritter and squander can both fit in the second blank. Although speculations is a better word
than trifles for the third blank, we can still check the word for the fourth blank. Someone can be
pushed or compelled to sink ones fortunes, not emulated- copied. Copied from whom?
Hence, the correct answer is option 1.
Discuss the solution with Testfunda users.

Solution #21: (03-May-09)


From the question itself, we know that (9) = 6
We calculate the totient of an integer less than 9 and co prime to 9, say 4.
(4) = 2 (1 and 3 are the only positive integers less than 4 that are co prime to it)
Now,
(4 + 9) = (13) = 12 (13 is a prime number and 1, 2, 3, ..., 12 are co prime to it)
And, (4) + (9) = 6 + 2 = 8
(4) + (9) < (4 + 9)
We can eliminate options (1) and (4).
(9 4) = (5) = 4 (1, 2, 3, 4 are positive integers less than and co prime to 5)
(9) (4) = 6 2 = 4
Option (2) may hold true.
But,
(9 4) < 2(9) (4) = 36 4 = 32
We can eliminate option (5) too.
(36) = 12 (1, 5, 7, 11, 13, 17, 19, 23, 25, 29, 31, and 35 are less than and co prime to 36)
And, (4) (9) = 12
(4) (9) = (36)
Hence, option (3) may also be true.
We now check option (2) with a different set of values of m and n.
We know that (36) = 12 and (23) = 22. And 36 and 23 are co prime to each other.
(36 23) = (13) = 12
And, (36) (23) = 12 22 = 10
(36 23) > (36) (23)
So we can eliminate option (2).
Hence, option 3.
Observation: If p is any prime number then, (p) = p 1 i.e. there are p 1 positive integers less
than p that are co prime to it.
Discuss the solution with Testfunda users.

Book 2- Question of the Day

161

www.TestFunda.com

The LEADER in Online Test Prep

Solution #22: (04-May-09)


The paragraph emphasizes the importance of transmission of ideas and practices for the constant
renewing of the social fabric. It further states that it is not automatic. To continue the line of
thought option 2 expresses it best by stating that the renewal is not automatic but that pains
need to be taken.
Options 3 and 4 may also continue the idea but they give detailed treatment to the concept on
rearing the young, a topic not mentioned in the paragraph.
Options 1 and 5 can be easily eliminated since they do not connect well with the penultimate
statement in the paragraph.
Hence, the correct answer is option 2.
Discuss the solution with Testfunda users.

Solution #23: (05-May-09)


Mr. Sharma goes to the market with Rs. 252 but we dont know if he spends the entire amount in
buying watermelons and mangoes.
If he bought x mangoes and y watermelons.
Then, 4x + 3y 252
We cannot determine the amount he spent in buying mangoes.
Hence, option 5.
Extra Learning:
If the question would have given that, Mr. Sharma spent the entire amount in buying watermelons
and mangoes then the question would have the following solution.
Let the number of mangoes he buys be x and the number of water melons he buys be y.
Then, 4x + 3y = 252
x = (252 3y)/4
The possible values of y such that x is a natural number (he has to buy at least one mango and one
watermelon) are given in the following sets:
(x, y) = (60, 4), (57, 8), (54, 12), (51, 16), (48, 20), (45, 24), (42, 28), (39, 32), (36, 36), (33, 40), (30,
44), (27, 48), (24, 52), (21, 56), (18, 60), (15, 64), (12, 68), (9, 72), (6, 76), (3, 80)
We can see that the values of y are multiples of 4 in ascending order and values of x are multiples of
3 in descending order.
But, the number of watermelons and mangoes are not co prime to each other, number of mangoes
is multiples of 4 and the difference between the number of mangoes and watermelons should be
minimum. We get two possible values of x and y, respectively 48 and 20 and 24 and 52.
The amount he spends on mangoes, in this case too, cannot be determined.
Discuss the solution with Testfunda users.
Book 2- Question of the Day

162

www.TestFunda.com

The LEADER in Online Test Prep

Solution #24: (06-May-09)


The second blank should be consistency and not inconsistency because the other quality
mentioned i.e. clearness is a positive quality whereas inconsistency is essentially negative.
Therefore, options 1 and 5 are eliminated.
Similarly, the third blank should be experience and not inexperience because, the statement
mentions qualification and inexperience is something that one would not quote as a qualification.
Therefore, option 4 is eliminated.
Option 3 cannot be the correct answer option because there is no such word as thoroghly.
Hence, the correct answer is option 2.
Discuss the solution with Testfunda users.

Solution #25: (07-May-09)


f(x2 + 1) = x4 + 7x2 + 9
Lets write x4 + 7x2 + 9 in terms of x2 + 1.
x4 + 7x2 + 9 = (x2 + 1)2 + 5(x2 + 1) + 3
Put x2 + 1 = t in function f,
f(t) = (t)2 + 5(t) + 3
Also, f(x2 + 1) = g(x2 1) = g[(x2 + 1) 2+
f(t) = g(t 2)
Now, we have, f(g(p)) = g(f(p))
f(g(p)) = f[f(p) + 2]

*Since g(t) = f(t + 2)]

g(p) = f(p) + 2
f(p + 2) = f(p) + 2

*Since g(t) = f(t + 2)]

(p + 2)2 + 5(p + 2) + 3 = p2 + 5p + 3 + 2
p2 + 4p + 4 + 5p + 10 + 3 = p2 + 5p + 5
4p + 12 = 0
p = 3
Hence, option 5.
Discuss the solution with Testfunda users.

Book 2- Question of the Day

163

www.TestFunda.com

The LEADER in Online Test Prep

Solution #26: (08-May-09)


Statement 1 mentions the need for lubrication. So, the next logical statement would be the effect of
lack of lubrication. Thus, 1-B link is identified. This eliminates options 3, 4, 5 based on this
connection.
The different types of lubricants used in different engines, from simple to complex, are described
serially in the D-C-A link.
Statement A links well with statement 6 as statement 6 describes the process of pressure
lubrication introduced in statement A.
Hence, the correct answer is option 1.
Discuss the solution with Testfunda users.

Solution #27: (09-May-09)


Consider option 1:

g(2, 5, 1) = min(4, 5, 3) = 3
h(2, 3, 1) = max{min(2, 3), min(3, 1), min(1, 2)} = max(2, 1, 1) = 2
k{f(2, 3, 6), g(2, 5, 1), h(2, 3, 1)} = k(3, 3, 2)

So option 1 is not true.

Consider, option 2:
g(4, 8, 2) = min(8, 8, 6) = 6
f(6, 5, 12) = max(3, 5, 4) = 5
g(4, 8, 2) > f(6, 5, 12)
So option 2 is true.

Consider, option 3:

So option 3 is also true.


Only option 2 and 3 are true.
Hence, option 4.
Discuss the solution with Testfunda users.

Book 2- Question of the Day

164

www.TestFunda.com

The LEADER in Online Test Prep

Solution #28: (10-May-09)


The sequence of DCB- unbreakable- can be very easily deciphered.
Statement D introduces the topic of the paragraph.
Statement C expands upon the more than it can consume locally mentioned in statement D and B
mentions the fact that these expatriates rarely forget their country of origin.
This eliminates options 1, 3 and 5.
The only difficulty- if at all- is in placing the statement A, before DCB (option 4). It is better placed at
the end so that undue importance is not attributed to it in the paragraph by placing it in the
beginning. It is better treated as an example and placed at the end rather than making it trigger the
other statements.
Hence, the correct answer option is 2.
Discuss the solution with Testfunda users.

Solution #29: (11-May-09)


If we carefully observe the algorithm, we get the values of x, y and p as follows,
Initial Values:
x = 1,
y = 1,
p=0
After loop 1:
x = 1 + 1 = 2 (Prime),
y = 1 + 2 = 3,
p=0+2+3=5
After loop 2:
x = 2 + 3 = 5 (Prime),
y = 3 + 5 = 8,
p = 5 + 5 + 8 = 18
After loop 3:
x = 5 + 8 = 13 (Prime),
y = 8 + 13 = 21,
p = 18 + 13 + 21 = 52

Book 2- Question of the Day

165

www.TestFunda.com

The LEADER in Online Test Prep

After loop 4:
x = 13 + 21 = 34 (Composite),
y = 21 + 34 = 55,
p = 52
After loop 5:
x = 34 + 55 = 89 (Prime),
y = 55 + 89 = 144 (Perfect square),
p = 52 + 89 + 144 = 285
After this, algorithm will jump to step 7,
p = 285 + 89 + 144 = 518
And then it stops.
Final value of p = 518
Hence, option 1.
Discuss the solution with Testfunda users.

Solution #30: (12-May-09)


Statement D is the only statement that can start the paragraph as it introduces the topic - economic
growth rate of China and India using new data.This eliminates option 2 and 5.
Statement B mentions the inelasticity in increase in regional growth and says that it is not so at the
national level.
Statement E then adds to this by saying that the national growth rate can, too, be considered to be
inelastic and supports this with an illustration. This establishes a BE link. This eliminates option 1.
Thus, the answer has to be between options 3 and 4.
Statement A will link well with statement D because while D introduces the topic by mentioning an
investigation carried out by Leong, statement A describes how this investigation was carried out,
i.e. explicitly identified using instrumental variables. Thus, the DA link at the beginning of the
sequence eliminates option 4.
Hence, the correct answer is option 3.
Discuss the solution with Testfunda users.

Book 2- Question of the Day

166

www.TestFunda.com

The LEADER in Online Test Prep

Solution #31: (13-May-09)


Let m be the marked price of the item and c be the cost price of the item.
As marked price of the item is 25% above cost price, so m = 1.25c
Maximum discount that the shopkeeper can give is 20% so that he does not bear any loss.
Let a, b, c (natural numbers) be the discount percentages offered by shopkeeper to 3 customers.

These prices are in A.P.

(100 a) + (100 c) = 2(100 b)


a + c = 2b
Here, a, b, c cannot exceed 20.
Number of Possible sets of (a, b, c) are as follows.
When a = 1;
(1, 2, 3), (1, 3, 5), (1, 4, 7), (1, 5, 9), (1, 6, 11), (1, 7, 13), (1, 8, 15), (1, 9, 17), (1, 10, 19)
Number of sets = 9
Similarly, When a = 2; Number of sets = 9
Similarly, When a = 3; Number of sets = 8
Similarly, When a = 4; Number of sets = 8
Similarly, When a = 5; Number of sets = 7
Similarly, When a = 6; Number of sets = 7
Similarly, When a = 7; Number of sets = 6
Similarly, When a = 8; Number of sets = 6
Similarly, When a = 9; Number of sets = 5
Similarly, When a = 10; Number of sets = 5
Similarly, When a = 11; Number of sets = 4
Book 2- Question of the Day

167

www.TestFunda.com

The LEADER in Online Test Prep

Similarly, When a = 12; Number of sets = 4


Similarly, When a = 13; Number of sets = 3
Similarly, When a = 14; Number of sets = 3
Similarly, When a = 15; Number of sets = 2
Similarly, When a = 16; Number of sets = 2
Similarly, When a = 17; Number of sets = 1
Similarly, When a = 18; Number of sets = 1
So, total possible sets = 90
These three discounts can be given to three different customers in 6 different ways.
The number of ways shopkeeper can do this = 90 6 = 540
Hence, option 5.
Discuss the solution with Testfunda users.

Solution #32: (14-May-09)


The main points about allegory are: It is a figure of rhetoric and can be visual and not limited to
language. It is broader in meaning and is fuller and sustained longer than a metaphor or analogy. A
parable is an example. The above points are captured succinctly in option 3. In comparison the other
options are deficient in one or the other points.
Options 1, 2 and 5 do not liken an allegory to a parable, an important point in the paragraph
since this point makes the meaning of an allegory clear and definite.
Option 4 misses out on the visual part of an allegory.
Hence, the correct answer is option 3.
Discuss the solution with Testfunda users.

Book 2- Question of the Day

168

www.TestFunda.com

The LEADER in Online Test Prep

Solution #33: (15-May-09)


As ABCD is a kite, its diagonals are perpendicular to each other and AC bisects BD.
AB2 = AE2 + BE2 = 25

But, BD = 2 BE
AE BE = 12
Now, (AE + BE)2 = (AE2 + BE2) + (2 AE BE)
= 25 + 24 = 49
AE + BE = 7
The possible values of AE and BE are (6, 1), (5, 2), (4, 3).
But, (6, 1) and (5, 2) are not possible as AEB is right triangle.
AE = 4 and BE = 3 ( AE > BE)
BD = 6
So the sides of the ABD are (5, 5, 6).

Hence, option 2
Discuss the solution with Testfunda users.

Book 2- Question of the Day

169

www.TestFunda.com

The LEADER in Online Test Prep

Solution #34: (16-May-09)


Option 1 is opposite of what the ending should be, the person should ideally be moving towards
priesthood and not away from it, once he has acknowledged GOD.
In Option 2, but makes the paragraph completion incorrect because the previous sentence states
clearly that man acknowledges the presence of GOD and moving towards priesthood would be a
natural corollary to that.
In option 3, the belief overpowers everything the society does is inverse in meaning to the
previous sentence where man acknowledges a vague acknowledgement of the presence of GOD,
the acknowledgement is not strong enough to overpower everything the society does
In Option 5, and the belief counts for little in their lives should ideally be but the belief counts for
little in their lives since the previous sentence reveals mans acknowledgement of GOD.
Option 4 with but the belief counts for little in their lives best completes the sentence.
Hence, the correct option is option 4.
Discuss the solution with Testfunda users.

Solution #35: (17-May-09)

Let the radius of the semi-circle be R and the radius of the small circle be r.
For semi-circle, AB2 = AO2 + BO2
(2R)2 = (2a)2 + (2a)2
4R2 = 8a2

Book 2- Question of the Day

170

www.TestFunda.com

The LEADER in Online Test Prep

For small circle, OG = OI + IG

Area of the shaded region = Area of big circle Area of semi-circle AOB Area of small circle with
center I Area of the segment AHB

Hence, option 2.
Discuss the solution with Testfunda users.

Solution #36: (18-May-09)


Statements A, C and E are correct.
The correct usage in statement B would be The more haste, the less speed. Lesser is a
comparative degree of the verb to compare. Since the verb haste in the first half of the sentence is in
simple present tense, so will be less. The sentence is an epigram. An epigram is a brief pointed
saying frequently introducing antithetical ideas, which excite surprise and arrest attention.
Statement D is also incorrect because a funeral as modest magnificence is wrong. The correct
sentence is a funeral of modest magnificence. Modest magnificence describes the noun funeral.
Modest magnificence is an oxymoron. An oxymoron is a combination of contradictory or
incongruous words- a paradox reduced to two words, usually in an adjective-noun ("eloquent
silence") or adverb-adjective ("inertly strong") relationship, and is used for effect, complexity,
emphasis, or wit.
Hence the correct answer is option 5.
Discuss the solution with Testfunda users.

Book 2- Question of the Day

171

www.TestFunda.com

The LEADER in Online Test Prep

Solution #37: (19-May-09)

Chord BC bisects OL and chord AB bisects OM,

Now consider OGF,


OG2 = OF2 + GF2
(2a)2 = a2 + GF2

Consider OBF, OB2 = BF2 + OF2


(4a)2 = BF2 + a2

In ABE, ABE = 45

* ABE is isosceles triangle+

and OBP = 30

* OBP is right angled triangle and OB = 2OP+

Book 2- Question of the Day

172

www.TestFunda.com

The LEADER in Online Test Prep

OBF = 15
BOF = 90 15= 75
AOB = 2 75 = 150
Area of shaded region = Area of the sector O-ANB Area of the triangle OAB Area of the
semicircle

Hence, option 4.
Discuss the solution with Testfunda users.

Solution #38: (20-May-09)


Option 1 cannot be concluded because if what we eat is our medicine, it contradicts the passage.
Everyone has a doctor in him (the writer is referring to the healing force within) is not enough to
conclude that everyone is a doctor. Therefore option 2 is incorrect.
Option 4 is incorrect because nothing about the source of the healing force can be concluded.
Option 3 can be concluded we should help the healing force our food should be our medicine and
vice versa makes option 3 true.
Hence, the correct answer is option 3.
Discuss the solution with Testfunda users.

Solution #39: (21-May-09)


Statement C introduces so-called protoplasm or cellular substance. Therefore the ideas of cellprotoplasm, which essentially is an idea emanating from protoplasm, in statements A and E can only
follow after statement C is mentioned. Any option having statement A or E preceding statement C
can therefore be ruled out. Therefore, we can eliminate options 3, 4 and 5.
In option 2, statement A precedes E whereas A can only follow statement E. Statement A mentions
The nucleus is the more important of the two. The two are cell-protoplasm and nucleus
mentioned in E. Therefore, option 2 can also be eliminated.
By elimination the correct answer option is 1. Lets evaluate option 1
It starts with statement D which introduces the idea of tiny cell.
Statement C talks of its composition.
Statement B mentions that inspite of most widely differing elements (mentioned in statement C) it
is constant and unchanging.
Statement E takes the composition further with cell-protoplasm and nucleus and A ends with the
final detail- that is of the nucleus being the more important of the two.There is a continuation of
ideas and a smooth flow of specific details introduced in succession without any abrupt changes.
Hence, the correct answer is option 1.
Discuss the solution with Testfunda users.

Book 2- Question of the Day

173

www.TestFunda.com

The LEADER in Online Test Prep

Solution #40: (22-May-09)


Statement C introduces so-called protoplasm or cellular substance. Therefore the ideas of cellprotoplasm, which essentially is an idea emanating from protoplasm, in statements A and E can only
follow after statement C is mentioned. Any option having statement A or E preceding statement C
can therefore be ruled out. Therefore, we can eliminate options 3, 4 and 5.
In option 2, statement A precedes E whereas A can only follow statement E. Statement A mentions
The nucleus is the more important of the two. The two are cell-protoplasm and nucleus
mentioned in E. Therefore, option 2 can also be eliminated.
By elimination the correct answer option is 1. Lets evaluate option 1
It starts with statement D which introduces the idea of tiny cell.
Statement C talks of its composition.
Statement B mentions that inspite of most widely differing elements (mentioned in statement C) it
is constant and unchanging.
Statement E takes the composition further with cell-protoplasm and nucleus and A ends with the
final detail- that is of the nucleus being the more important of the two.There is a continuation of
ideas and a smooth flow of specific details introduced in succession without any abrupt changes.
Hence, the correct answer is option 1.
Discuss the solution with Testfunda users.

Solution #41: (23-May-09)


BC = Side of the small equilateral triangle = 3a units

The point O divides AH (median) in the ratio 2 : 1

Area of the shaded region = Area of semicircle F-DOE Area of the rhombus OBGC (Area of
trapezium BCED Area of semicircle H-BFC)

Hence, option 2.
Discuss the solution with Testfunda users.
Book 2- Question of the Day

174

www.TestFunda.com

The LEADER in Online Test Prep

Solution #42: (24-May-09)


Options 3 and 5 give us a negative result on reading the gospel which is untrue and opposite to what
the penultimate statement in the paragraph wants us to do, that is to free our mind of prejudice and
read the gospel.
Option 1 is vague and hidden with sinister meaning which is not intended by the author.
Option 4 is frivolous and cannot be inferred from the paragraph. The word Although does not link
well with the penultimate statement of the paragraph.
Therefore option 2: that it would bring a portrait of freshness and power to those who read the
gospel without prejudice is a positive inference and one which the author intends.
Hence, the correct answer is option 2.
Discuss the solution with Testfunda users.

Solution #43: (25-May-09)


Given , are the roots of the equation x2 + px + q 2 = 0
+ = p and = q 2

Also , are the roots of the equation x2 px + q 4 = 0


+ = p and = q 4

It is given that = 48
(q 2)(q 4) = 48
q2 6q 40 = 0
q2 10q + 4q 40 = 0
q(q 10) + 4(q 10) = 0
(q 10)(q + 4) = 0
Book 2- Question of the Day

175

www.TestFunda.com

The LEADER in Online Test Prep

q = 10 or 4
Also, it is given that

If q = 10, then we have

p=9
If q = 4, then we have

p=9
Thus, in both the cases i.e. q = 10 or 4, p has a unique value of 9.
Hence, option 1.
Discuss the solution with Testfunda users.

Solution #44: (26-May-09)


The paragraph begins with heat kills - then it does not discuss anything else. The heat then
becomes global warming. Thats all. The last statement cannot bring in anything new or that which
requires clarification. This eliminates options 3, 4 and 5.
It is a close choice between options 1 and 2. Option 2 loses out on account of introducing new
elements such as lack of resources and health care.
Option 1 continues on the same theme. The only statement among the ones given in the options
that can be considered as the last statement of the given paragraph.
Hence, the correct answer is option 1.
Discuss the solution with Testfunda users.
Book 2- Question of the Day

176

www.TestFunda.com

The LEADER in Online Test Prep

Solution #45: (27-May-09)


Given , , , form an increasing G.P.
Let = a, = ar, =ar2 and = ar3
Given that , are the roots of the equation x2 4x + p = 0
+=4
a + ar = 4
a(1 + r) = 4
(i)
Also, given that , are the roots of the equation x2 36x + q = 0
+ = 36
ar2 + ar3 = 36
ar2(1 + r) = 36
(ii)
Dividing equation (ii) by equation (i), we get r2 = 9
Thus we have r = 3 or 13
But since r is the common ratio of an increasing G.P, it cannot be negative.
r=3
Substituting r = 3 in equation (i), we get a = 1
We have = 1, = 3, = 9 and = 27
Also, we have = p and = q
p + q = 3 + 243 = 246
Hence, option 5.
Discuss the solution with Testfunda users.

Solution #46: (28-May-09)


This question is interesting becuase of the technical nature of the content, rather than the inherent
difficulty of solving it.
The CD link can be deciphered. Statement C mentions what the different transistor packages are
made of while statement D mentions that the type of package often dictates the power rating and
frequency characteristics. Therefore, options 2,4 and 5 can be eliminated.
The BA link is also clear. Statement B introduces surface mount device or SMD while A mentions
the reason why SMDs have better high frequency characteristics. Therefore, option 1 can be
eliminated.
Hence, the correct answer is option 3.

Discuss the solution with Testfunda users.

Book 2- Question of the Day

177

www.TestFunda.com

The LEADER in Online Test Prep

Solution #47: (29-May-09)

The sum of all the possible values of N = 11C0 + 11C1 + 11C2 + + 11C11 = 211 = 2048
Hence, option 4.
Discuss the solution with Testfunda users.

Solution #48: (30-May-09)


Option 1 inverts what the people of Vondervotteimittis are and therefore can be eliminated.
Option 2 and 3 can both be considered as true for the people of that town but the best answer
option is option 4, which captures the ideas of both options 2 and 3 that is, change (alterations) and
adventure (risk). Hence, the correct answer is option 4.
Discuss the solution with Testfunda users.

Book 2- Question of the Day

178

www.TestFunda.com

The LEADER in Online Test Prep

Solution #49: (31-May-09)


Let the first term of the geometric progression be a and the common ratio be r.
The nth term of a geometric progression is given by tn= arn 1
We need to determine the value of the thirteenth term i.e. ar12.
From statement A:
Given that t6 = 4
ar5 = 4
But from this we cannot determine the value of ar12.
Statement A alone is not sufficient.
From statement B:
Given that t10 t16 = 4096
ar9 ar15 = 212
a2r24 = 212
(ar12)2 = (26)2
ar12 = (26)
We cannot determine the unique value of the thirteenth term from statement B alone.
Statement B alone is also not sufficient.
From statement A and B together:
ar5 is positive, either both a and r are positive or both are negative.
When both a and r are positive, ar12 is positive, but when both a and r are negative, ar12 is negative.
We cannot determine the unique value of the thirteenth term from statement A and B together
also.
The question cannot be answered on the basis of the two statements.
Hence, option 5
Discuss the solution with Testfunda users.

Book 2- Question of the Day

179

www.TestFunda.com

The LEADER in Online Test Prep

Solution #50: (01-Jun-09)


The paragraph begins by stating the beneficial effects of beet root juice on hypertension and ends
(without the last statement) by stating that it might be an effective low cost treatment for
hypertension.
A statement that will continue this is one which points out in what other ways it could be beneficial.
Option 1 does this by suggesting that beetroot as well as nitrate rich vegetables can contribute to a
healthy cardiovascular system.
Options 2 and 4 try to undermine the findings of the paragraph. This is not warranted and does not
make for an effective conclusion.
Option 3 does nothing to conclude the paragraph. It merely reiterates something that has already
been mentioned in the paragraph.
Option 5 excludes the nitrate part and brings in a diet rich in fruit and vegetables- a new
element.
Hence, the correct answer is option 1.

Discuss the solution with Testfunda users.

Solution #51: (02-Jun-09)


Given , are the roots of the equation 7x2 5x + 2 = 0

Since, the product of the roots is positive and their sum is also positive, therefore, both the roots are
positive. Also, the sum is less than 1, therefore both the roots are less than 1.
Now, we use the formula for sum of infinite geometric progression, for positive common ratio < 1.
(1 + + 2 + 3 + ... up to )(1 + + 2 + 3 + ... up to )

Hence, option 2.
Discuss the solution with Testfunda users.
Book 2- Question of the Day

180

www.TestFunda.com

The LEADER in Online Test Prep

Solution #52: (03-Jun-09)


The paragraph states that men working on building a railway were being attacked by lions and the
native coolies were convinced that the spirits of two chiefs had taken this form to protest against a
railway being built through their country. Thus, it can be inferred that the lions were not against
humans building the railways, but against the building of the railway- an important distinction to
take note of.
Option 1 is therefore incorrect because the attack on humans was not the way to seek revenge for
the lions, but to somehow stop the railways from being built.
Options 3 and 4 contradict the paragraph.
Options 2 and 5 are close, option 2 scores over option 5 by mentioning stopping its progress
meaning the progress of the railways- not merely stopping work in the area.
Hence, the correct answer is option 2.
Discuss the solution with Testfunda users.

Solution #53: (04-Jun-09)


p(4, 16) = p(5, 16) + p(4, 12)
p(4, 12) = p(5, 12) + p(4, 8)
p(4, 8) = p(5, 8) + p(4, 4)
p(4, 4) = 1
Consider p(5, 8),
p(5, 8) = p(6, 8) + p(5, 3)
p(5, 3) = 0 ( 3 < 5)
p(5, 8) = p(6, 8)
p(6, 8) = p(7, 8) + p(6, 2)
p(6, 2) = 0 ( 2 < 6)
p(6, 8) = p(7, 8)

(i)
(ii)

Similarly, p(7, 8) = p(8, 8) = 1


p(5, 8) = 1
p(4, 8) = p(5, 8) + p(4, 4) = 1 + 1 = 2
(iii)
p(5, 12) = p(6, 12) + p(5, 7)
Consider p(6, 12),
p(6, 12) = p(7, 12) + p(6, 6)
p(6, 6) = 1
p(7, 12) = p(8, 12) + p(7, 5)
p(7, 5) = 0 ( 5 < 7)
Continuing in the same manner as above we get,
p(7, 12) = 1
p(5, 7) = p(6, 7) + p(5, 2)
p(5, 2 ) = 0 and p(6, 7) = 1
p(5, 7) = 1
p(6, 12) = p(7, 12) + p(6, 6) = 1 + 1 = 2
p(5, 12) = p(6, 12) + p(5, 7) = 2 + 1 = 3
(iv)
Substituting (iii) and (iv) in (ii) we get,
p(4, 12) = 2 + 3 = 5
(v)
Similarly we can find, p(5, 16) = 6
(vi)
Substituting (v) and (vi) in (i), we get,
p(4, 16) = 11
Hence, option 1.
Discuss the solution with Testfunda users.
Book 2- Question of the Day

181

www.TestFunda.com

The LEADER in Online Test Prep

Solution #54: (05-Jun-09)


Option 1 is judgmental as it refers to civilization as a farce.
Option 2 talks about problems not mentioned in the paragraph.
Option 4 is advisory as it mentions- dont vanquish nature.
Option 5 is advisory too as it suggests that man should embrace nature as well as his fellow
creatures.
Option 3 captures the gist of the paragraph.
Hence, the correct answer is option 3.
Discuss the solution with Testfunda users.

Solution #55: (06-Jun-09)

On rearranging the equation, we get,

On squaring both the sides, we get,

On squaring both the sides, we get,

On squaring both the sides, we get,


16x = 25x2
x(25x 16) = 0
Either x = 0 or 25x 16 = 0
But x 0, because x is positive.

Hence, option 3.
Discuss the solution with Testfunda users.

Book 2- Question of the Day

182

www.TestFunda.com

The LEADER in Online Test Prep

Solution #56: (07-Jun-09)


Option 1 is incorrect because there is no justification that impressionists were an odd bunch with
artists of all extremities forming the group. Also Renoir was not a pure impressionist.
Option 2 is incorrect because both Renoir and Manet were not pure impressionists.
Option 4 is incorrect because pure impressionists were the only true impressionists is not
mentioned in the paragraph.
Option 5 is incorrect because in the face of criticism from its proponents, impressionists were
consistent in their pursuit of an art of spontaneity, sunlight, and colour is inconsistent with what
has been mentioned in the paragraph.
Option 3 is correct because it summarizes all the main ideas of the paragraph.
Hence, the correct answer is option 3.
Discuss the solution with Testfunda users.

Solution #57: (08-Jun-09)


The speed of the rail engine is 42 km/hr when no compartment is attached.
Now, the reduction in speed (say v) is directly proportional to the square root of the number of
compartments (say n) attached.

[Here, k is some positive constant]


Reduction in speed of the train (v) = 42 24 = 18 km/hr when 9 compartments are attached to
the rail engine, so we get,
k=6
Let the number of compartments attached be x, when the speed of the engine reduces to zero.
Therefore, reduction in speed of the rail engine = 42 0 = 42 km/hr

x = 49
But when 49 compartments are attached, the speed of the engine reduces to zero,
Maximum number of compartments should be one less than 49, i.e. 48, so that the rail engine can
move.
Hence, option 1.
Discuss the solution with Testfunda users.

Solution #58: (09-Jun-09)


This can be added as an extra premise to the whole argument. For example , the statement can
read: A stupid mans report of what a clever man says can never be accurate, because he
unconsciously translates what he hears into something he can understand, and a stupid person can
report only what he understands. None of the other options can act as an assumption required to
make the statement true.
Option 5 is incorrect because the statement only says he cannot report accurately, it does not say
that he cannot report at all.
Option 1 cannot be deciphered from the statement as to whether stupidity and cleverness are
relative or absolute.
Similarly, options 2 and 3 cannot be implied from the statement.
Hence, the correct answer is option 4.
Discuss the solution with Testfunda users.
Book 2- Question of the Day

183

www.TestFunda.com

The LEADER in Online Test Prep

Solution #59: (10-Jun-09)

On rationalizing the denominator of the LHS, we get,

Hence, option 1
Discuss the solution with Testfunda users.

Solution #60: (11-Jun-09)


Option 1 strengthens the case because the charges are discrimination and stereotyping.
Option 2 is not a sufficient reason to bar Garcia or to weaken his case.
Option 3 is does not weaken his case that since no woman complains, therefore he should not as
well nor does it prevent him from complaining.
Option 5 seems to weaken his case, but it does not address the issues raised by Garcia.
Option 4 weakens his case because if it is meant to honour housewives, men will first have to prove
that they are housewives- the generally understood meaning of the term housewife is that it is a
woman confined to the house.
Hence, the correct answer is option 4.
Discuss the solution with Testfunda users.

Book 2- Question of the Day

184

www.TestFunda.com

The LEADER in Online Test Prep

Solution #61: (12-Jun-09)


Two digit number XY can be written as 10X + Y.
Now 10X + Y = 7X + 3X + Y
Since 7X is divisible by 7, 10X + Y i.e. the number XY will be divisible by 7, if 3X + Y is divisible by 7.
From statement A alone:
Given X + Y = 7
If X = 7 and Y = 0, then the number XY is divisible by 7.
If X = 3 and Y = 4, then XY is not divisible by 7.
Hence statement A alone is not sufficient.
From statement B alone:
Given 4X + 6Y is divisible by 7.
We know that 7X + 7Y is divisible by 7.
i.e. 4X + 3X + 6Y + Y is divisible by 7.
i.e. (4X + 6Y) + (3X + Y) is divisible by 7.
Since 4X + 6Y is divisible by 7, we can conclude that 3X + Y is also divisible by 7.
Hence XY is divisible by 7.
So statement B alone is sufficient.
Hence, option 2.
Discuss the solution with Testfunda users.

Solution #62: (13-Jun-09)


The argument that needs to be strengthened is that the government should act on the issue of
deciding what works and what does not in alternative medicine.
Option 1 states that consumer protection laws should be strengthened and does not directly
address the argument of the pharmacologist that the government should first decide what works
and what does not.
Option 2 states that alternative medicines do not work- which has to be decided by the
government.
Option 4 does not address the issue of alternative medicine.
Option 5 like option 1 does not call for government intervention.
Option 3 strengthens the argument by stating that agencies constituted to licence conventional
drugs, excuse homeopathy from proving whether it works at all. This calls for the need for
government intervention to decide whether homeopathy- or alternative medicine- really works.
Hence, the correct answer is option 3.
Discuss the solution with Testfunda users.

Book 2- Question of the Day

185

www.TestFunda.com

The LEADER in Online Test Prep

Solution #63: (14-Jun-09)

On squaring both the sides of the equation, we get,

On squaring both the sides of the equation, we get,

But, this value of x does not satisfy the given equation.


Hence, option 5.
Discuss the solution with Testfunda users.

Book 2- Question of the Day

186

www.TestFunda.com

The LEADER in Online Test Prep

Solution #64: (15-Jun-09)


Option 5 attributes the cause of the decline of Afghanistan and Iraq to acts of terrorism rising from
anti-west sentiments and ideologies. Thus, option 5 in effect attributes the cause of peace declining
to factors other than economic downturn. The conclusion of the study states that it is economic
downturn that has made the world more violent. The economic factors are not included in the case
of Afghanistan and Iraq, hence this challenges the conclusion.
The other options, 1 to 4, support the conclusion.Thus, by process of elimination also, option 5 can
be selected.
Hence, the correct answer is option 5.
Discuss the solution with Testfunda users.

Solution #65: (16-Jun-09)


Consider statement I alone:
M = (1 + 2)n 2n 1
Since n is a natural number, using the binomial expansion formula for n 0, (1 + x)n = nC0 + nC1x +
n
C2x2 + + nCnxn
M = (1 + 2)n 2n 1 = (1 + 2n + nC2 22 + + nCn 2n) 2n 1
= 22(nC2 + nC3 2 + . + nCn 2n 2)
So M is divisible by 4 (i.e. 22), if n > 1.
Statement I is sufficient to answer the question.
Consider statement II alone:
n3 > 1, hence n > 1.
Now, for fractional values of n, M will not be divisible by 4, whereas for integer values of n in the
range given, M will be divisible by 4.
Statement II alone is not sufficient to answer the question.
Hence, option 1.
Discuss the solution with Testfunda users.

Book 2- Question of the Day

187

www.TestFunda.com

The LEADER in Online Test Prep

Solution #66: (17-Jun-09)


The purpose of the paragraph is to point out that traditional marketing models based on a need gap
approach will not work with the youth as they have no need gap. Option 4 brings this paragraph to a
smooth closure.
Option 1 and option 3 which may be considered (erroneously) for the answer states certain needgaps need for liberation and empowerment(option 1), and the need for legitimisation(option
3). Tthe youth has no such need gaps.
Hence, the correct answer is option 4.
Discuss the solution with Testfunda users.

Solution #67: (18-Jun-09)


Suppose that 13 children transact with the 14th child. Each of them retains 2 of his own kind with
himself and gives away 13 of his kind to the 14th child. The 14th child gives one chocolate of his kind
in return to each of the 13 children. So, now the 14th child has 2 chocolates of his kind and 13 13
chocolates of 13 different kinds. The 13 children have 2 chocolates each of their own kind and 1
chocolate each of the 14th kind.
Next, the 14th child takes away 14 chocolates from the 15th child and gives him one chocolate each of
the 14 kinds that he has. So, now the 14th and 15th children have all the 15 kinds of chocolates.
Now, again the 14th child transacts with each of the 13 children and gives them one chocolate each
of the 13 kinds that they do not have and takes one chocolate each of their own kind from them.
Now all the children have all the 15 kinds of chocolates.
This kind of exchange of chocolates gives the minimum number of transactions.
The total number of transactions here = 13 + 1 + 13 = 27
Hence, option 1.
Discuss the solution with Testfunda users.

Solution #68: (19-Jun-09)


Option is 1 is incorrect for the generalisation about the car industry; they are Detroits big three.
Option 2 and option 3 are possibilities; they are not conclusive the question is not about
inferences, but about what can be validly concluded they cannot be validly concluded.
Option 4 is likely but not conclusive as GMs bankruptcy depends on an if.
Option 5 is well supported by the paragraph and especially by the first line of the paragraph the
paragraph is written in order to explain the conclusion.
Hence, the correct answer is option 5.
Discuss the solution with Testfunda users.

Book 2- Question of the Day

188

www.TestFunda.com

The LEADER in Online Test Prep

Solution #69: (20-Jun-09)


|x2 y2| = n
|(x + y)(x y)| = n
It is very clear that statement I is necessary to answer the question, because statements II or III
alone or II and III together are not sufficient to answer the question.
Therefore let us consider different cases taking statement I as true i.e. if it is given that (x + y) is odd.
Case 1:
If x = 3 and y = 2, (x + y) = 3 + 2 = 5 (odd)
(x y) = 3 2 = 1 and n = |(x + y)(x y)| = |5 1| = 5 (odd)
Case 2:
If x = 1.5 and y = 1.5, (x + y) = 1.5 + 1.5 = 3 (odd)
(x y) = 1.5 1.5 = 0 and n = |(x + y)(x y)| = |3 0| = 0 (not an odd number)
But, this case can be eliminated by statement II, i.e. if we take x y.
Case 3:
If x = 2.6 and y = 0.4, (x + y) = 2.6 + 0.4 = 3 (odd)
(x y) = 2.6 0.4 = 2.2 and n = |(x + y)(x y)| = |3 2.2| = 6.6 (fraction, not an odd number)
But, this case can be eliminated by statement III, i.e. if we take n as an integer.
Case 4:
If x = 3.5 and y = 1.5, (x + y) = 3.5 + 1.5 = 5 (odd)
(x y) = 3.5 1.5 = 2 and n = |(x + y)(x y)| = |5 2| = 10 (Even)
We can see that, even if we consider all the three statements I, II and III together, case 1 and 4 are
valid and giving different answers.
The question cannot be answered uniquely by using all the three statements I, II and III together.
Hence, option 4.
Discuss the solution with Testfunda users.

Solution #70: (21-Jun-09)


The purpose of the paragraph is to state the challenge of making animal rights issue get an
important role in politics the challenge is because tigers, buffaloes and birds dont have any
votes. Option 1 explicitly addresses both the issues and closes the paragraph. The sentence
immediately next to the last states the potential to protect wild life. Now In addition .... what
else can it do? Option 1 fits the bill it can attract political interest the challenge can be met.
Option 2 is eliminated because of forefront.
Option 3 is eliminated because mere mass mobilisation is not the purpose of the paragraph.
Option 4 is incorrect .Animal rights issue is already addressed in the second last sentence as actually
protect wildlife. It cannot become an important issue in politics unless politicians are involved.
Option 5 too is logically abrupt and whole attention eliminates it.
Hence, the correct answer is option 1.
Discuss the solution with Testfunda users.

Book 2- Question of the Day

189

www.TestFunda.com

The LEADER in Online Test Prep

Solution #71: (22-Jun-09)

We will first find the area of the sector P-RQS and then subtract the area of PRS from it so that we
can get the area of the segment RQSR.
In PRT, PR = 2a and PT = a
mPRT = 30 and mRPT = 60

Area of segment RQSR = Area of sector P-RQS Area of PRS

Also, Diagonal of the square = Radius of circle


Diagonal of the square = 2a

Area of shaded region = 2 *Area of the circle Area of the square 2 Area of the segment RQSR+

Hence, option 3.
Discuss the solution with Testfunda users.
Book 2- Question of the Day

190

www.TestFunda.com

The LEADER in Online Test Prep

Solution #72: (23-Jun-09)


Option 1 may explain why more young people are affected, but it will not explain the reverse pattern
- hence is not adequate.
Option 2 is contrary to what is stated about swine flu.
If option 3 is true, more old people should suffer from swine flu.
If option 5 is correct, young people should not be affected by swine flu.
Option 4, if true, could explain why the old people are not affected that their bodies have been
used to similar viruses hence have certain immunity to these viruses.
Hence, the correct answer is option 4.
Discuss the solution with Testfunda users.

Solution #73: (24-Jun-09)


A short way of solving questions with inequalities is to substitute suitable values for the variables as
per the conditions in the question and then as per the conditions given in the statements.
Using statement I alone:
n is an even number.
Taking n = 2, we get the L.H.S as 2! i.e. 2

Now, if the value of x is 0, the value of the above expression becomes 1.

However, if the value of x is 2, the value of the R.H.S becomes 2.25

Statement I is not sufficient to answer the question.


Using statement II alone:
x=n

Taking various integral vales for n we get the following values for both the sides

Book 2- Question of the Day

191

www.TestFunda.com

The LEADER in Online Test Prep

Statement II is insufficient to answer the question.

Using both the statements:


The possible values of n are 2, 4, 6 and so on.
From the table in statement II we find that apart from the case when n = 1

Both the statements together are sufficient to answer the question.


Hence, option 4.

Alternatively,
Arithmetic Mean Geometric Mean
The Arithmetic Mean of the first n natural numbers their Geometric mean

Book 2- Question of the Day

192

www.TestFunda.com

The LEADER in Online Test Prep

Using statement I alone:


n is an even number.
From equation (i), the given inequality will be true if x n.
But we do not know the value of x.
Statement I is not sufficient to answer the question.
Using statement II alone:
x=n

The equality holds when x = n = 1


Statement II is insufficient to answer the question.
Using both the statements:
x = n and n is even,
x>1

Both the statements together are sufficient to answer the question.


Hence, option 4.
Discuss the solution with Testfunda users.

Solution #74: (25-Jun-09)


We need to first understand a critic has written the paragraph about Kallats art and then uses
probably one of his quotes to close the paragraph. The urban milieu of Mumbai remains his primary
muse... the details are of this life are spelt out in the paragraph. An apt comment from the artist
will explain this muse of the artist. No other comment of the writer is directly related to what the
critic has explained in the paragraph. In order to explain the influence of Mumbai life on the artist
the critic uses the quote from the writer stated in option 5. No other option would help the critic this
way.
Hence, the correct answer is option 5.
Discuss the solution with Testfunda users.
Book 2- Question of the Day

193

www.TestFunda.com

The LEADER in Online Test Prep

Solution #75: (26-Jun-09)


Equation of the circle is x2 + y2 4x 1 = 0
x2 + y2 4x 1 + 5 = 5
x2 4x + 4 + y2 = 5

Comparing the above equation with the general equation of a circle,


(x h)2 + (y k)2 = r2, we get,
Centre of the circle C (2, 0) and

Now, as shown in the figure, point P can take positions, P and P.


(CP) = (CP) = (CQ)
CQP = CPQ and CQP = CPQ
But, CQP = CQP = 60
CPQ = CPQ = 60
CPQ and CPQ are equilateral triangles.

Hence, option 1.
Discuss the solution with Testfunda users.

Book 2- Question of the Day

194

www.TestFunda.com

The LEADER in Online Test Prep

Solution #76: (27-Jun-09)


Since the experts clearly attribute the success (among other factors) to the ad campaigns of the
winners - for focussing on 'positivity, inclusiveness, youthfulness, pan-India appeal etc., option 1 is
contrary and not the assumption.
The last sentence may justify option 2, but it is not the view of the political experts.
Option 3 is clearly the assumption when they attribute the success (at least partly) to the ad
campaigns.
Option 4, though true, is not an assumption in the analysis - instead it is the opinion of the author.
Option 5 is incorrect because the experts do not believe it (attributing success and failure in the
elections to ad campaigns) is an oversimplification they believe it contributed to the success.
Hence, the correct answer is option 3.
Discuss the solution with Testfunda users.

Solution #77: (28-Jun-09)


The largest possible 6-digit odd number is 999999.
From statement I alone,
The average of the first two odd prime numbers = (3 + 5)/2 = 4
(X/2)2 = 4X
X = 16
Sum of the digits of X is 1 + 6 = 7
Now we can find the remainder when 999999 is divided by 7.
[The remainder will be zero in this case, as every 6 digit number in which first three digits are same
as last three digits is always divisible by 7, 11 and 13.]
Hence, statement I alone is sufficient.
From statement II alone,
We have many two digit numbers and we cannot find any unique answer.
Hence, statement II alone is not sufficient.
Hence, option 1.
Discuss the solution with Testfunda users.

Book 2- Question of the Day

195

www.TestFunda.com

The LEADER in Online Test Prep

Solution #78: (29-Jun-09)


Not a difficult choice if one notices the direction of the paragraph. Models were celebrities then
models were Bollywood and cricket stars now they are little known but well remembered
characters hence, they are no longer celebrities or stars though one can still put a face to them
(the face is remembered through the character).
Great performance in option 1 is an overstatement about ads.
Option 3 states that they are household names which is not true.
Option 4 states they are quickly forgotten which is not true.
'Cultures' and 'identities' in option 5 are irrelevant.
Hence, the correct answer is option 2.
Discuss the solution with Testfunda users.

Solution #79: (30-Jun-09)


Let P win a games and lose b games.
Total number of games played by all three = a + b
Also, the total number of games won by P and Q will be b
Let Q win c games.
Q loses (a + b c) games.
R wins (a + b) (a + c) = (b c) games and loses (a + b) (b c) = (a + c) games.
5a 3b = 20
...(i)
5c 3(a + b c) = 12
8c 3a 3b = 12
...(ii)
5(b c) 3(a + c) = 12
5b 3a 8c = 12
...(iii)
From (i) and (ii),
ca=1
...(iv)
From (iii) and (iv),
5b 11a = 20
...(v)
From (i) and (v),
a = 5 and b = 15
c=6
The three played 5 + 15 = 20 games in all.
Hence, option 2.
Alternatively,
If P, Q and R win p, q and r games respectively, they lose (q + r), (p + r) and (p + q) games
respectively.
As a win gives 5 points and a loss 3 points,
5p 3q 3r = 20
...(i)
5q 3p 3r = 12
...(ii)
5r 3q 3p = 12
...(iii)
Adding (i), (ii) and (iii),
p q r = 20
p + q + r = 20
Hence, option 2.
Discuss the solution with Testfunda users.
Book 2- Question of the Day

196

www.TestFunda.com

The LEADER in Online Test Prep

Solution #80: (01-Jul-09)


Motivation of employees, and innovation of ideas (in HR) mentioned in the passage are for the
purpose of running a successful organisation, which makes options 1 and 2 correct.
Option 3 is about market perceptions which are not within the scope.
Option 4 could be considered only if it was motivated employees.
The first 3 sentences of the paragraph indicate that options 1 and 2 are the basic assumptions in the
argument.
Hence, the correct answer is option 5.
Discuss the solution with Testfunda users.

Solution #81: (02-Jul-09)


Perimeter of ABCD = length of arc AB + 6 + length of arc DC + 6 = 22 cm
Length of arc AB + length of arc DC = 22 12 = 10 cm

(i)

Sector OAB and sector ODC are similar,

From equation (i),


Length of arc AB = 2 cm and length of arc DC = 8 cm
Now, area of figure ABCD = area of sector ODC area of sector OAB

= 32 2
= 30 cm2
Hence, option 1.
Discuss the solution with Testfunda users.

Solution #82: (03-Jul-09)


People walking in the streets look out for different things. ... Me, I look out for bananas . But not
any bananas .... This is where the writer breaks off to Cavendishes. The paragraph is incomplete
unless we know what kind of bananas he is looking for. This makes option 4 the correct answer. The
other options give more information about either the Cavendishes or the Americans; they do not
fully close/complete the paragraph.
Hence, the correct answer is option 4.
Discuss the solution with Testfunda users.

Book 2- Question of the Day

197

www.TestFunda.com

The LEADER in Online Test Prep

Solution #83: (04-Jul-09)


We have, cos(x y), cos x, cos(x + y) are in H.P.
If three numbers a, b and c are in H.P. then,

Hence, option 3.
Discuss the solution with Testfunda users.
Book 2- Question of the Day

198

www.TestFunda.com

The LEADER in Online Test Prep

Solution #84: (05-Jul-09)


.... for 'am admi', the purpose of national disaster management is two-fold: the national
government must promote safe economic growth as well as help reduce risks faced by the poor. if
these two things do not go hand in hand they cannot be achieved, making the demands, by default,
unattainable. A literal examination of this statement indicates the first option.The other options do
not prove this and can be ruled out.
Hence, the correct answer is option 1.
Discuss the solution with Testfunda users.

Solution #85: (06-Jul-09)


x is the largest 3 digit number which when divided by 3 or 8 leaves a remainder of 2.
x should leave a remainder of 2 when divided by the LCM of 3 and 8 i.e 24
We are interested in the largest three digit number, we should first find out the largest three digit
multiple of 24.
This can be obtained by first dividing 999 by 24 which leaves a remainder of 15.
999 15 = 984 is the largest three digit multiple of 24
Now adding the remainder 2 to the number gives 986 which is the required value of x.
y is the largest number with which when 486, 686 and 986 are divided the remainders are the
same.
Taking the difference between any two out of the three numbers,
686 486 = 200 and 986 686 = 300
y is the HCF of these two differences.
y is the HCF of 200 and 300 which is 100.
If a number N can be expressed as:
N = ap bq , where a and b are prime
then the number of co-primes of N which are less than N is given by,

Now, 100 = 22 52
Number of co-primes of 100 less than 100 is given as

Number of co-primes of 100 less than 100 is 40.


Hence, option 4.
Discuss the solution with Testfunda users.
Book 2- Question of the Day

199

www.TestFunda.com

The LEADER in Online Test Prep

Solution #86: (07-Jul-09)


The paragraph states that lycopene ingested in its natural form is not absorbed - hence it cannot
reduce the oxidation of harmful fats in the blood and thus stave off heart disease. Option 3 states
that consumption of tomatoes can help reduce oxidation of fats whereas it cannot.
All the other options help strengthen what is stated in the paragraph.
Hence, the correct answer is option 3.
Discuss the solution with Testfunda users.

Solution #87: (08-Jul-09)


If we join the centers of each circle we get the following figure.

Here AB = AE + EB
The radius of each circle is 1 metre.
AE = EB = 1
AE + EB = 2 metres
DL = LB
DL + LB = 2 metres
AD = 2 metres
Similarly, BM = MC
ABD and ABC are equilateral triangles with side = 2 metres
AE = EB
DE is the median of ABD
In an equilateral triangle the median is also the angle bisector as well as the perpendicular
bisector.
DE is also the angle bisector for ADB
m BDE = 30
PE passes through the diameter of the circle with centre D,
m PDB = mPDL = 150
Book 2- Question of the Day

200

www.TestFunda.com

The LEADER in Online Test Prep

m DBA + m CBA = 60 + 60 = 120


m LBE + m MBE = 120
Angle subtended by the major arc LM = 240

Hence, option 3.
Alternatively,
We know circumference of circle = 2 radius
A circle corresponds to 360.
We have
Arc PL = 150
Arc LM = 240
Arc LQ = 150
Total arc degree = 150 + 240 + 150 = 540
Arc of 360 corresponds to a length of 2 radius.
Arc of 540 corresponds to a length of

Hence, option 3.
Discuss the solution with Testfunda users.

Book 2- Question of the Day

201

www.TestFunda.com

The LEADER in Online Test Prep

Solution #88: (09-Jul-09)


Option 1, if true, would result in Maruti's share of sales in Suzuki Motor going up in percentage
terms- not necessarily profits.
Options 2 and 3 mention increase in sales of Maruti's products- that does not necessarily translate
into increase in its share of profits.
Option 4 can be eliminated as we cannot infer from the information given in the option- launch of
Suzuki's top line models in the Indian market first- whether that translated into Maruti increasing its
share of Suzuki's profits.
If shares of profits of other subsidiaries of Suzuki Motors in various markets had declined in the fiscal
year- then we can infer that Maruti's share of profits in Suzuki must have increased.
Hence, the correct answer is option 5.
Discuss the solution with Testfunda users.

Solution #89: (10-Jul-09)


Consider statement A alone,
We can deduce that as the polygon has only two diagonals it has to be a quadrilateral.
But we cannot conclude anything about its structure.
The question cannot be answered on the basis of statement A alone
Consider statement B alone,
It just says that the sum of products of all pairs of opposite sides of the polygon is 45cm2.
From statement B we cannot conclude anything.
The question cannot be answered on the basis of statement B alone.
Consider statements A and B together,
From statement A we deduce that the given polygon is a quadrilateral and the product of its
diagonals is 45 cm2.
From statement B we know that the sum of product of all pairs of opposite sides of the polygon is
also 45 cm2.
We know that, for a cyclic quadrilateral sum of product of all pairs of opposite sides is equal to
product of its diagonals.
We conclude that the given polygon is cyclic.
We can answer the question by using statements A and B together.
Hence, option 4.
Discuss the solution with Testfunda users.

Book 2- Question of the Day

202

www.TestFunda.com

The LEADER in Online Test Prep

Solution #90: (11-Jul-09)


The question in option 1 is a rhetorical question asked not to elicit an answer but to state its
negative as the answer. The dubious reputation of IRDA and the appointment of a retired IAS/IRS to
the post of ombudsman becomes a feather in its cap when it is certain that the appointment is
faulty.
Options 2, 3 and 5 explain certain terms that may require clarification to an uniformed reader but do
not help complete the paragraph.
Option 4 can be considered for the answer. If the appointment had been made option 4 could have
served as the answer. Since only applications have been invited and the possibility of the stranglehold is increasing, the first question (option) is more relevant than an exclamation. The possibility
more aptly closes it than the categorical exclamation.
Hence, the correct answer is option 1.
Discuss the solution with Testfunda users.

Solution #91: (12-Jul-09)


Using statement A alone,
From statement A we can determine only the distance between point P and vertex A.
From this we cannot determine distance of point P from the sides of the triangle.
The question can not be answered by using statement A alone.
Using statement B alone,
Since we know the co-ordinates of vertices B and C, we can find the length of the sides of the
equilateral ABC.
We can determine the height of ABC.

Let s and h be the side and height of ABC respectively.


Now, Area of ABC = Area of ABP + Area of ACP + Area of BPC

h = PD + PF + PE
As the length of each side of ABC is known, the height h can be determined.
The question can be answered by using statement B alone.
Hence, option 2.
Discuss the solution with Testfunda users.
Book 2- Question of the Day

203

www.TestFunda.com

The LEADER in Online Test Prep

Solution #92: (13-Jul-09)


We need to look for an option that most seriously weakens the hypothesis. It need not destroy the
hypothesis.
Option 3 by stating that a plane is fitted with mechanisms that can dissipate electricity safely and
completely lightning is electricity weakens the hypothesis that the plane was damaged by
lightning/electricity.
The other options either support the hypothesis or are milder than option 3.
Options 2,4 and 5 in a way prove the hypothesis.
Option 1 has to be negated as it reduces the chances of damage, but does not provide an effective
refutation of the hypothesis.
Hence, the correct answer is option 3.
Discuss the solution with Testfunda users.

Solution #93: (14-Jul-09)


Let a denote the smallest angle of the polygon and d be the common difference and n be the
number of angles of the polygon.
The angles of the polygon will be of the form a, a + d, a + 2d, ... and so on.
The product of the 4th and the 5th term of the progression is 18200.
( a + 3d ) ( a + 4d ) = 18200 ...(i)
5th term when divided by the second term gives a quotient of 1 and remainder of 30
( a + 4d ) = ( a + d ) + 30 ...(ii)
Solving equations (i) and (ii) we get,
a = 100 and d = 10
Sum of the interior angles of a polygon = ( 2n 4 ) 90
n/2 [ 2a + ( n 1) d ] = ( 2n 4 ) 90 ...(iii)
n = 9 or 8
Number of sides of the polygon = Number of interior angles = 9 or 8
Number of diagonals that can be drawn in a polygon is given by n( n 3 )/2
Number of diagonals that can be drawn can be 27 or 20
Hence, option 4.
Discuss the solution with Testfunda users.

Book 2- Question of the Day

204

www.TestFunda.com

The LEADER in Online Test Prep

Solution #94: (15-Jul-09)


The passage mentions the correlation between peace and wealth creation- in times of peace
motivated people (businesses) have the environment for wealth creation. In order to strengthen this
argument we need to say that it is true, and provide some fact to prove it. The paragraph mentions
2000-2007 period as one in which conflicts reduced, option 1 states that during this time the global
GDP increased - the correlation is well established and the argument is strengthened.
Option 2 is irrelevant as it is about how defence spending is divided between personnel and,
probably, equipment.
Option 3 is also irrelevant because it talks about the projected increase in defence expenditurethere is no correlation except that economic productivity may decrease. Options 4 and 5 similarly do
not show the correlation between peace and wealth, but the relation between economic downturn
and defence spending.
Hence, the correct answer is option 1.
Discuss the solution with Testfunda users.

Solution #95: (16-Jul-09)


Let the total number of all animals be x.
One-fourth of all animals with fishes

One-fourth with insects

One-sixth with amphibians

One-twelfth with reptiles

One-eighth with birds

One-tenth with mammals

The rest is man.

Therefore man is 2.5% of all animals.


Hence, option 2.
Discuss the solution with Testfunda users.

Book 2- Question of the Day

205

www.TestFunda.com

The LEADER in Online Test Prep

Solution #96: (17-Jul-09)


How will climate change affect agriculture? This question is already answered in the paragraph. The
idea now to be brought to a logical conclusion in the paragraph is that of the Conference of Parties
of the UN Framework Convention on Climate Change. Option 3 closes this by urging the participants
global community is especially significant from this point of view to use the opportunity to
ensure food security. This addresses the issues of agriculture and the poorest of the poor
adequately to close the paragraph.
Hence, the correct answer is option 3.
Discuss the solution with Testfunda users.

Solution #97: (18-Jul-09)

We rationalize the denominator of RHS by multiplying numerator and denominator by,

Which gives us,

At t = 1 equality does not hold, therefore we can cancel (t 1) from numerator and denominator

Book 2- Question of the Day

206

www.TestFunda.com

The LEADER in Online Test Prep

Squaring both sides, we get,

, we have,

Hence, option 2.
Discuss the solution with Testfunda users.

Solution #98: (19-Jul-09)


Job losses slowed dramatically in May 2009 down from 504,000 in April to 345, 000 fewest since
September certain sectors are still reporting widespread job losses. This is in essence what the
paragraph is. Then how did the decline in number happen? We need to answer that issue to close
the paragraph. Option 4 explains how the decline stopped. All the other options will leave the
paragraph still open.
Hence, the correct answer is option 4.
Discuss the solution with Testfunda users.

Book 2- Question of the Day

207

www.TestFunda.com

The LEADER in Online Test Prep

Solution #99: (20-Jul-09)


For a quadratic equation ax2 + bx + c = 0,
Sum of the roots = b/a and
Product of the roots = c/a
Therefore, for the given quadratic equation,
the sum of the roots = (K + 5) and product of the roots = 3(2K + 1)
Now, 3(K + 5) = 3(2K + 1)
K + 5 = 2K + 1
K=4
Therefore, the second equation will become x4 4x3 + mx2 + nx + 1 = 0
Now, if we take the second option m = 6, n = 4, the equation will become x4 4x3 + 6x2 4x + 1 = 0
In this equation, the sign of the terms is changing 4 times, hence it will have 4 positive roots.
According to "Descartes' rule of signs", the number of positive roots of a polynomial with real
coefficients is equal to the number of "changes of sign" in the list of coefficients, or is less than this
number by a multiple of 2.
Hence option 2.
Discuss the solution with Testfunda users.

Solution #100: (21-Jul-09)


The correct answer is option 2. Option 4 too could be considered for the answer and is perhaps as
good from the point of view of the narrative. However, I have found a trigger; the second last
sentence of the paragraph raises questions: trigger for what? The answer is a trigger for the
symptoms of a disorder. Hence option 2 completes the paragraph more logically and completely
brought to a smooth closure. Also, on closer examination we can see that option 4 is a generalised
inference in other words what is already implied in the paragraph. There is no need to state it. It is
actually stating the obvious.
Hence, the correct answer is option 2.
Discuss the solution with Testfunda users.

Book 2- Question of the Day

208

www.TestFunda.com

The LEADER in Online Test Prep

Solution #101: (22-Jul-09)


As it is known that any contestant can enter and exit from any gate, the first contestant can enter
from any of the six gates. He latches that door from inside and after completing his task he moves
out from another door latching it from outside.
Hence at this position 2 doors are latched, one from inside and the other from outside. When the 2nd
contestant starts his task he enters from a door which was not used by the 1st contestant and latches
it from inside, completes his task and moves from some other door and then latches it from outside.
The 3rd contestant does the same.
Now at this stage all the six doors are latched, three from inside and three from outside. Hence the
next contestant can enter from any of the three doors latched from outside.
As the 4th contestant enters by opening a door latched from outside and leaves by opening a door
latched from inside, at this stage there are 4 latched doors: two latched from outside and other two
latched from inside
Now the 5th contestant enters who latches the doors he uses i.e. if he comes across a latched door
and he can open it he uses it and latches it from other side.
So when he starts his task there are two open doors, two doors latched from inside and two doors
latched from outside so there arise following cases:
He enters from a door latched from outside and leaves from a door latched from inside
He enters from a door latched from outside and leaves from a door which is not latched
He enters from a door which is not latched and leaves from a door which is not latched
He enters from a door which is not latched and leaves from a door latched from inside
Depending on the above cases the situation after contestant 5 has moved out can be tabulated as
follows

Now when the 6th contestant starts he can select any door at random. His probability of entering the
room depends on the doors used by the 5th contestant.
If 5th contestant follows case 1 then,
Book 2- Question of the Day

209

www.TestFunda.com

The LEADER in Online Test Prep

If 5th contestant follows case 2 then,

If 5th contestant follows case 3 then,

If 5th contestant follows case 4 then,

So the probability that the 6th contestant is able to enter through a randomly selected door is given
as follows

Hence, option 3.
Discuss the solution with Testfunda users.

Solution #102: (23-Jul-09)


Option 2 is the most neutral sentence available to close the paragraph. Elaborate and innovation
famine are well supported by the paragraph. It matches the tone as well.
Option 1 is rather judgmental about what is going on at Microsoft. The word is official is somewhat
out of place.
Similarly competition and expenditure are out of place too, eliminating 4 and 5.
Hence, the correct answer is option 2.
Discuss the solution with Testfunda users.
Book 2- Question of the Day

210

www.TestFunda.com

The LEADER in Online Test Prep

Solution #103: (24-Jul-09)

Put n = n + 1 in (i)

Put n = n + 1 in (ii)

Put n = n + 1 in (iii)

= f(n)
f(n + 5) = f(n), and the smallest value of p for which f(n + p) = f(n) is 5
Hence, option 5.
Discuss the solution with Testfunda users.

Book 2- Question of the Day

211

www.TestFunda.com

The LEADER in Online Test Prep

Solution #104: (25-Jul-09)


Options 1,3 and 5 are beyond the scope of the information provided, and are hence eliminated.
Two major themes are present in the passage:
1. there are only local issues being debated in state after state
2.

... it now appears there simultaneously exists a silent undercurrent, even a mini-wave, at
the national level...

These two themes are properly introduced in option 4. Option 2 puts the focus on 'electorate' which
may not necessarily be true in the context of issues being debated. hence, option 2 can be
eliminated.
Hence, the correct answer is option 4.
Discuss the solution with Testfunda users.

Solution #105: (26-Jul-09)


Case 1:
a=b=c=d=e=1
There is just one arrangement,
1 sequence.
Case 2:
We have four 1's, say a = b = c = d = 1 and e 1, then e = 2, 3, 4, 5, 6.
Now as all the variables are independent,
we have 5 5 = 25 sequences.
Case 3:
We have three 1's, then last two can be (2, 2), (2, 3), (3, 3), (2, 4) and (2, 5).

we get 20 sequences.

we get 3 20 = 60 sequences.
a total of 80 sequences.
Case 4:
We have two 1's, then the other three numbers must be 2.

Adding all the four cases we get 1 + 25 + 80 + 10 = 116 sequences.


Hence, option 4.
Discuss the solution with Testfunda users.

Book 2- Question of the Day

212

www.TestFunda.com

The LEADER in Online Test Prep

Solution #106: (27-Jul-09)


Sweetening the breath in option 1and paused to do her part in option 2 eliminate them. Option
1 make the issue sound frivolous, while option 2 contradicts the paragraph.
Option 3 talks of America, whereas the rest of the passage is at a global level.
Option 4 is too abrupt a beginning.
As cows belch methane, and methane contributes to global warming, climate scientists want them
to belch less. Option 5 is an apt start.
Hence, the correct answer is option 5.
Discuss the solution with Testfunda users.

Solution #107: (28-Jul-09)


The person has to reach city L latest by 1:30 am on 25th June and he starts from A at 5 am on 24th
June.
He has maximum 20.5 hours to reach L from A.
We first need to find routes which require less than 20.5 hours to reach L from A.
Let us first consider the direct route A C F H K L. (i)
We get that this route requires 20 hours (excluding the breaks for refuelling).
Indirect routes including the cities used in the above route should not be considered as the time
required to travel through any of those routes will be more than the time required for the above
route i.e. 20 hours and hence will not be valid.
For example:
ABCFHKL
ABCFEGHKL
ABCFJHKL
ABCDEFHKL
ABCDEFJHKL
ACFEGHKL
ACFJHKL
ACDEFHKL
ACDEFJHKL
Now lets consider another direct route i.e. A B D E G K L (ii)
This route requires 19 hours (excluding the breaks for refuelling).
Indirect routes which can be neglected are:
ABCDEGKL
ABCDEGHKL
ABDEGHKL
Book 2- Question of the Day

213

www.TestFunda.com

The LEADER in Online Test Prep

As the time required in these routes will be greater than 20.5 hours.
As the time required to travel from A to D via B is 6 hours and the time required via C is 7 hours.
Route A C D E G K L should also be considered. (iii)
This route requires 20 hours (excluding the breaks for refuelling).
Indirect route which can be neglected is A C D E G H K L
Now let us consider another direct route A C I J H K L
This route requires 23 hours and hence can be neglected.
Indirect route which also can be neglected is A B C I J H K L
There are few more indirect routes which require more than 20.5 hours and hence can be neglected:
ABCFEGKL
ABDEFHKL
ABDEFJHKL
There are only three routes between A and L which require time less than 20.5 hours.
From (i), (ii) and (iii) we get:
ACFHKL
ABDEGKL
ACDEGKL
Now we know that his car already has 30 litres of fuel. Hence, he can start travelling without
incurring any fuel cost. His first fuel cost will be incurred whenever he stops for fuelling.
It also requires 4 litres of fuel every hour.
He can travel maximum 7.5 hours at a stretch, post which he will have to take a break of 15 mins
for refuelling which can be done only at cities.
Now to travel through this route 1 i.e. A C F H K L, he will have to take a break for refuelling
at C as he will not be able to drive his car continuously for 8 hours from A to F.
Similarly he will have to take breaks at F, H and K also.
Each break takes 15 minutes.
He will not be able to reach L from A in 20.5 hours.
This route can also be neglected.
Now we will calculate the costs required to travel through the remaining two routes.
Book 2- Question of the Day

214

www.TestFunda.com

The LEADER in Online Test Prep

For route A B D E G K L
He will take two breaks at city D and city G for refuelling.
The total time required would be 19.5 hours.
In this route he travels through 4 State highways and 2 Expressways.
Toll required on this route is Rs. 440 (Rs. 50 on each State highway and 120 on each Expressway)
He has to fill up the entire tank. Since he has used up 24 litres of fuel between A and D, he still has 6
litres left in the tank. Hence, he needs to fill up only 24 litres.
Similarly, since he has used up 28 litres of fuel between D and G, he still has 2 litres left in the tank.
Hence, he needs to fill up only 28 litres.
He will have to refuel 24 litres at city D and 28 litres at city G.
Cost of refuelling will be Rs. 2600
Total money spent will be Rs. 3040.

For route A C D E G K L
He will take two breaks at city D and city G for refuelling.
The total time required would be 20.5 hours.
In this route he travels through 5 State highways and 1 Expressway.
Toll required on this route is Rs. 370 (Rs. 50 on each State highway and 120 on each Expressway)
He will have to refuel 28 litres at city D and 28 litres at city G.
Cost of refuelling will be Rs. 2800
Total money spent will be Rs. 3170.
The cheapest route possible to City L from A is A B D E G K L
Hence, option 3.
Discuss the solution with Testfunda users.

Book 2- Question of the Day

215

www.TestFunda.com

The LEADER in Online Test Prep

Solution #108: (29-Jul-09)


To get the first sentence right we need to think of how the writer is going to lead his reader through
the information he wants to present. Option 4 states the problem and then the paragraph explains
it. Option 1 and 5 begin with the solution it is one of the ways to start a paragraph. But here the
writer is definitely not writing to advocate that solution, but to highlight the problem. Hence option
4 scores over 1 and 5.
Option 3 one of the challenges is also not the purpose of the paragraph. The inference may be
true but the purpose of the paragraph is not to highlight this challenge.
Option 2 states half the problem which is vague as the paragraph does not state the other half (if
at all there).
Hence, the correct answer is option 4.
Discuss the solution with Testfunda users.

Solution #109: (30-Jul-09)


Let the integers be a, ar and ar2, then a + ar + ar2 = 111
a(1 + r + r2) = 111
We note that (1 + r + r2) > 0 , hence a > 0
a(1 + r + r2) = (1 111), or (3 37), or (37 3), or (111 1)
So, we have the following four cases:
Case 1: When a = 1
1 + r + r2 = 111
r2 + r 110 = 0
(r + 11)(r 10) = 0
r = 10 or 11
So, we get two sets of solutions (1, 10, 100) and (1, 11, 121).
Case 2: When a = 3
1 + r + r2 = 37
r2 + r 36 = 0
This has no integral solutions.
Case 3: When a = 37
1 + r + r2 = 3
Book 2- Question of the Day

216

www.TestFunda.com

The LEADER in Online Test Prep

r2 + r 2 = 0
(r 1)(r + 2) = 0
r = 1 or 2
So, we get two sets of solutions (37, 37, 37) and (37, 74, 148). The first set is to be rejected as the
integers are not distinct.
Case 4: When a = 111
1 + r + r2 = 1
r2 + r = 0
r(r + 2) = 0
r = 0 or 1
So, we get two set of solutions (111, 0, 0) and (111, 111, 111). Both are to be rejected as the
integers are not distinct.
From the above four cases, we have 3 distinct valid sets.
Hence, option 2.
Discuss the solution with Testfunda users.

Solution #110: (31-Jul-09)


The two things mentioned in the paragraph are:
1) concern for the well-being of others, (even unrelated others), and,
2) of finely crafted material objects both useful and ornamental.
These two most closely correspond to morality and culture. Option 5 may come close but the
writer is not talking about love when he states concern; art is different from objects both useful and
ornamental.
Hence, the correct answer is option 1.
Discuss the solution with Testfunda users.

Book 2- Question of the Day

217

www.TestFunda.com

The LEADER in Online Test Prep

Solution #111: (01-Aug-09)


Taking the reciprocal of the first relation we get

Or

Taking reciprocals again, we get

xn = (2n 1)x1
Using the above in x1 + x2 + x3 + + x1005 = 2010
we get x1 + 3x1 + 5x1 + + 2009x1 = 2010
Or
10052x1=2010

Hence, option 2.
Discuss the solution with Testfunda users.

Solution #112: (02-Aug-09)


Tomatoes are beneficial to health because they contain lycopene. Based on this assumption
scientists have engineered a variety that contains more lycopene. The assumption is that when one
consumes lycopene it is beneficial to health. Hence if a tomato contains more lycopene it is better.
Option 2 is not assumed by the scientists - their efforts are to make lycopene easily available.
Option 3 is also not assumed because we cannot say (there is no data) that the scientists chose
tomatoes for this reason. For the same reason options 4 and 5 are also eliminated.
Hence, the correct answer is option 1.
Discuss the solution with Testfunda users.

Book 2- Question of the Day

218

www.TestFunda.com

The LEADER in Online Test Prep

Solution #113: (03-Aug-09)


Let abcd be a peculiar number, then
10a + b + 10c + d = 10b + c
10a + d = 9(b c)
RHS is a multiple of 9, hence LHS has to be a multiple of 9.
To find the peculiar number just before 1978, it is easy to see a = 1 and d = 8 and therefore b c = 2.
To keep the number as close to 1978, b = 8 and hence c = 6. So the number just before 1978 is 1868.
To find the peculiar number just after 1978, we need to note that 1978 is the biggest peculiar
number less than 2000.
Hence a = 2, which makes d = 7 ( 27 is a multiple of 9).
This gives b c = 3, which gives b = 3 and c = 0. Hence the number is 2307.
The sum of two peculiar number is 2307 + 1868 = 4175
Hence, option 4.
Discuss the solution with Testfunda users.

Solution #114: (04-Aug-09)


The question has to actually answer the question, ....but where did these organic molecules come
from? The answer as suggested in the paragraph is that they came from outside (either from a
comet or an asteroid). In that case life must have existed elsewhere in the universe. Therefore, the
correct answer option is option 1.
Option 2 merely repeats the idea that has to be strengthened.
Option 3, 4 and 5 mention inorganic whereas we need to prove the organic aspect.
Hence, the correct answer is option 1.
Discuss the solution with Testfunda users.

Book 2- Question of the Day

219

www.TestFunda.com

The LEADER in Online Test Prep

Solution #115: (05-Aug-09)


It is clear that the digit 1 is the digit that will first get written 119 times, since it will occur in the
hundreds place for all numbers greater than 100 and less than 200.
To find the number at which Ram stops, we need to count the number of 1s that are used up to that
number.
Assume that the number is greater than 100.
Then, the total number of times the digit 1 is used in writing the numbers from 1 to 99 is 20 (it is
used 10 times in the units place and 10 times in the tens place).
It is used 11 times in writing the numbers from 100 to 109 (ten times in the hundreds place and once
in the units place).
Similarly, it is used 21 times in writing the numbers from 110 to 119, and 11 times in the subsequent
intervals of ten.
Counting in this fashion, we find that the 119th occurrence of 1 occurs when he has written 180.
The total of the numbers up to this point is = 1 + 2 + 3 + + 180 = (180 x 181) / 2 = 90 x 181 = 16290.
Hence, option 2.
Discuss the solution with Testfunda users.

Solution #116: (06-Aug-09)


Options 1 is incorrect because of competitive markets whereas the fact is that perfect
competition eliminates economic rent.
Option 2 is not correct because it states In economics rent is ... both the meanings of rent are
from economics.
Option 4 is vague in ordinary definition. Secondly, it does not explain what rents and economic
rents are.
Option 5 incorrectly states in economics rent is .. whereas both the meanings are from economics.
Economic rent is acceptable as a different term with different meaning, but we cannot say rent is
one thing and in economics rent is another thing.
Hence, the correct answer is option 3.
Discuss the solution with Testfunda users.

Book 2- Question of the Day

220

www.TestFunda.com

The LEADER in Online Test Prep

Solution #117: (07-Aug-09)


Using statement A alone:
(1, 98), (2, 97) (49, 50) and (99), we can pick only one number from each set and 99.
So we have different sets, which lead to many different sums, hence insufficient data.
Statement A alone is not sufficient.
Using statement B alone:
(1, 99), (2, 98), (3, 96) ... (49, 51) and (50), we can pick only one number from each set and 50.
So we have different sets, which lead to many different sums, hence insufficient data.
Statement B alone is not sufficient.
Using statements A and B together:
No two numbers sum to 99 or 100, then if we arrange the set as 99, 1, 98, 2, 97, , 51, 49, 50, then
any two adjacent terms add up to 99 or 100, so we can pick only one out of any two adjacent terms.
We further group the set into pairs, and then we have 49 pairs and the number 50. As we have to
choose 50 terms, only one number from each pair can be chosen.
Hence, 50 must be definitely chosen, which causes 49 to be rejected. Hence, 51 has to be chosen
which results in 48 to be rejected and 52 to be chosen and so on.
Hence the chosen set of numbers is 50, 51, 52, , 97, 98, 99. Hence we can find the sum by
subtracting the sum of the first 49 natural numbers from the sum of the first 99 natural numbers.
Statement A and B together are sufficient to answer the question.
Hence, option 4.
Discuss the solution with Testfunda users.

Solution #118: (08-Aug-09)


The options are word play with similar and dissimilar and convergence and divergence. A closer
examination will help you understand the differences.
Option 1 mentions structure and biology which are in the context of the paragraph the same thingbehaviour and biology are required.Secondly, its definition of divergence is incorrect.
Option 2, the definition of divergence is incorrect. It should be closely related animals and not
unrelated animals.
In option 3 the definition of convergence is incorrect. It should be similar responses to similar
environmental conditions.
In option 4, the definition of convergence is incorrect. It should read similar environmental
conditions and not dissimilar environmental conditions.
Option 5 correctly captures the essence of the text.
Hence, the correct answer is option 5.
Discuss the solution with Testfunda users.

Book 2- Question of the Day

221

www.TestFunda.com

The LEADER in Online Test Prep

Solution #119: (09-Aug-09)


Using f(A, B) = f(A 1, f(A, B 1)) on f(1, 1500),
we get f(1, 1500) = f(0, f(1, 1499))
= f(1, 1499) + 1 (using f(0, k) = k + 1)
= f(0, f(1, 1498) + 1
= f(1, 1498) + 2
= f(1, 0) + 1500
= f(0, 1) + 1500
= 1 + 1 + 1500
= 1502
f(1, 1500) = 1502
Hence, option 5.
Discuss the solution with Testfunda users.

Solution #120: (10-Aug-09)


Option 1 is incorrect because it makes an assumption that only animal spirits guided an
entrepreneur.
Option 2 is incorrect because animal spirits do not make an entrepreneur unmindful of the risks.
Option 3 is incorrect because animal spirits do not help him put aside the loss.
Option 5 is incorrect because animal spirits do not help him postpone the thoughts.
Option 4 captures the essence.
Hence, the correct answer is option 4.
Discuss the solution with Testfunda users.

Book 2- Question of the Day

222

www.TestFunda.com

The LEADER in Online Test Prep

Solution #121: (11-Aug-09)

Adding all these, we see that the first part of each term cancels with the last part of the previous
term, leaving only the overall first and last terms.

Hence, option 3.
Discuss the solution with Testfunda users.

Solution #122: (12-Aug-09)


Option 1 distorts the ideas in the passage in cults including Sikhism Sikhism is not a cult but a
religion.
Option 2 states combined the influences of Ramananda and Sufi masters.. which is again a
distortion the passage states the best tenets of both.. implying Hinduism and Islam rather than of
the specific masters. Also in option 2, cults which became the forerunner of Sikhism is also a
distortion.
Option 3 erroneously states that Kabir founded Sikhism; also the influences mentioned are too
specific.
Option 5 is inadequate in highlighting the main points.
Option 4 can be chosen as the best because it states the main ideas without distorting the facts.
Hence, the correct answer is option 4.
Discuss the solution with Testfunda users.

Book 2- Question of the Day

223

www.TestFunda.com

The LEADER in Online Test Prep

Solution #123: (13-Aug-09)


Let us denote such a number by XY, where X is the digit in the tens place and Y is the digit in the
units place.
The number formed by reversing the digits is denoted by YX.
The sum of these numbers = 10X + Y + 10Y + X = 11(X + Y)
This must be equal to n2, for some integer n. If 11(X + Y) = n2, we must have both n and (X + Y)
divisible by 11.
However, since X and Y are both digits, the maximum possible sum they can have is 18, and
therefore (X + Y) = 11
The possible numbers with this property are 29, 38, 47, 56, 65, 74, 83 and 92.
There are 8 such numbers.
Hence, option 4.
Discuss the solution with Testfunda users.

Solution #124: (14-Aug-09)


The paragraph can be summarised thus: The interest in Existentialism (as it spread in Europe) is
diverse: religious, metaphysical, moral and political. Existentialism is able to accommodate all this
because (as a philosophy) it draws from various sources. Among those sources are: subjectivism of
St. Augustine, Dionysian Romanticism of Nietzsche, and the nihilism of Dostoyevsky. Option 1 best
captures this essence.
Option 2 mentions focus on several aspects of existence- somehting not mentioned in the
paragraph.
Option 4 which comes close attributes the interest entirely to the three- option 1 mentions it as in
parts hence is an accurate reflection of the paragraph.
Options 3 and 5 do not mention the sources of Subjectivism, Romanticism and Nihlism in any detail,
viz St. Augustine, Nietzsche and Dostoyevsky.
Hence, the correct answer is option 1.
Discuss the solution with Testfunda users.

Book 2- Question of the Day

224

www.TestFunda.com

The LEADER in Online Test Prep

Solution #125: (15-Aug-09)


If each inlet tap can fill the tank in 24 seconds, eight of them together can fill it in 24/8 = 3 seconds
Similarly, the four outlet taps together can empty the tank in 35/4 seconds.
When all pipes are open, then in one second, (1/3) (4/35) = (35 12)/105 = 23/105 of the tank is
filled up.
Therefore, in 4 seconds, 92/105 of the tank is filled up, which is approximately 87.5 %.
Hence, option 1.
Discuss the solution with Testfunda users.

Solution #126: (16-Aug-09)


Z particle and W particle are introduced in statement A. Their characteristics are continued in E. AE is
the best logical positioning. Therefore, options 3,4 and 5 are eliminated.
Statement C, while continuing the W and Z theme introduces the mediation of weak force. The
intermediary in D links to statement C. D also mentions when the concept arose. Therefore, option
2 is eliminated.
Statement B then states what happened decades after that and later.
This sequence makes option 1, ECDB the best choice.
Hence, the correct answer is option 1.
Discuss the solution with Testfunda users.

Solution #127: (17-Aug-09)


Of the four numbers, three are even and one is odd, hence the sum cannot be even. Hence, the
number is not divisible by 2.
234 = (24 - 1)4 which will give remainder (-1)4 = 1 on division by 3. 18 and 72 are each divisible by 3.
21162 = (2115 + 1)2 which will give a remainder 12 = 1 on division by 3.
Hence, a total of 1 + 1 = 2 as remainder on division by 3. Hence, the number is not divisible by 3.
234 = (20 + 3)4 which gives a remainder 34 = 81 = 1 on division by 5.
The last digit of 182, 722 and 21162 are 4, 4, and 6 respectively, hence they give remainders of 4, 4,
and 1 on division by 5.
Hence, total remainder is 1 + 1 + 4 + 4 = 10, which is divisible by 5.
Hence, the given number is divisible by 5.
Hence, option 3.
Discuss the solution with Testfunda users.

Book 2- Question of the Day

225

www.TestFunda.com

The LEADER in Online Test Prep

Solution #128: (18-Aug-09)


The one obvious area of excess in statement E is in the background of the otherwise rather
prosperous decade of the 1920s described in statement C. Also note that C cannot be placed
anywhere else in the sequence but immediately after A. Therefore, options 1,2 and 4 are eliminated.
Statements E and D seamlessly go together in that order with the theme of housing. Therefore,
option 5 is eliminated.
As a result, the beginning of statement B helps conclude the discussion explaining the depression
of 1929.
Hence, the correct answer is option 3.
Discuss the solution with Testfunda users.

Solution #129: (19-Aug-09)


a2 + 2b2 = 2009b = 41 49b
As RHS is divisible by 49, so should be the LHS.
Since the LHS is divisible by 49, it leaves a remainder 0 when divided by 7.
This is possible only if both the parts of the LHS individually leave a remainder 0 when divided by 7
or the sum of their remainders is 7.
Whenever a square is divided by 7, it can only leave 0, 1, 2, or 4 as a remainder.
Hence, the sum of their remainders can never be 7.
Hence, both the remainders have to be 0.
Hence both a and b are divisible by 7.
Let a = 7x and b = 7y, then the equation becomes x2 + 2y2 = 287y
We can still note that RHS is divisible by 7 and so is the LHS.
Hence x = 7m and y = 7n, we get m2 + 2n2 = 41n
m2 = n(41 - 2n), now as m is positive integer, we get n < 21.
We do trial and error for n = 1 to n = 20 and check for what value of n we get RHS as perfect square.
We get n = 16 as the only solution, which gives m = 12
a = 49m = 49 12 and b = 49n = 49 16
a + b = 49 (12+16) = 49 28 = 1372
Hence option 1.
Discuss the solution with Testfunda users.

Book 2- Question of the Day

226

www.TestFunda.com

The LEADER in Online Test Prep

Solution #130: (20-Aug-09)


The clue is the last line of the paragraph. In spite of much speculation, there appears to be hope
now. This would hold true if some new information is now made available. In this context, option 3
fits perfectly.
Option 1 is irrelevant to the paragraph.
Options 2 and 4 introduce new elements such as whims of the Lord and the internet which cannot
be deduced from the paragraph.
Option 5 is too strongly worded to be the correct answer option. The penultimate statement of the
paragraph signalls hopes of demystifying Nostradamus' texts. Therefore to be unravel all of his
mysteries is too strong a statement and does not link well with the penultimate statement.
Option 3 ends the paragraph well by giving the reason for the hope mentioned in the penultimate
statement of the paragraph.
Hence, the correct answer is option 3.
Discuss the solution with Testfunda users.

Solution #131: (21-Aug-09)


We know that Akshay tells the truth only on a single day of the week. If the statement on day 1 is
untrue, it means that he tells the truth on either Monday or Tuesday. Now, if the statement on day 3
is untrue as well, it means that he tells the truth on either Wednesday or Friday. Since Akshay tells
the truth on only one day, both the statements cannot be simultaneously untrue. So, exactly one of
these statements must be true and hence the statement on day 2 is definitely untrue.
Now, we consider two different cases
Case (i):
Statement on day 1 is true.
Hence, the statement on day 3 is untrue. From this, it follows that Akshay tells the truth on either
Wednesday or Friday. So, day 1 is either Wednesday or Friday. So, day 2 has to be a Thursday or
Saturday. But that would imply that the statement on day 2 is true, which contradicts with our
conclusion. Hence, this case is not possible.
Case (ii):
Statement on day 3 is true.
Hence, the statement on day 1 is untrue. From this, it follows that Akshay tells the truth on
either Monday or Tuesday. So, day 3 is either Monday or Tuesday. So, day 2 has to be a Sunday or
Monday. If it Monday, then it consistent with the above made conclusion that the statement made
on day 2 is untrue. So day 2 is a Monday and day 3 is a Tuesday. Therefore, the day on which Akshay
tells the truth is Tuesday.
Hence, option 2.
Discuss the solution with Testfunda users.

Book 2- Question of the Day

227

www.TestFunda.com

The LEADER in Online Test Prep

Solution #132: (22-Aug-09)


Statement 3 is a conclusion as well as an assumption. The comparison with the other destinations
supports world class
The author mentions the merits of India, comparing certain tourist destinations. Therefore, option 1no better place- is not an assumption.
The author also does not assume there is no reason- he states there are more reasons to stay- that
does not imply that there is no reason to travel to other destinations. Therefore, option 2 can be
eliminated.
Option 4 may be inferred as a remote possibility but not as an assumption of the author.
Option 5 is in no way intended or assumed.
Hence, the correct answer is option 3.
Discuss the solution with Testfunda users.

Solution #133: (23-Aug-09)


There are 64 cubes forming the larger cube and 64 = 4 4 4
So, there are 8 smaller cubes such that none of their face is visible. The remaining 56 cubes are to be
arranged in a manner so as to minimize the required total. These 56 cubes can be divided into three
parts depending on the number of faces visible outside.
i). Cubes with three faces visible
This will be the case for all the 8 corners of the cube. So the smaller cubes numbered 1 to 8 should
be arranged to form the corners. The total of the numbers on all the faces of these cubes is
(1+ 2 + 3+ 8) 3 = 108
ii). Cubes with two faces visible
This will be the case for all the cubes which form the edges of the larger cube except the
corners. There are 24 such cubes and so, they should be numbered 9 to 32. The total of the numbers
on all the faces of these cubes is
(9 + 10 + 11 + 32) 2 = 984
iii). Cubes with one face visible
This will be the case for all the remaining cubes. Therefore, the total number of cubes with only one
face visible is (56 8 24) = 24. So these cubes should be numbered 33 to 56. The total of the
numbers on all the faces of these cubes is (33 + 34 + 35 + 56) = 1068
Therefore, the sum of all these numbers is 108 + 984 + 1068 = 2160.
This is the minimum sum of all the numbers that appear on the faces of the larger cube.
Hence, option 1.
Discuss the solution with Testfunda users.

Book 2- Question of the Day

228

www.TestFunda.com

The LEADER in Online Test Prep

Solution #134: (24-Aug-09)


The gist of the paragraph is: In law a defendant can stop (terminate) the proceedings against him on
certain grounds. This called abatement. The plaintiff can re-initiate it. In equity (natural justice)
through abatement the proceedings are suspended until the defect is cured. Option 3 captures this
gist.
Option 1 is incorrect because the factual distortion with until. Plaintiff is free to re-initiate the
proceedings- abatement is not until such re-initiating takes place.
Option 2 describes Abatement in equity rather than abatement in law.
Option 4 distorts the meaning of the paragraph since terminations are initiated by the defendant
and not the plaintiff.
Option 5 does not mention abatement in law in particular.
Hence, the correct answer is option 3.
Discuss the solution with Testfunda users.

Solution #135: (25-Aug-09)


Bini did not get a composite number in any of her throws.
Therefore, the outcome of the die in each throw is one out of 1, 2, 3 or 5.
If in any throw the outcome is 1, 3 or 5, the letters Bini can select are odd numbered letters of the
alphabet.
Hence, in such a case, Bini can select letters out of A, C, E, G, I, K, M, O, Q, S, U, W and Y.
Therefore, there are 5 vowels and 8 consonants that Bini could use to make words.
The total number of ways she could make 3-letter words using 3 different letters from these 13
letters is 13P3.
But since a word should contain atleast one vowel, all the three letters of a word cannot be
consonants.
The total number of 3-letter words that can be formed from these letters such that they contain only
consonants is 8P3
Therefore, the total number of 3-letter words that Bini could have made such that all the 3 letters
are different and each word contains atleast one vowel is

= 13 12 11 8 7 6
= 1716 336
= 1380
Book 2- Question of the Day

229

www.TestFunda.com

The LEADER in Online Test Prep

Now, if in any throw the outcome is 2, the letters Bini can select are the even numbered letters of
the alphabet i.e. B, D, F, H, J, L, N, P, R, T, V, X and Z.
Since, there are no vowels among these letters, no word containing atleast one vowel can be made.
After one throw, Bini creates all the words possible and only then throws the die for the second
time.
Now, in each throw, the outcome being one among 1, 2, 3 or 5, the total number
of possible outcomes are 4 4 = 16
We divide these possibilities in two cases
Case (i):
The outcome is 2 in each throw.
In this case the total number of words of the required kind that Bini can make is 0.
Case (ii):
In all the other 15 possibilities except the one mentioned in case (i), the number of possible
words would be 1380.
This is because if the outcome is 2 in of the throws and 1, 3 or 5 in the other throw, then the total
number of words that Bini can make is
0 + 1380 = 1380
Even if in both the throws, the outcome obtained is one out of 1, 3 or 5, the words made both the
times would be the same and so the total number of words of the required kind will be 1380 only.
So, the answer would differ in both the cases depending on the outcome.
Since we do not know the exact outcome of the two throws, we cannot find the number of words
that Bini made. Note that the question does not ask for the number of words that she could have
possibly made.
Hence, option 5.
Discuss the solution with Testfunda users.

Solution #136: (26-Aug-09)


The gist of the paragraph is: Carbon emissions from deforestation are an environmental externalityits economics is not accounted for. Landowners have no incentives NOT to cut down trees. To
resolve this, voluntary carbon credits (for avoided deforestation) is suggested. There is already a
market for this.
Option 5 captures this essence.
Option 1 is too generic. The paragraph is about deforestation in particular and not the entire gamut
of environmental economics. Further we cannot ascertain from the paragraph whether voluntary
credits are effective.
Option 2 is categorical in compensates and resolves- the suggestive element is missing. (In the
correct option could be accommodates the suggestion part).
Option 3 is eliminated because of must be.
Option 4 is too categorical in stating that voluntary carbon credits compensate landowners and
decreases carbon emissions. The paragraph is suggestive on this issue.
Hence, the correct answer is option 5.
Discuss the solution with Testfunda users.

Book 2- Question of the Day

230

www.TestFunda.com

The LEADER in Online Test Prep

Solution #137: (27-Aug-09)


Let S be the sum of digits of N.
Then by the given condition,
N + S = 10000000
N = 10000000 S

(i)

The largest possible value of N is 9999999.


The largest possible value of S is 63.
S 63
The minimum value of N is 9999937.
9999937 N 9999999
The first 5 digits of N are 99999.
Let last two of N be x and y, respectively.
N = 99999xy
= 9999900 + 10x + y
S = 45 + x + y
As the first five digits of the number are divisible by 9, therefore, the remainder that N gives on
division by 9 will be same as the remainder given by x + y on division by 9.
Substituting the values of S and N in (i).
9999900 + 10x + y = 10000000 (45 + x + y)
11x + 2y = 55
The possible values of (x, y) are (5, 0), (3, 11) and (1, 22).
As x and y are the digits of the number N, therefore, x = 5 and y = 0.
N = 9999950 and x + y = 5
The remainder when x + y is divided by 9 is 5.
The remainder when N is divided by 9 is 5.
Hence, option 2.
Discuss the solution with Testfunda users.
Book 2- Question of the Day

231

www.TestFunda.com

The LEADER in Online Test Prep

Solution #138: (28-Aug-09)


Option 1 is ridiculous in animal personalities are not recognised ( as if there are animal celebrities).
Option 2 is ridiculous in Animals have personalities like humans- they do not have personality like
humans but they have personality traits the way humans have personality traits. The option states
animals are like humans.
Options 4 and 5 are equally ridiculous for the incorrect comparison made between people and
animals.Dogs do not have personalities or personality traits like humans but rather they have
personality traits the same way as humans have personality traits.
Option 3 captures the essence of the paragraph perfectly and succintly.
Hence, the correct answer is option 3.
Discuss the solution with Testfunda users.

Solution #139: (29-Aug-09)


Let S be the sum of the first 46 prime numbers.
S = 2 + sum of 45 prime numbers greater than 2
All prime numbers greater than two are odd.
We know that the sum of even number of odd numbers is even and the sum of odd number of odd
numbers is odd.
The sum of the 45 odd (prime numbers greater than 2) numbers is odd.
The sum of the 45 prime numbers greater than 2 is odd.
Sum of an odd and an even number is always an odd number.
S will be an odd number.
We can eliminate options 1, 3 and 4.
Also, we know that there are 25 prime numbers less than 100.
The sum of first 25 prime numbers = 2 + 3 + 5 + 7 + 11 + 13 + 17 + 19 + 23 + 29 + 31 + 37 + 41 + 43
+ 47 + 53 + 59 + 61 + 67 + 71 + 73 + 79 + 83 + 89 + 97 = 1060
From the 26th to the 46th prime number each number is greater than 100.
The sum of numbers from the 26th prime number to the 46th prime number will be greater than
2100.
The sum of first 46 prime numbers will be greater than 3160.
We can eliminate option 2.
Hence, option 5.
Discuss the solution with Testfunda users.
Book 2- Question of the Day

232

www.TestFunda.com

The LEADER in Online Test Prep

Solution #140: (30-Aug-09)


The correct usage in statement 1 is equivalent which means amounting to. Equal to is literal as
explained in a mathematical equation two plus two is equal to four. Therefore, options 1 and 2
are eliminated.
In statement 2, rather.than is the correct grammatical usage.Therefore, option 4 is eliminated.
In statement 3, older than is correct usage. Alternatively it is elder to.
In statement 4, Veracious means truthful, while voracious means ravenous. Veracious fits in
here in the context of the statement. A statement cannot possibly be voracious.
To elicit means to get information and is the apt word here; illicit means illegal. Therefore,
option 3 is eliminated.
Hence, the correct answer is option 5.
Discuss the solution with Testfunda users.

Solution #141: (31-Aug-09)


If logb a1 > logb a2 then
I.

If b > 1 then a1 > a2

(i)

II.

If 0 < b < 1 then a1 < a2

(ii)

Also, if (x a)(x b) 0 then x a or x b when a < b

(iii)

And, if (x a)(x b) 0 then a x b when a < b

(iv)

Consider the given inequality,


log x 4 (2x2 14x + 21) log x 4 (x2 16)

Case 1: (x 4) > 1
If (x 4) > 1 then we can say that x (5, ).
Then by (i) we get that,
2x2 14x + 21 x2 16
x2 14x + 37 0

Book 2- Question of the Day

233

www.TestFunda.com

The LEADER in Online Test Prep

Case 2: 0 < (x 4) < 1


If 0 < (x 4) < 1 then x (4, 5).
Then by (ii) we get that,
2x2 14x + 21 x2 16
x2 14x + 37 0

4<x<5
But 2x2 14x + 21< 0 for some x (4, 5).
We have to find x such that 2x2 14x + 21 > 0 and x (4, 5).
2x2 14x + 21 > 0

Hence, option 4.
Discuss the solution with Testfunda users.

Book 2- Question of the Day

234

www.TestFunda.com

The LEADER in Online Test Prep

Solution #142: (01-Sep-09)


One of the meanings of cast as a verb is to make suitable or accordant; tailor. Caste is a noun
which means a hereditary social group in Hinduism and is inappropriate in this context. Therefore,
options 2 and 4 are eliminated.
Claque (n) means a group of persons hired to applaud an act or performer and is the appropriate
word here. Clack (n) means chatter.
Faun (n) means an imaginary creature, half man and half goat and is the apt word here. Fawn (v)
means to show affection; as a noun fawn is a young deer. Therefore, we can eliminate option 1.
Greave (n) means a piece of armour for the shin. Grieve (v) means to feel sorrow and is not the
appropriate word here.
One of the meanings of flare (v) is to expand or open outward in shape. Flair (n) generally refers
to skill or tendency. Flare is appropriate here. Therefore, we can eliminate option 5.
Hence, the correct answer is option 3.
Discuss the solution with Testfunda users.

Solution #143: (02-Sep-09)


From statement A alone.
Let the integers be in AP with the first term a and common difference d.
nth term of the AP = Tn = a + (n 1)d

The value of the common difference, d, is known but the value of the first term, a, is not known.
We cannot find the average of the first n 1 integers on the basis of this statement alone.
Statement alone A is insufficient.
From statement B alone.
A is the coefficient of the second highest power of x in the given equation.
A is the sum the n roots of the given equation
A is the sum the n integers of the given set.
We need to know the nth integer of the set, to find the average of the first n 1 integers of the set.
Statement B alone is insufficient.
Combining statement A and B.
We find that we know both the sum and the common difference of n integers in AP.
We can calculate first term a.
We can calculate the n integers of the set.
But neither its is given that the set is arranged in ascending or descending order nor do we know the
nth element of the set.
We cannot find the average of first n 1 integers of the set using statements A and B both.
Hence, option 5.
Discuss the solution with Testfunda users.

Book 2- Question of the Day

235

www.TestFunda.com

The LEADER in Online Test Prep

Solution #144 (03-Sep-09)


Obtuse implies such bluntness as makes one insensitive in perception or imagination and is the
correct word here. Abstruse is difficult to understand. Therefore, options 2 and 4 are eliminated.
Precipitous means steep or perpendicular. Precipitate implies reckless.
Mucus is the noun- the secretion and is the correct word in this context; mucous is adjective as
in mucous membrane. Therefore, option 1 is eliminated.
Tortuous among other things means intricate or circuitous. Tortulous means swelled like a
knotted cord. Tortuous is the correct word contextually.
Disinterested means impartial or unbiased; uninterested implies not interested- lacking in
concern and is the correct word here. Therefore, option 3 is eliminated.
Hence, the correct answer is option 5.
Discuss the solution with Testfunda users.

Solution #145: (04-Sep-09)


From statement A alone.
We know a sequence of 1000 consecutive positive integers that has no prime number.
Now if we reduce the every number of the sequence by 1.
We might increase the number of primes in the given sequence by 1 or decrease the number of
primes by 1 or the number of primes in the given sequence will remain same.
This process can change the number of prime by 1.
If this process is continued till the first 1000 positive integers then there will be at least 50 primes in
this sequence (1-100 has 25).
We must have encountered one sequence of 1000 numbers having exactly 5 prime numbers in this
process.
Statement A alone is sufficient.
From statement B alone.
If we decrease the first term by 1and go to the first 1000 positive integers, we are not sure if the
number of primes are decreasing or increasing at any point of time so cant say anything conclusive.
If we add 1 to each number of the sequence even then we cant say whether primes will decrease or
increase.
Statement B alone is insufficient.
Hence, option 1.
Discuss the solution with Testfunda users.

Book 2- Question of the Day

236

www.TestFunda.com

The LEADER in Online Test Prep

Solution #146: (05-Sep-09)


Complaisant means obliging and is the appropriate word here; complacent means
unconcerned, self-satisfied. Therefore, options 1 and 4 are eliminated.
Devisor means a testator; one, who bequeaths (through a will) ones real estate. Deviser is a
planner or contriver. Therefore, option 5 is eliminated.
Auger is a tool and is the appropriate word here; augur means to foretell. Therefore, option 2
is eliminated.
Climatic is related to the climate; climactic is related to the climax and is the appropriate
word.
A caret is (^) mark made to show the place where something is to be inserted; carat refers to
purity/weight of gold diamonds etc. Caret is the appropriate word here.
Hence, the correct answer is option 3.
Discuss the solution with Testfunda users.

Solution #147: (06-Sep-09)


Let the base of the number system used in mars be n.
(15)n, (40)n, (k01)n and (122)n are in arithmetic progression.
When expressed in the decimal system, these numbers will be (5 + n), 4n, (1 + kn2) and (n2 + 2n + 2),
respectively.
If the numbers are in AP in base n then their corresponding values in base 10 will also be in AP.
(5 + n), 4n, (1 + kn2) and (n2 + 2n + 2) are in arithmetic progression in the decimal number system.
Since in a AP, the difference between any two consecutive terms is equal,
4n (5 + n) = (1 + kn2) 4n
7n kn2 = 6

(i)

Also, (1 + kn2) 4n = (n2 +2n +2) (1 + kn2)


2kn2 = n2 + 6n
2kn = n + 6

...[ n 0+

2kn n = 6

...(ii)

Equating (i) and (ii), we get


7n kn2 = 2kn n
7 kn = 2k 1

...[ n 0+

2k 8 + kn = 0

Book 2- Question of the Day

237

www.TestFunda.com

The LEADER in Online Test Prep

For k to be an integer, we can take n = 0, 2 or 6.


But n = 0 is not possible.
For n = 2, k = 2 and for n = 6, k = 1
Since n is the base of the number system and k01 is the number expressed in the system to the base
n, k < n. Therefore, n = 2 is not possible.
So k = 1 and n = 6.
Therefore expressing the number of chocolates in the decimal system, we get
(15)6 = 11
(40)6 = 24
(k01)6 = (101)6 = 37
(122)6 = 50
So the number of chocolates with Jadoo, Badoo, Kadoo and Ladoo are 11, 24, 37 and 50,
respectively.
The total number of chocolates that Aarav counted in the decimal number system = 11 + 24 + 37 +
50 = 122
Option 1 can be eliminated.
Now (122)10 can be expressed ass 442 and 233 in base 5 and base 7 respectively.
Options 3 and 2 can also be eliminated.
Now (122)10 = (322)6 = (233)7
(122)10 cannot take any value between 233 and 322 in any base.
278 cannot be a valid representation of (122)10
Hence, option 4.
Discuss the solution with Testfunda users.

Solution #148: (07-Sep-09)


Option 1 is incorrect in The Gay Science dealing with the origin ...
Option 2 and 4 are incorrect because of two well-defined periods- there are three.
Option 5 is incorrect because of the Romantic perspective followed by Schopenhauer, Wagner...
The Romantic perspective and the influence of Schopenhauer and Wagner is the same period.
Option 3 captures the essence of the paragraph without distorting the original ideas.
Hence, the correct answer is option 3.
Discuss the solution with Testfunda users.

Book 2- Question of the Day

238

www.TestFunda.com

The LEADER in Online Test Prep

Solution #149: (08-Sep-09)


Let N = abcdefghij be a multiple of 11111 such that a, b, c, d, e, f, g, h, i and j are distinct.
a, b, c, d, e, f, g, h, i and j will be equal to 1, 2, , 9 in some order.
a + b + c + d + e + f + g + h + i + j = 1 + 2 + 3 ++9 = 45.
The sum of digits of N is divisible by 9.
N is divisible by 9.
Now, 11111 = 41 271
gcd( 11111, 9) = 1
N has to be divisible by 11111 and 9 both.
N is divisible by 99999.
N = abcde 100000 + fghij is divisible by 99999.
abcde + fghij is divisible by 99999.
abcde + fghij = 99999k, for some natural number k.
But abcde < 99999 and fghij < 99999
abcde + fghij < 199998
abcde + fghij = 99999
a+f=b+g=c+h=d+i=e+j=9
(a, f), (b, g), (c, h), (d, i), (e, j) can take values (0, 9), (1, 8), (2, 7), (3, 6) and (4, 5).
As a = 0 is not possible, therefore, we have two cases.
Case 1: (a, f) (0, 9) or (9, 0)
(a, f) can take four possible pairs of values (1, 8), (2, 7), (3, 6) and (4, 5). Also, each value can swap
positions, i.e. (a, f) can be both (1, 8) and (8, 1) in two ways.
Number of possible values of (a, f) = 4 2!
(b, g) can take four possible pairs of values if the value taken by the pair (a, f) are not considered.
Also, each value can swap positions in two ways.
Number of possible values of (b, g) = 4 2!
Similarly, number of possible values of (c, h) = 3 2! , number of possible values of (d, i) = 2 2! and
number of possible values of (e, j) = 1 2!
Total possible values from case 1 = (4 4 3 2 1) (2!)5
= 4 4! 25
Case 2: (a, f) = (9, 0)
(a, f) can take one value (9, 0). The vales cant swap positions as a cannot be zero.
(b, g) can take four possible pairs of values (1, 8), (2, 7), (3, 6) and (4, 5). Also, each value can swap
positions, i.e. (b, g) can be both (1, 8) and (8, 1) in two ways.
Number of possible values of (b, g) = 4 2!
Similarly, number of possible values of (c, h) = 3 2! , number of possible values of (d, i) = 2 2! and
number of possible values of (e, j) = 1 2!
Total possible values from case 1 = (1 4 3 2 1) (2!)4
= 4! 24
Number of 10-digit positive integers with distinct digits that are multiples of 11111
= 4 4! 25 + 4! 24
= 3456
Hence, option 3.
Discuss the solution with Testfunda users.

Book 2- Question of the Day

239

www.TestFunda.com

The LEADER in Online Test Prep

Solution #150: (09-Sep-09)


The probable starters by comparison are statements C and D. Statements A and B get easily
eliminated in comparison to C and D. This eliminates options 4 and 5.
DA and and CB are mandatory pairs.
Evaluating options 1, 2 and 3, option 1 gets eliminated because of the ED pairing in option 1- which
makes no sense.
The pairing AB in option 3 is similarly illogical.
Therefore, the most logical sequence is DACBE.
Hence, the correct answer is option 2.
Discuss the solution with Testfunda users.

Solution #151: (10-Sep-09)


Let the number k have 8 factors.
If a number N = ambncp... where a,b,c... are prime numbers, then the number of factors of
N = (m + 1)(n + 1)(p +1)...
Here, N = k and (m + 1)(n + 1)(p + 1) = 8
The number of ways in which 8 can be written as a product of two or more numbers is,
8=18
=24
=222
Case 1: 8 = 1 8
To satisfy this case, k = a(8 1) b(1 1), where a and b are prime numbers.
In this case k = a7, for some prime number a.
k3 = a21 , for some prime number a.
The number of factors of k 3 in Case 1 = (21 + 1) = 22
Case 2: 8 = 2 4
To satisfy this case, k = a(2 1) b(4 1), where a and b are prime numbers.
In this case k = a1b3 , for some prime numbers a and b.
k3 = a3b9 , for some prime numbers a and b.
The number of factors of k 3 in case 2 = (3 + 1) (9 + 1)
= 4 10
= 40
Case 3: 8 = 2 2 2
To satisfy this case, k = a(2 1) b(2 1) c(2 1) where a, b and c are prime numbers.
In this case k = a1b1c1, for some prime numbers a, b and c.
k3 = a3 b3 c3 , for some prime numbers a, b and c.
The number of factors of k 3 in case 3 = (3 + 1) (3 + 1) (3 + 1)
=444
= 64
There is no other way of expressing 8 as a product of positive integers.
The total number of factors of k 3 can be either 22, 40 or 64 but not 27.
Hence, option 2.
Discuss the solution with Testfunda users.
Book 2- Question of the Day

240

www.TestFunda.com

The LEADER in Online Test Prep

Solution #152: (11-Sep-09)


The relationship is antonymous. Ephemeral means temporary; Perennial means enduring.
Analogous means similar, alike or uniform. Differentiated is non uniform or dissimilar.
Inducible means persuaded or influenced and Deducible, which means inferable or concludable
are not antonyms.
In Option 5, transcendental and metaphysical are almost synonymous with both of them being
meaning lofty or abstract.
Option 4 is verb and noun and has no apparent relationship except in appearance. Affect means to
influence and Affectation means an artificial trait. They show no antonymous relationship.
Option 1 is a change of form. Evolution leads to transformation. They are certainly not antonyms.
Hence, the correct answer is option 3.The relationship is antonymous. Ephemeral means
temporary; Perennial means enduring.
Analogous means similar, alike or uniform. Differentiated is non uniform or dissimilar.
Inducible means persuaded or influenced and Deducible, which means inferable or concludable
are not antonyms.
In Option 5, transcendental and metaphysical are almost synonymous with both of them being
meaning lofty or abstract.
Option 4 is verb and noun and has no apparent relationship except in appearance. Affect means to
influence and Affectation means an artificial trait. They show no antonymous relationship.
Option 1 is a change of form. Evolution leads to transformation. They are certainly not antonyms.
Hence, the correct answer is option 3
Discuss the solution with Testfunda users.

Book 2- Question of the Day

241

www.TestFunda.com

The LEADER in Online Test Prep

Solution #153: (12-Sep-09)


To find the maximum value of one of the 30 integers, we minimize the remaining 29 integers
As all the integers are positive and distinct, the minimum values that the 29 integers can have is
12381, 12382, 12383, . . ., 12408, 12409
The sum of the first 29 numbers = 12381 + 12382 + ...+ 12409
= (12380 + 1) + (12380 + 2) + ...+ (12380 + 29)
= (12380 29) + (1 + 2 + ... + 29)
= 359455
The sum of 30 integers with an average 12410 = 12410 30
= 372300
The maximum value that the 30th integer can take is 372300 359455 = 12845
Hence, option 3.
Note: Another method for calculating 12381 + 12382 + ...+ 12409 is,
12381 + 12382 + ... + 12409 = (12000 29) + (300 19) + (400 10) + (80 9) + (90 10) + 3 (1 +
2 + ... + 9)
= (12000 29) + (300 19) + (400 10) + (80 9) + (90 10) + (3 45)
But the method given in the solution is quicker.
Discuss the solution with Testfunda users.

Solution #154: (13-Sep-09)


The relationship is that of synonym. Prevaricate- to deviate from the truth and Equivocate- to
avoid committing oneself to what one says- meaning to lie or be evasive. This synonymous
relationship is to be found only in option 3.
Procrastinate- to put off or delay is synonymous with protract- to extend or delay.
Reschedule and resume are almost opposites.
Obviate means to prevent and elucidate is to explain.
Advocate and abandon are also almost opposites and show no synonymous relationship.
Hence, the correct answer is option 3.
Discuss the solution with Testfunda users.

Book 2- Question of the Day

242

www.TestFunda.com

The LEADER in Online Test Prep

Solution #155: (14-Sep-09)


The number of ways of writing a number N as product of two integers is equal to the number of
rectangles that can be formed with integral sides whose area is N square units.
It is given that the number of ways of writing 5n as a product of two integers is 9 and 5n is not a
perfect square.
Number of factors of 5n = 9 2 = 18
It is given that the number of ways of writing 7n as a product of two integers is 12 and 7n is not a
perfect square.
Number of factors of 7n = 12 2 = 24
Now, 18 = 1 18 or 2 9 or 3 6
Case 1: For 18 = 1 18
5n = a(1 1) b(18 1) = b17, for some prime numbers a and b.
5n = b17, for some prime number b.
b = 5 and n = 516
But then 7n = 7 516
Number of factors of 7n = (1 + 1) (16 + 1)
= 2 17
= 34
Case 1 is inadmissible.

Case 2: For 18 = 3 6
5n = a(3 1) b(6 1) = a2b5, for some prime numbers a and b.
5n = a2b5, for some prime numbers a and b.
As 5 is a prime number, therefore, either a = 5 or b = 5.
Subcase 1: a = 5
5n = 52b5, for some prime number b.
n = 5b5, for some prime number b.
7n = 7 5 b5
Book 2- Question of the Day

243

www.TestFunda.com

The LEADER in Online Test Prep

Either b = 7 or b is a distinct prime number.


But b cannot be a distinct prime number as 35n has to be a perfect square.
If b = 7 then 7n = 76 5
The number of factors of 7n = (6 + 1) (1 + 1)
=72
= 14 (inadmissible)
Subcase 2: b = 5
5n = a255, for some prime number a.
n = 54 a2, for some prime number a.
7n = a2 54 7
Either a = 7 or a is a distinct prime number but in both the cases 35n will not be a perfect square.
Case 2 is inadmissible.
Case 3: 18 = 2 9
5n = a(2 1) b(9 1) = a1b8, for some prime numbers a and b.
5n = a1b8, for some prime numbers a and b.
As 5 is a prime number, therefore, either a = 5 or b = 5.
Subcase 1: a = 5
5n = 51b8, for some prime number b.
n = b8, for some prime number b.
7n = 7 b8
In this case 35n will not be a perfect square.
Subcase 1 is inadmissible.
Subcase 2: b = 5
5n = a158, for some prime number a .
n = 57a, for some prime number a.
7n = 7 57 a

Book 2- Question of the Day

244

www.TestFunda.com

The LEADER in Online Test Prep

Either a = 7 or a is a distinct prime number.


If a is a distinct prime number then,
Number of factors of 7n = (1 + 1) (7 + 1) (1 + 1)
=282
= 32 (inadmissible)
If a = 7 then,
7n = 72 57
Number of factors of 7n = (2 + 1) (7 + 1)
=38
= 24
Also, 35n = 72 58
35n is also a perfect square.
a = 7 and b = 5
The number of ways of writing (72 58) as a product of two numbers is equal to the number of
rectangles with integral sides whose area is 35 square units.
The number of rectangles with integral sides that have area 35n

= 14
Hence, option 2.
Discuss the solution with Testfunda users.

Solution #156: (15-Sep-09)


Option 1 is incorrect because it states that Yeatss symbolism arose from his own experiences.
Option 2 is ambiguous and awkward in achieved peculiar vitality in alignment with his own
experience. Therefore, it is eliminated.
Option 3 is partial- it misses the very important point about how Yeatss symbolism achieved
vitality.
Option 4 is incorrect because it states based on traditional images with a peculiar vital quality
distorting the ideas.
Option 5 captures the essence and more importantly does not distort what is stated in the
paragraph.
Hence, the correct answer is option 5.
Discuss the solution with Testfunda users.
Book 2- Question of the Day

245

www.TestFunda.com

The LEADER in Online Test Prep

Solution #157: (16-Sep-09)


The idea is not to divide the numbers directly but to find a divisibility rule and use it.
The remainders given by 1 and 100 on division by 101 are 1 and 1 respectively.
Also, all the numbers given in the options are twelve digit numbers.
So we will find a condition such that a number abcdefghijkl is divisible by 101.

Remainder given by abcdefghijkl on division by 101

abcdefghijkl is divisible by 101 if kl ij + gh ef + cd ab is divisible by 101.


Also if it is not, the remainder obtained on division of abcdefghijkl by 101 is same as the remainder
obtained on division of kl ij + gh ef + cd ab by 101.
We apply this divisibility rule to the given options.

Option 1
123456789123 gives the same remainder as 23 91+ 78 56 + 34 12 = 24 = 101 24 = 77 on
division by 101.
123456789123 gives a remainder 77 on division by 101.

Option 2
231456789231 gives the same remainder as 31 92 + 78 56+14 23 = 48 = 101 48 = 53 on
division by 101.

Book 2- Question of the Day

246

www.TestFunda.com

The LEADER in Online Test Prep

231456789231 gives a remainder 53 on division by 101.

Option 3
312456789312 gives the same remainder as 12 93 + 78 56 + 24 31 = 66 = 101 66 = 35 on
division by 101.
312456789312 gives a remainder 35 on division by 101.

Option 4
213456789213 gives the same remainder as 13 92 + 78 56 + 34 21 = 44 = 101 44 = 57 on
division by 101.
213456789213 gives remainder 57 on division by 101.
123456789123 gives the largest remainder.
Hence, option 1.
Discuss the solution with Testfunda users.

Solution #158: (17-Sep-09)


The midst of a bust and the feeling the need to do something are related. Hence the header
statement is related to statement F. This eliminates options 1,2 and 3.
The but in D follows from F. Therefore, option 5 is eliminated.
The bottom of the cycle and the top of the cycle are logically linked in BC which in turn is linked
with the mention of the cycle in statement D. We need to to acknowledge- E and if we dontG go well together making the series FDBCE the most logical sequence.
Hence, the correct answer is option 4.
Discuss the solution with Testfunda users.

Book 2- Question of the Day

247

www.TestFunda.com

The LEADER in Online Test Prep

Solution #159: (18-Sep-09)


We know that x2 gives the same remainder as 100 does on division by 19.
x2 gives remainder 5 on division by 19.
x2 5 is divisible by 19.
Also, x2 5 95 will be divisible by 19.
[Note: We do this to convert x2 5 in the form (x a)(x + a), where a is an integer in order to
simplify calculations.]
x2 100 is divisible by 19.
(x 10)(x + 10) is divisible by 19.
Either (x 10) is divisible by 19 or (x + 10) is divisible by 19.

Case 1: (x 10) is divisible by 19


x = 10, 29, 48, 67, 86

Case 2: (x + 10) is divisible by 19


x = 9, 28, 47, 66, 85
x can take 10 values in all.
Hence, option 4.
Discuss the solution with Testfunda users.

Solution #160: (19-Sep-09)


We need to first understand a critic has written the paragraph about Kallats art and then uses
probably one of his quotes to close the paragraph. The urban milieu of Mumbai remains his primary
muse... the details of this life are spelt out in the paragraph. An apt comment from the artist will
explain this muse of the artist. No other comment of the writer is directly related to what the critic
has explained in the paragraph. In order to explain the influence of Mumbai life on the artist the
critic uses the quote from the writer stated in option 5. No other option would help the critic this
way. Therefore, options 1,2,3 and 4 can be eliminated. The correct answer option must link with
Kallat's primary muse- the urban milieu of Mumbai.
Hence, the correct answer is option 5.
Discuss the solution with Testfunda users.

Book 2- Question of the Day

248

www.TestFunda.com

The LEADER in Online Test Prep

Solution #161: (20-Sep-09)


Let the side lengths of the rectangular floor be m and n.
Area of each tile is 1 square unit.
Total number of tiles used mn.
Area to be covered with blue tiles = (m 2)(n 2)
= mn 2m 2n + 4
The number of purple tiles = total number of tiles number of blue tiles
= mn (mn 2m 2n + 4)
= 2m + 2n 4
As, the number of blue tiles is equal to the number of purple tiles, we have
2m + 2n 4 = mn 2m 2n + 4
mn 4m 4n + 8 = 0
(m 4)(n 4) = 8 = 8 1 = 4 2 = 2 4 = 1 8
As, m, n are integers, so are m 4 and n 4.
m 4 = 8 and n 4 = 1 which gives (m, n) = (12, 5)
Similarly from the other cases, we get (m, n) = (8, 6), (6, 8), (5, 12)
12, 5, 6 and 8 units can be side length of the rectangular floor.
Only 16 cannot be the side length of the rectangular floor.
Hence, option 5.
Discuss the solution with Testfunda users.

Solution #162: (21-Sep-09)


The gist of the passage is: Quantity theory of money is the foundation stone of Monetarism; the
theory is built on the Fisher equation, MV = PT; V and T are constant. Hence M (stock of money) = P
(price level). Increase M, the price level increases. Without the formula and other details option 4
captures this essence.
Option 1 mistakes Monetarism for Quantity theory of money.
Option 2 and 5 miss mentioning the very important Quantity theory of money part.
Option 3 is a nonsensical option.
Hence, the correct answer is option 4.
Discuss the solution with Testfunda users.
Book 2- Question of the Day

249

www.TestFunda.com

The LEADER in Online Test Prep

Solution #163: (22-Sep-09)

Hence, option 2.
Discuss the solution with Testfunda users.

Solution #164: (23-Sep-09)


Options 1 and 3 miss out on the countries part. Further, analysis of information for the purpose of
forecasts and planning is factually incorrect.
Option 4 misses out on the purposes part.
In option 2 it is compressed into development of information.
Option 5 also misses out on the development of data.
Hence, the correct answer is option 2.
Discuss the solution with Testfunda users.

Book 2- Question of the Day

250

www.TestFunda.com

The LEADER in Online Test Prep

Solution #165: (24-Sep-09)


X and Y have no common prime factors.
This means that gcd(X, Y) = 1.
Also, X2 + Y2 = (X + Y)2 2XY
We know that Z divides (X2 + Y2) and (X + Y), and therefore, it will divide either 2 or XY.
Case 1: Z divides XY.
Z either divides X or divides Y.

* gcd(X, Y) = 1]

If Z divides X, since it divides (X + Y), it must divide Y as well and vice versa.
Z divides X and Y both.
As gcd(X, Y) = 1, therefore, Z = 1
Case 2: Z divides 2.
From this case , Z = 1 or 2.
From case 1 and case 2, Z can take two values 1 and 2.
Hence, option 3.
Discuss the solution with Testfunda users.

Solution #166: (25-Sep-09)


The conclusion is that exposure to microbes strengthens the immune system aganist diseases.
We look at the options in the order of correctness:
Option 4 supports the argument.
Options 2 and 5 provide data that does not directly challenge the conclusion. Option 2 supports the
immune system being strengthened by growing up on farms - this is another source of reducing
allergies, but does not weaken the conclusion. Option 5 talks about the process of getting allergies this again does not weaken the conclusion.
The choice is between option 1 and 3. Option 3 states that children suffering from infections etc had
been exposed to disease causing microorganisms by older siblings and attended nursery and had
not improved immunity. This directly contradicts the paragraph, and is more direct than option 1
with "day care" (which brings in issues of whether the children are taken care of properly.......).
Hence, the correct answer is option 3.
Discuss the solution with Testfunda users.

Book 2- Question of the Day

251

www.TestFunda.com

The LEADER in Online Test Prep

Solution #167: (26-Sep-09)


Consider the first term of A.

.
.

Adding all these terms, we see that the first term of each pair cancels with the second term of the
previous pair, leaving only the first and last terms of the entire sequence.

Hence, option 2.
Discuss the solution with Testfunda users.

Solution #168: (27-Sep-09)


There is nothing in the paragraph to support options 1 and 4.
Option 2 goes beyond the paragraph - "less than 80%", aganist the 80% stated in the paragraph.
Given the fact that the Aviation sector will not meet this target, there is no reason for us to conclude
that the overall target will be exceeded.
We are left to choose between options 3 and 5. Look at the argument made in the paragraph:
The overall target for reduction (of emissions in the UK) by 2050 is 80%. This includes aviation
industry. The target for aviation is now reduced (to 1990 levels rather than those of 2005 obviously
in 2005 aviation industry emissions were greater).
The overall target can be achieved in two ways 1. Having the aviation sector fulfill its share by dropping to 80% of 1990 levels, or
2. By having other sectors of the economy make deeper cuts to make up the loss caused by the
aviation sector not picking up its share.
Method A may affect the aviation sector, but Method B does not. Hence, the "will" in option 5 is
difficult to justify.
The "may" in option 3 makes it the correct option, since it fits in with Method B, and the "may" let us
choose it with even the possibility of it happening.
Hence, the correct answer is option 3.
Discuss the solution with Testfunda users.

Book 2- Question of the Day

252

www.TestFunda.com

The LEADER in Online Test Prep

Solution #169: (28-Sep-09)


Let the cubes be initially kept in the following order.

The labels 1, 2, 3, 4 are camouflaged as a, b, c, d (In some order).


Step 1: Amar takes the leftmost cube and places it rightmost.

Step 2: Amar takes the third cube from the right and places it rightmost.

After the two steps the labels on the blocks get arranged in ascending order.

a = 3, b = 1, c = 4 and d = 2
Initially the cubes were arranged in the following order.

Hence, option 4.
Note: Exchange it with the rightmost cube and places it rightmost have different connotations.
Discuss the solution with Testfunda users.
Book 2- Question of the Day

253

www.TestFunda.com

The LEADER in Online Test Prep

Solution #170: (29-Sep-09)


Proscribe implies a ban. Prescribe implies ordering the use of a medicine or other treatment
and would be the apt word here. Therefore, options 1 and 3 are eliminated.
Tantamount means equal to; paramount means supreme. Tantamount is the correct
word contextually. Therefore, option 5 is eliminated.
Denigrate is to belittle or defame and is the appropriate word in the context of the sentence.
Degrade means a lowering of grade. Therefore, option 4 is eliminated.
Hence, the correct answer is option 2.
Discuss the solution with Testfunda users.

Solution #171: (30-Sep-09)


We know that, f(x) < 0 for all values of x, except x = 2. Also, f(x) is a parabola of the form y = x2,
therefore, f(x) will be a downward parabola with vertex at x = 2.
f(x) = (x 2)2
Also, g(x) < 0 for all values of x except 2 x 3. Also, g(x) is a parabola of the form of y = x2,
therefore, g(x) will be an upward parabola intersecting the x axis at x = 2 and x = 3.
g(x) = (x 2) (x 3)

Let the abscissa of point M be m.

Book 2- Question of the Day

254

www.TestFunda.com

The LEADER in Online Test Prep

We can see from the graph that the minimum value of h(x) is at point M.
M is the point of intersection of the curves f(x) and g(x) other than (2, 0).
The abscissa of M will be the root of the equation, f(x) g(x) = 0 other than x = 2.
f(x) g(x) = 0
(x 2)2 (x 2) (x 3) = 0
(x 2)2 + (x 2) (x 3) = 0
(x 2)*(x 2) + (x 3)+ = 0
(x 2)(2 x 5) = 0

Hence, option 4.
Discuss the solution with Testfunda users.

Solution #172: (01-Oct-09)


Poltergeist is a ghost or an apparition. The relationship between the words is synonymous.
Balderdash means nonsense.
Moonshine means nonsense.
Alacrity means alertness
Avidity means ardent desire or craving; eagerness
Riposte means reply or retort
Therefore, moonshine is the only option which is in a synonymous relationship with balderdash.
Hence, the correct answer is option 1.
Discuss the solution with Testfunda users.
Book 2- Question of the Day

255

www.TestFunda.com

The LEADER in Online Test Prep

Solution #173: (02-Oct-09)


Consider values of f(x) for different values of x.

Consider values of g(y) for different values of y.

For different natural numbers x, f(x) can take values 0, 1, 2, 3, 4 only.


Also, for different natural numbers y, g(y) can take values 0, 1, 2 only.
Hence the product f(x) g(y) can take values 0, 1, 6 and 8, but it can never take the value 12.
Hence, option 5.
Discuss the solution with Testfunda users.

Solution #174: (03-Oct-09)


Turbid is generally used in reference to a liquid and means 'cloudy or opaque'; turgid means
swollen and is the correct word here. Therefore, options 2 and 5 are eliminated.
Exceptionable means open to objection; exceptional means excellent. The actors performance
was exceptional. Therefore, option 3 is eliminated.
A fawn is a young deer, and a light brown colour; a faun is a Roman deity that is part man, part
goat. In this case, the animal was light brown in colour.
Unsociable means not enjoying the company of or engaging in activities with others; insocial is
not a word.
Venal means associated with bribery; corruptible; venial, is used in Christian theology in
reference to a minor sin.
Hence, the correct answer is option 4.
Discuss the solution with Testfunda users.

Book 2- Question of the Day

256

www.TestFunda.com

The LEADER in Online Test Prep

Solution #175: (04-Oct-09)


Since a, b, c, p, q and r are all distinct real numbers, therefore, AM > GM
As a and p are distinct real numbers, therefore, a2 and p2 are also distinct real numbers.

a2 + p2 > 2ap

(i)

Similarly, b2 + q2 > 2bq


And, c2 + r2 > 2cr

(ii)
(iii)

(i) + (ii) + (iii) gives,


a2 + p2 + b2 + q2 + c2 + r2 > 2ap + 2bq + 2cr
(a2 + b2 + c2) + (p2 + q2 + r2) > 2ap + 2bq + 2cr
101 + 101 > 2ap + 2bq + 2cr
202 > 2(ap + bq + cr)
ap + bq + cr < 101
Hence, option 2.
Discuss the solution with Testfunda users.

Solution #176: (05-Oct-09)


Option 1 strengthens the argument by linking the earlier cause of happiness (education and income)
to height. This makes height (almost) responsible for income, education, and happiness as well.
Option 3 is tangential to the main argument. It does not bring height into the picture.
Option 2 talks about the very tallest people whereas the paragraph talks about the taller ones. This
changes the context under consideration.
Option 4 brings in a new element- 'height tolerance' which, is ambiguous- tolerance by whom? The
essence of the paragraph is circumvented here.
Option 5 directly negates a key element of the argument- that taller men experience more positive
emotions.
Hence, the correct answer is option 5.
Discuss the solution with Testfunda users.

Book 2- Question of the Day

257

www.TestFunda.com

The LEADER in Online Test Prep

Solution #177: (06-Oct-09)


Consider the given inequality,
ABC

= 27p2 30pq 25q2


27p2 30pq 25q2 0

p, q > 0, therefore, n > 0


27n2 30n 25 0
27n2 + 15n 45n 25 0
3n(9n + 5) 5(9n + 5) 0
(3n 5)(9n + 5) 0
Either (3n 5) 0 and (9n + 5) 0 or (3n 5) 0 and (9n + 5) 0

Case 1: (3n 5) 0 and (9n + 5) 0

Book 2- Question of the Day

258

www.TestFunda.com

The LEADER in Online Test Prep

This case is inadmissible as n > 0.

Case 2: (3n 5) 0 and (9n + 5) 0

2x2 5x + 1 0

Hence, option 2.
Discuss the solution with Testfunda users.

Solution #178: (07-Oct-09)


The apparent paradox is that in spite of there being several b-schools their demand is increasing
(even with higher fees price/value in the paragraph).
Option 1 does not resolve this apparent paradox.
Option 2 is applicable to only some of the b-schools hence the general increase in demand is not
resolved.
Option 4 may account for the increase in fees, but not the increase in demand.
Option 5 talks about graduates rather vague as the issue of management education/management
graduates needs to be addressed. Option 3 resolves the paradox by giving a reason for people to
rush to the B-schools even though costly as the corporate rewards them later.
Hence, the correct answer is option 3.
Discuss the solution with Testfunda users.

Book 2- Question of the Day

259

www.TestFunda.com

The LEADER in Online Test Prep

Solution #179: (08-Oct-09)

We have to consider two cases here.


Case 1:
2x + 7 > 1 i.e. x > 3

(i)

In this case, from the given inequality we have


x2 < 3x + 18
x2 3x 18 < 0
(x + 3)(x 6) < 0
3 < x < 6

(ii)

From (i) and (ii), we have


x (3, 6)

(iii)

Case 2:
0 < 2x + 7 < 1

In this case, from the given inequality we have


x2 > 3x + 18
x2 3x 18 > 0
(x + 3)(x 6) > 0
x < 3 or x > 6
x (, 3) (6, )

(v)

From (iv) and (v), we have

Book 2- Question of the Day

260

www.TestFunda.com

The LEADER in Online Test Prep

Finally, from (iii) and (vi), we have

But x cannot be 0 as log 0 and consequently P is not defined.


Therefore,

and x 0
Hence, option 5.
Discuss the solution with Testfunda users.

Solution #180: (09-Oct-09)


Option 1 helps to support the findings that women not only earn less but the gap grows over time.
The out of the work force women pull down the average (total earning). The gap grows over time
because more of them are dropping out of work force. Option 2 and 3 are skewed in favour of
women rather than men and may help to weaken the argument rather than support it. Options 4
does nothing to weaken or strengthen the argument. 75% in option 5 will not impact the average
(mean). It may, however, weaken the argument considering it may be a skewed sample data.
Hence, the correct answer is option 1.
Discuss the solution with Testfunda users.

Book 2- Question of the Day

261

www.TestFunda.com

The LEADER in Online Test Prep

Solution #181: (10-Oct-09)


By definition,

and f3(x) = f0(f2(x)) = f0(x)


Since f3(x) = f0(x), we can conclude that fn(x) = fk(x), where k is the remainder obtained when n is
divided by 3.
f2009(2009) = f2(2009) = 2009
Hence, option 2.
Discuss the solution with Testfunda users.

Solution #182: (11-Oct-09)


Option 3 is not assumed - as cancer being less or more life threatening the study merely indicates
there may be a relation.
Option 5 talks about more research whereas the question is related to this particular research.
We are left three options, which are more difficult to eliminate from.
Option 4 is tangential to the conclusion of the study - the converse could be true - that cancer makes
patients more susceptible to depression - quite possible given the prognosis that a cancer patient
may have.
Option 1 is a conclusion of the study, and not an assumption. Moreover, it talks only about
diagnosis, and not about symptoms. The study is broader in its scope to include all those with
psychological distress.
The study concludes that there is a need to screen cancer patients. The relation between
psychological distress and death in cancer patients has to be proved true for the authenticity of the
study. If all other factors affecting the survival (of cancer patients) were taken into consideration,
the link between psychological distress and death becomes obvious because of the death rates.
Hence, the correct answer is option 2.
Discuss the solution with Testfunda users.

Book 2- Question of the Day

262

www.TestFunda.com

The LEADER in Online Test Prep

Solution #183: (12-Oct-09)


Since A is a power of a prime number, all its factors will be powers of the same prime number.
Thus, the factors of A are 1, P, P2, P3, , PN.
X is the product of all positive integral factors of A.
X = 1 P P2 P3 PN
= P(1 + 2 + 3 + + N)

Thus,

Now, among the options, only option 2 gives a positive integer value of N.
Hence, option 2.
Discuss the solution with Testfunda users.

Solution #184: (13-Oct-09)


The argument to be undermined is innocent people should not be watched. This assumes that
innocent people are being watched or there is an intention to watch innocent people. (Do not
misinterpret it as innocent people coming in view of the cameras watch has a different
significance). Option 4 states that there is no intention to watch the innocent people; the cameras
are installed with an intention to observe criminals and not innocent people. None of the other
options deal with the issue of innocent people being watched or not watched.
Hence, the answer is option 4.
Discuss the solution with Testfunda users.

Book 2- Question of the Day

263

www.TestFunda.com

The LEADER in Online Test Prep

Solution #185: (14-Oct-09)


Let n = 10000.
X = n(n + 1)(n + 2)(n + 3) + 1
= (n2 + 3n)(n2 + 3n + 2) + 1
= (n2 + 3n)2 + 2(n2 + 3n) + 1
= (n2 + 3n + 1)2
Substituting the value of n we get,
X = (100002 + 30000 + 1)2
= (100030001)2
= Y2
Y = 100030001
Hence, option 5.
Discuss the solution with Testfunda users.

Solution #186: (15-Oct-09)


This paragraph seems to have been written in a somewhat sarcastic tone. However, the conclusion
must still follow from what is given in the passage.
Options 1 and 4 cannot be concluded as these points go beyond the scope of what the passage is
talking about. Option 1 is the opposite of what has been discussed in the passage - the rights of the
husband with regard to his wife. Option 4 goes beyond male rights to conclude about marriage which cannot be done without looking at the wife's rights, and marriage in a holistic view.
Option 3 could have a conclusion only without the sarcastic tone. Moreover, it is quite generic.
Option 5 is ruled out as there is no mention of a letter being a legal document.
Option 2 is the most logical conclusion as the main argument dealt with in the passage is that the
husband has the right to read his wifes mail as he has legal ownership of her and her possessions.
Hence, the correct answer is option 2.
Discuss the solution with Testfunda users.

Book 2- Question of the Day

264

www.TestFunda.com

The LEADER in Online Test Prep

Solution #187: (16-Oct-09)


D = x3 + y3 (x y)(x2 + 4y2)
D = x3 + y3 x3 4xy2 + x2y + 4y3
D = y[5y2 4xy + x2]
D = y[x2 4xy + 4y2 + y2]
D = y[(x 2y)2 + y2]
y, (x 2y)2 and y2 are all positive.
D is positive.
Hence, option 1.
Discuss the solution with Testfunda users.

Solution #188: (17-Oct-09)


Who's is a contraction of who is or who has, while whose is the possessive, used in questions
such as whose is this? and whose turn is it? Whose is the correct word contextually. Therefore,
options 1 and 5 can be eliminated.
Wreath (n and v) with no 'e' at the end means 'arrangement of flowers'. Hence the plural wreaths;
while wreathe with an 'e' is a verb meaning 'envelop, surround, or encircle'. Wreaths is the
correct word contextually. Therefore, option 4 can be eliminated.
Ironic implies an attempt to be amusing or provocative by saying usually the opposite of what is
meant: made the ironic observation that the government could always be trusted. Sardonic
implies scorn, mockery, or derision that is manifested by either verbal or facial expression:
surveyed the scene with a sardonic smile. Therefore, option 2 can be eliminated.
Uninterested means to find something boring or dull. Disinterested means impartial.
Comprise is a verb meaning to consist of or be composed of. Compose means to make up the
constituent parts of. If we use comprises the sentence should read: ... division of India comprises
28 states and ...
Hence, the correct answer is option 3.
Discuss the solution with Testfunda users.

Book 2- Question of the Day

265

www.TestFunda.com

The LEADER in Online Test Prep

Solution #189: (18-Oct-09)


2686 = 686 + 1500 + 500
For simplicity let a = 686, b = 1500 and c = 500
X = (a + b + c)3 a3 b3 c3
= [(a + b + c)3 a3+ *b3 + c3]
= (a + b + c a)[(a + b + c)2 + a (a + b + c) + a2+ (b + c)[b2 bc + c2]
= (b + c) {[(a + b + c)2 b2] + a (a + c) + ab + bc + (a2 c2)}
= (b + c) {(a + b + c + b)(a + b + c b) + a (a + c) + b (a + c) + (a + c)(a c)
= (b + c)(a + c)[a + c + 2b + a + b + a c]
= 3 (a + b)(b + c)(c + a)
3, (686 + 500) = 1186, (1500 + 686) = 2186 and (1500 + 500) = 2000 are factors of X.
Hence, option 5.
Discuss the solution with Testfunda users.

Solution #190: (19-Oct-09)


A perquisite is a special right or privilege enjoyed as a result of one's position; prerequisite is
something that is required as a prior condition for something else. Perquisite is the correct word
here. Therefore, options 2 and 5 are eliminated.
Perspicuous means 'expressing things clearly' and is the apt word here while perspicacious
means 'having a ready understanding of things'. Therefore, option 3 is eliminated.
Principal in this context means a company represented by a sales person. Principle means an
accepted rule of action or conduct and is inappropriate in this context.
Proscribe is a rather formal word meaning 'condemn or forbid' and is the correct word here,
whereas prescribe means either 'issue a medical prescription' or 'recommend with authority'.
Therefore, option 1 is eliminated.
Do not confuse regretful which means 'feeling or showing regret', with regrettable which means
'giving rise to regret; undesirable'. Regrettable is apt in this context.
Hence, the correct answer is option 4.
Discuss the solution with Testfunda users.

Book 2- Question of the Day

266

www.TestFunda.com

The LEADER in Online Test Prep

Solution #191: (20-Oct-09)


Since X, Y and Z are in Harmonic Progression,
Y is the harmonic mean of X and Z.

Hence, option 5.
Discuss the solution with Testfunda users.

Solution #192: (21-Oct-09)


The passage talks about strict laws related to certain kinds of associations. However, option 1 has
the issue that it talks about "amusement, etc" in general, and not necessarily through associations.
Option 2 takes away the entire responsibility of the ceremonies from the family - for which there is
no justification in the passage.
There were strict laws on associations other than funeral colleges - which could have resulted in a
lack of places for people to meet and interact. However, "raison d'etre" (the main reason for
existence) allows us to eliminate it.
By emulating the practices of the funeral colleges Christianity managed to hold meetings and keep
together as a community. The option is vague on how far beyond, which lets us conclude it since if
they survived the second and third centuries (with some degree of structure), they would have
survived for some at least beyond. This makes option 4 correct.
There is very little support for either "socio-political", or "socio-religious" in the passage, allowing us
to eliminate it.
Hence, the correct answer is option 4.
Discuss the solution with Testfunda users.

Book 2- Question of the Day

267

www.TestFunda.com

The LEADER in Online Test Prep

Solution #193: (22-Oct-09)


The name of soldiers can have at most 3 characters, and it must include at least one A.
Case 1: The name of the soldier is one character long.
The only name possible is A.
Only 1 soldier can have a name that is one character long.
Case 2: The name of the soldier is 2 characters long.
Total number of two character names = 26 26 = 262
Total number of two character names without the letter A = 25 25 = 252
Total number of two character names with A = 262 252 = 51
51 soldiers can have names that are two characters long.
Case 3: The name of the soldier is three characters long.
Proceeding in the same way as case 2,
Total number of three character names with either 1, 2 or 3 As = 263 253 = 1951
1951 soldiers can have names that are three characters long.
Total number of unique names satisfying the conditions given in the question
= Total number of names from case 1 + case 2 + case 3
= 1 + 51 + 1951
= 2003
Hence, option 1.
Note: It is not mentioned in the question that the name of a soldier cannot be "Ari". Therefore, we
can assume that the name of a soldier can be the same as that of the King.
Discuss the solution with Testfunda users.

Solution #194 (23-Oct-09)


Apposite means relevant or appropriate while the antonym to this word is opposite.
While referring to a persons hairline, the word receding is more apt.
Rigour means strictness or harshness. The medical term for stiffness after death is Rigor mortis.
An illness weakens a person while a test would be conducted to find out what ailed a persons
health.
A lien means the right to hold or sell a debtors property as payment or security against a debt.
Hence, the correct answer is option 4.
Discuss the solution with Testfunda users.

Book 2- Question of the Day

268

www.TestFunda.com

The LEADER in Online Test Prep

Solution #195: (24-Oct-09)


Consider the given expression,

Therefore,

y1 + y2 + + y8

Hence, option 4.
Discuss the solution with Testfunda users.
Book 2- Question of the Day

269

www.TestFunda.com

The LEADER in Online Test Prep

Solution #196: (25-Oct-09)


The parents have not been adopted. Instead they are adopting children so they are adoptive.
If something precedes x, it comes before x. Here, the word is proceed which is "to continue or
advance".
Born mean "brought forth by birth" while borne means, "to hold up; support as well as to bring forth
(young); give birth to". It is not possible to give birth to taunts so borne is a better fit than born.
Alternative has to do more with "choice" and alternate has more to do with "succession". Alternate
would be to "take turns" whereas alternative would be "the other choice". Thus, alternative is our
word here.
In the paragraph, the society does not seem to appreciate the couples adoption. Hence, they do not
think it as an act of bravery but more as an act of bravado which means "boldness or audacity".
Hence, the correct answer is option 1.
Discuss the solution with Testfunda users.

Solution #197: (26-Oct-09)

Clearly the maximum value of K would be .


Book 2- Question of the Day

270

www.TestFunda.com

The LEADER in Online Test Prep

Also, it is given that x < 0.


Therefore,

Hence, option 4.
Discuss the solution with Testfunda users.

Book 2- Question of the Day

271

www.TestFunda.com

The LEADER in Online Test Prep

Solution #198: (27-Oct-09)


To inflict a blow indicates that someone (doer) has "caused or meted out a blow". One is afflicted
(troubled, make distressed or upset) "by maladies or illnesses".
Allude is "to refer indirectly to something". To elude is "to escape or avoid". The lady used to allude
or refer to her imagined illnesses.
Unsufferable is used for persons or their behaviour (which is intolerable) and not for illnesses.
Insufferable is the right word here, as it means "difficult or impossible to endure, intolerable".
The fourth word is forebodings (future misfortunes) as the sentence ends with about future health
concerns. To forbid is "to command someone not to do something".
Excess is "more" whereas access, our last word here, is "right to use of (information)".
Hence, the correct answer is option 2.
Discuss the solution with Testfunda users.

Solution #199: (28-Oct-09)


Let N = 1! 2! 3! 9! 10!
We know that the power of a prime p in n! can be determined by the formula

Using it for n = 1, 2, 3, 10 and p = 2, 3, 5 and 7 in the formula we get,


1! 2! 3! 9! 10! = 237 317 57 74
Any divisor of N which is perfect square will be of the form 22a 32b 52c 72d.
a can take the values 0, 1, 2, 19 = 20 values.
Similarly b, c and d will take 9, 4 and 3 values, respectively.
Total number of divisors of N that are perfect squares = 20 9 4 3 = 2160.
Hence, option 2.
Discuss the solution with Testfunda users.

Book 2- Question of the Day

272

www.TestFunda.com

The LEADER in Online Test Prep

Solution #200: (29-Oct-09)


Backward here relates to conservative or orthodox or more correctly "not progressive". Therefore, it
is our first word for this paragraph. Backwards means "towards the back" or in a reverse order.
Antidotes are "cures or remedies" and the word does not apply here. Anecdotes are "witty tales or
stories". His anecdotes could blow away peoples gloom if they were annoyed or feeling bad about
something.
Aggravate is "to make worse" and is not suitable for this sentence.
One feels bad about someone behaving badly. Therefore, bad is our fourth word.
Elicit is "to draw out" (laughter in this case). Illicit is "illegal".
Hence, the correct answer is option 4.
Discuss the solution with Testfunda users.

Solution #201: (30-Oct-09)


x2 18x 1 = 2y 34
x2 18x 1 = 2y 81
x2 18x + 80 = 2y
(x 8)(x 10) = 2y
As RHS is a power of 2, hence both the terms of the LHS should also be powers of 2.
Since the terms on the left differ by 2, only (4, 2) and (2, 4) can be the two numbers represented
by (x 8) and (x 10).
For (4, 2) we get x 8 = 4 or x = 12 and thus y = 3
For (2, 4) we get x 8 = 2 or x = 6 and thus y = 3
Thus (12, 3) and (6, 3) are the only two ordered pairs satisfying the conditions.
Hence, option 3.
Discuss the solution with Testfunda users.

Book 2- Question of the Day

273

www.TestFunda.com

The LEADER in Online Test Prep

Solution #202: (31-Oct-09)


Sentences A and B have an agreement error. The phrase, all the facts is plural and should take the
verb are in place of is. Further, in B, the subject is process (singular) and therefore, the verb
permit should become permits.
The error in sentence E may perhaps get a wee bit difficult to figure out. To understand it, we can
condense the sentence, starting from seven tenths to ninety five hundredths of it.
Preposition to should replace preposition in (the one that precedes ninety-five) in the sentence.
Hence, the correct answer is option 2.
Discuss the solution with Testfunda users.

Solution #203: (01-Nov-09)

BPC = 72
BOC = 144
As AB and AC are tangents to the circle with centre O, ABO = ACO = 90
BAC = 360 (ABO + ACO + BOC)
BAC = 36
XAY = 36
XPY = 144

(opposite angles of cyclic quadrilateral are supplementary)

Hence, option 4.
Discuss the solution with Testfunda users.

Book 2- Question of the Day

274

www.TestFunda.com

The LEADER in Online Test Prep

Solution #204: (02-Nov-09)


Statement C has an error in parallelism. The elements of the sentence, the sharp morning air,
interest in training, the unexpected popping need to be parallel. We need to add an article (the)
just before the word interest and grotesque.
All the other statements are grammatically correct.
Hence, the correct answer is option 3.
Discuss the solution with Testfunda users.

Solution #205: (03-Nov-09)


Consider the given inequality,

We know that |xy| = |x| |y|


|6x| = |6||x| = 6|x|
And,
|2x| = |2||x| = 2|x|
Therefore, the given inequality is,

As,

12 4 4 + 2|x|
* 4 + 2|x| > 0]
|x| 22
22 x 22
x can take 45 integer values.
Hence, option 4.
Discuss the solution with Testfunda users.
Book 2- Question of the Day

275

www.TestFunda.com

The LEADER in Online Test Prep

Solution #206: (04-Nov-09)


Grammatically, comprise is not immediately followed by a preposition of. Therefore, the first word
is consists. We can eliminate options 1,3 and 5.
The author had self-belief or confidence and not confidant- which means one in whom secrets
are confided.
Similar to comprise, despite does not require a preposition immediately following it. Thus, we
will use in spite. Therefore, we can eliminate option 4.
Din is noise or commotion, dine means to to have dinner. Our word is dinning - from din.
Presently means in some time, when there is still some time left for something to happen or take
place. Therefore, the correct word is currently- which means now.
Hence, the correct answer is option 2.
Discuss the solution with Testfunda users.

Solution #207: (05-Nov-09)


Let the side length of AP, BP, CP be x, y and z, respectively.
The given data can be represented as follows,

Using Cosine Rule in APC,


28x cos = 142 + x2 z2

(i)

Using Cosine rule in ABP,


we get 24y cos = 122 + y2 x2

(ii)

Similarly, using cosine rule in BCP,


20z cos = 102 + z2 y2
Book 2- Question of the Day

(iii)
276

www.TestFunda.com

The LEADER in Online Test Prep

Adding, equations (i), (ii) and (iii), we get,


(28x + 24y + 20z) cos = 142 + 122 + 102

Also,
Area of ABC = Area of APC + Area of ABP + Area of BCP

Since, we have the lengths of the three sides of ABC, we can calculate its area using the Herons
formula,
Semiperimeter of ABC = (a + b + c)/2 = (10 + 12 + 14)/2 = 18

Hence, option 4.
Discuss the solution with Testfunda users.

Book 2- Question of the Day

277

www.TestFunda.com

The LEADER in Online Test Prep

Solution #208: (06-Nov-09)


There are 2 major clues. The first clue is series of failures or troubles in Georges life. The second
clue comes from last 2 lines, which indicate a financial crisis of some sort. Thus, option 1 which
combines both Georges misfortune and the financial crisis is the best option.
Option 2 can be negated as it follows from the last line alone and not from the entire paragraph.
Options 4 and 5 can be negated as they are mildly negative while the general trend of the paragraph,
calls for something which is strongly negative.
Option 3 is eliminated as it is contradictory to the given tone of the paragraph.
Hence, the correct answer is option 1.
Discuss the solution with Testfunda users.

Solution #209: (07-Nov-09)


By Wilsons theorem, a natural number p > 1 is a prime number iff (p 1)! + 1 is divisible by p.
16! gives a remainder 1 on division by 17.
From the solution above we know that, n gives the same remainder as 31!/17 on division by 17.

= (17 + 14) (17 + 13) (17 + 12) (17 + 1) (16 15 14 2 1)


= (17 + 14) (17 + 13) (17 + 12) (17 + 1) 16!
(17 + 14) (17 + 13) (17 + 12) (17 + 1) 16! gives the same remainder as
(14! 16!) on division by 17.
By Wilson's theorem we know that , 16! Gives remainder 1 on division by 17.
14! 15 16 gives remainder 1 on division by 17.
14! 15 16 gives the same remainder as 14! 2 1 on division by 17.
14! 15 16 gives the same remainder as 14! 2 on division by 17.
14! 2 gives remainder 1 on division by 17.
14! will give remainder 8 on division by 17.
14! 16! will give remainder 8 1 = 8 on division by 17.
14! 16! will give remainder 17 8 = 9 on division by 17.
31!/17 will give remainder 9 on division by 17.
n will give remainder 9 on division by 17.
Book 2- Question of the Day

278

www.TestFunda.com

The LEADER in Online Test Prep

Hence, option 4.
Alternatively,
The question can be solved without using Wilson's theorem as follows.
We can rewrite the equation as

The remainder of n on division by 17 is same as

Let this remainder be R.

All the terms of this expression except the last term are divisible by 17.
R is the remainder when (14 13 12 2 1)2 2 1 is divided by 17.
(14!)2 2 = ((14 11)(8 2)(13 4)(12 7)(10 5)(6 3)(9 1))2 2
= (154 16 52 84 50 18)2 81 2
We can see that all the terms in this expression, except 81 2, are of the form 17k 1
R will be the remainder when 81 2 is divided by 17.
R=9
Hence, option 4.
Discuss the solution with Testfunda users.

Book 2- Question of the Day

279

www.TestFunda.com

The LEADER in Online Test Prep

Solution #210: (08-Nov-09)


Sensuous implies gratification of the senses for the sake of aesthetic pleasure; the sensuous
delights of great music. Sensual tends to imply the gratification of the senses or the indulgence of
the physical appetites as ends in themselves. Sensual would be apt in the context of the sentence.
Therefore, options 2 and 5 are eliminated.
Luxurious suggests the providing of or indulgence of sensuous pleasure inducing bodily ease and
languor; a luxurious hotel. Voluptuous implies more strongly abandonment especially to sensual
pleasure; a voluptuous feast. Therefore, option 1 is eliminated since voluptuous is the apt word in
this context.
Veracious is truthful, honest or accurate. Voracious means ravenous or having a huge appetite
and is the correct word in this context.
Ternary is based on the number three. Tertiary means third in place, order, degree, or rank.
Here, we want to express a number system based on the number three. Therefore, option 3 is
eliminated.
Premier (adj) means first in rank and is the correct word contextually. Premiere as a noun
means first performance. Premiere is not an adjective.
Hence, the correct answer is option 4.
Discuss the solution with Testfunda users.

Solution #211: (09-Nov-09)


The line 3x 2y = 0 passes through origin.
Let k be the x-coordinate of the other end of the diameter.
The other end of the diameter is (k, 3k/2), where k is a real number.
As, the center is the midpoint of the diameter,

All the radii of a circle are equal, we will use it to find the value of k and thus center and hence radius
of the circle.
Using distance formula,

Squaring and solving we have

Book 2- Question of the Day

280

www.TestFunda.com

The LEADER in Online Test Prep

Hence, option 3.
Discuss the solution with Testfunda users.

Solution #212: (10-Nov-09)


Discreet (adj) wise, cautious and not saying anything which might cause trouble and is the right
word in this context. Discrete (adj) means constituting a separate entity. Therefore, options 2 and
4 are eliminated.
Expedient means based on or marked by a concern for self-interest rather than principle;
expeditious means fast and is the inappropriate word here. Therefore, option 1 is eliminated.
Eminently means very and is the appropriate word contextually; imminently means ready to
take place.
Elect through ballot and select is by choice. Select is the correct word here as the Prime
Minister selects members of the cabinet.
Hence, the correct answer is option 3.
Discuss the solution with Testfunda users.

Book 2- Question of the Day

281

www.TestFunda.com

The LEADER in Online Test Prep

Solution #213: (11-Nov-09)


Consider the given series,

Let Tn denote the nth term of the given series.

Hence, option 4.
Discuss the solution with Testfunda users.

Book 2- Question of the Day

282

www.TestFunda.com

The LEADER in Online Test Prep

Solution #214: (12-Nov-09)


Books embedded in an imbricated fashion means that the embedded books were regularly
arranged in an overlapping style.
The books are extant and not extinct, that is, they exist and are not lost. Extent is range, degree or
amount.
Diffident is shy or hesitant. The time or era was difficult or tough.
Hanged, as a past tense and a past participle of hang, is used in the sense of "to put to death by
hanging.
Hung is used in the sense of hung parliament or a photograph was hung on the wall. People who
fought the injustices of their time would certainly be hardy, resilient or tough people.
Hardly means barely or to almost no degree.
Hence, the answer is option 1.
Discuss the solution with Testfunda users.

Solution #215: (13-Nov-09)


Consider the given equation,

We have, p2 + 2p 15 = 0
(p 3 )(p + 5) = 0
p = 3 or 5

Book 2- Question of the Day

283

www.TestFunda.com

The LEADER in Online Test Prep

Case 1: p = 3

4x2 6x 3 = 0

Case 2: p = 5

4x2 + 10x + 5 = 0

Hence, option 4.
Discuss the solution with Testfunda users.

Book 2- Question of the Day

284

www.TestFunda.com

The LEADER in Online Test Prep

Solution #216: (14-Nov-09)


Abet (A) is to approve, encourage, and support. Betted is the past form of bet and does not suit the
context of the sentence.
The police interrogate (question) criminals. Celebrities and like or candidates seeking employment
are interviewed.
From the second sentence, if the robber kept to himself, it indicates that he may be reticent
(unforthcoming, uncommunicative) instead of reluctant (hesitant).
Taut is tight and if he wasnt very communicative, he was tight-lipped or taut-lipped (A). Tout is to
advertise, publicize or flaunt.
Site is a place or location. To cite is to quote or allude to and hence is our final word for this
paragraph.
Hence, the answer is option 3.
Discuss the solution with Testfunda users.

Solution #217: (15-Nov-09)


Consider the following inequality,

Squaring both sides we get ,

(301 2 17.3) < m < (301 + 2 17.3)


267 m 335
We get a total of 335 267 + 1 = 69 values for m.
Hence, option 1.
Discuss the solution with Testfunda users.

Solution #218: (16-Nov-09)


Moribund means dying. From the second sentence, if the psychopath was thrilled then he was
reading with morbid (sinister or gruesome) interest.
Palate is roof of the mouth whereas palette is the board on which a painter mixes colours or which
he uses while painting.
Here, the peasants (not the pheasant as the character commits homicides or human murders)
palate was pierced.
Pour is used in the sense of pouring (rain or water or financial aid). Here pored is to be used. It
means to read intently.
Prosecute is to initiate civil or criminal proceedings against someone in a court of law. Persecute is to
annoy or harass.
Hence, the answer is option 1.
Discuss the solution with Testfunda users.

Book 2- Question of the Day

285

www.TestFunda.com

The LEADER in Online Test Prep

Solution #219: (17-Nov-09)


Let A and B work on the project alone for x and y days respectively.
They work together for (13 x y) days.
A can finish the work alone in 12 days.

B can finish the work alone in 18 days.

When they work together, their efficiency is reduced by 20%, so they work at 80% of their efficiency.

52 x 2y = 36
x + 2y = 16
x and y can take values (2, 7), (4, 6), (6, 5), (8, 4), (10, 3), (12, 2) and (14, 1).
The maximum value that x + y can take is 13. Also, (13 x y) > 0.
x and y can take values (2, 7), (4, 6), (6, 5) and (8, 4).
By the condition given in the question, x + y should be a prime number, therefore, x = 6 and y = 5.
A works alone for 6 days, B works alone for 5 days and A and B work together for 2 days.
Hence, option 4.
Discuss the solution with Testfunda users.
Book 2- Question of the Day

286

www.TestFunda.com

The LEADER in Online Test Prep

Solution #220: (18-Nov-09)


Answer Option 1. The gist of the argument is as follows: Statistics may not be giving us the right
picture about the conflict between economic development and the protection of environment. If
statistics (measure) tells us that a better GDP is better life, we may do things to improve GDP and
these things may harm the environment, and vice versa. The conclusion that can follow from the
above is that we should hence have better measures (statistics) to understand the relation between
economic well being and environment protection. Option 1 states this explicitly.
Option 2 is only one sided.
Option 3 mentions demand and is ambiguous and outside the scope of the argument.
Option 4 generalises and misses the thrust of this argument.
Option 5 is contrary to the argument.
Hence, the correct answer is option 1.
Discuss the solution with Testfunda users.

Solution #221: (19-Nov-09)


From statement A alone,
If p = 4 then 4! < x < 5!
24 < x < 120
In the given range, there are prime as well as composite numbers.
Statement A alone is not sufficient.
From statement B alone,
We know that p! is divisible by 1, 2, 3, ..., p.
p! + 2 will be divisible by 2, p! + 3 will be divisible by 3, ..., p! + p will be divisible by p.
As, p! + 2 x p! + p, x is divisible by 2 or 3 or 4 or 5 ... or p.
x will not be a prime number.
Statement B alone is sufficient.
Hence, option 2.
Discuss the solution with Testfunda users.

Solution #222: (20-Nov-09)


The main idea is: puppies learn to respond to touch based on whether the mother has been petted
during her pregnancy. The concluding statement about the puppys sense of touch would depend on
this information.
In option 2 can be eliminated as there is no supporting data for puppies behaving in an intolerant
manner if they have had a cruel time in utero. The passage mainly concerns itself with the
consequences of positive touches and there is no mention of the effect of negative touches.
Option 3 can be eliminated as there no evidence of the exponential nature of the relationships.
Option 4 is eliminated as there is no data on the development of other senses.
Option 5 is eliminated as there is no data on mother's lifestyle.
Option 1 fit in as a conclusion statement as it has been established in the passage that petting the
mother leads to docile puppies that tolerate touching.
Hence, the correct answer is option 1.
Discuss the solution with Testfunda users.
Book 2- Question of the Day

287

www.TestFunda.com

The LEADER in Online Test Prep

Solution #223: (21-Nov-09)

Rationalising each term of X we get,

= 13
Hence, option 5.
Discuss the solution with Testfunda users.

Solution #224: (22-Nov-09)


The main idea is: osteoporosis is caused in women due to lack of calcium- men can be affected
similarly. The assumption we need to find is a statement that shows how it is possible to apply what
applies to women to the men as well.
We can rule out Option 1 as the author has concluded (which is why) that men should check the
level of calcium, based on the data provided about early onset of osteoporosis in women.
Option 2 is ruled out as we have to find a statement that finds a specific relation between men and
women.
Option 3 is ruled out as it does not refer to why men will get osteoporosis as well.
Option 5 is ruled out as it does not show how what has been found out about women is made to
apply to men.
Only if the author assumes that Since low calcium causes osteoporosis in women, it will have a
similar effect on men too the argument presented in main data holds true..
Hence, the correct answer is option 4.
Discuss the solution with Testfunda users.

Book 2- Question of the Day

288

www.TestFunda.com

The LEADER in Online Test Prep

Solution #225: (23-Nov-09)


We have that,
An + 1 = An2 + 3An + 1
An + 1 + 1 = An2 + 3An + 2
An + 1 + 1 = (An + 1)(An + 2)

Rearranging, we get,

Also by the definition of An, A201 > 0


In addition, S > 0, since it is the sum of positive terms.

The least integer greater than S = 1


Hence, option 1.
Discuss the solution with Testfunda users.
Book 2- Question of the Day

289

www.TestFunda.com

The LEADER in Online Test Prep

Solution #226: (24-Nov-09)


The main argument presented in this passage is that the fathers' rights movement say that domestic
violence studies based on the CTS show that men comprise a "significant portion" of the victims of
domestic violence.
Option 2 talks about shelter - whereas the argument is about violence - the connection is indirect
rather than direct.
Option 3 only offers an elaboration of the fact provided in the passage, they call for more services to
be provided for male victims of domestic violence.
Option 4 is a feeling and does not strengthen the argument.
Option 5 is ruled out as it does not indicate if the victim is the husband - if it were then it would
strengthen the argument that men comprise a significant portion of the victims of domestic
violence.
If there are more men who are victims of domestic violence but arent able to talk about it, then it
means that their number will add to the already significant portion of the victims of domestic
violence.
Hence, the correct answer is option 1.
Discuss the solution with Testfunda users.

Solution #227: (25-Nov-09)


The given data can be represented in the following manner.

Let G be the point of intersection of the medians.


G is the centroid of ABC.
G divides both the medians AM and CN in the ratio 2 : 1.
Since CN = 6 cm, therefore, CG = 4 cm and GN = 2 cm
Also, since AM = 9 cm, we have AG = 6 cm and GM = 3 cm
Let GAC = and NAG =
From GAC, we get that,

Book 2- Question of the Day

290

www.TestFunda.com

The LEADER in Online Test Prep

From NAG, we get that,

Now, in BAC, A = +
tan A = tan ( + )

Hence, option 5.
Discuss the solution with Testfunda users.

Solution #228: (26-Nov-09)


The main idea in the passage is that higher order primates have a sense of reciprocity and fairness.
The example given to support this idea is that of chimpanzees who remember who they are
obligated towards and to whom they are not. To further strengthen this argument we need a
statement that elaborates this idea or example.
Options 3 and 4 introduce new ideas that do not support the argument. Options 3 introduces
morality into the picture. Option 4 introduces the idea of group behaviour.
Option 5 talks about social order in the "ancestors of the human population", but we do not know
how these are related to the social animals and the higher order primates discussed in the passage.
Option 2 weakly supports the argument, but is eliminated compared to option 1, which supports it
much more directly.
Option 1 elaborates the example given in the passage, that of the chimpanzees. It explains the
concept of chimpanzees being able to remember who has done them a favour since the statement
chimpanzees are likely to share food with individuals who have done them the favour of grooming
them.
Hence, the correct answer is option 1.
Discuss the solution with Testfunda users.

Book 2- Question of the Day

291

www.TestFunda.com

The LEADER in Online Test Prep

Solution #229: (27-Nov-09)


We look for integer values of a, b and c such that a3 + b3 + c3 = 3abc
This equation is satisfied by (a, b, c) (1, 1, 1)
d=1+1+1=3

Hence, option 2.
Alternatively,
(a + b + c)3 = a3 + b3 + c3 + 3a2b + 3ab2 + 3b2c + 3bc2 + 3a2c + 3ac2 + 6abc
As a3 + b3 + c3 = 3abc
d3 = 3abc + 3a2b + 3ab2 + 3b2c + 3bc2 + 3a2c + 3ac2 + 6abc
d3 = 3a2b + 3ab2 + 3abc + 3b2c + 3bc2 + 3abc + 3a2c + 3ac2 + 3abc
d3 =3ab(a + b + c) + 3bc(a + b + c) + 3ac(a + b + c)
d3 = 3(a + b + c)(ab + bc + ac)
As (a + b + c) = d
d3 = 3d(ab + bc + ac)
d2 = 3(ab + bc + ac)
(a + b + c)2 = a2 + b2 + c2 + 2(ab + bc + ac)

Hence, option 2.
Discuss the solution with Testfunda users.
Book 2- Question of the Day

292

www.TestFunda.com

The LEADER in Online Test Prep

Solution #230: (28-Nov-09)


Frazers theory states that at the start man had an unfounded belief in magic, then he started
believing that gods controlled nature, thereafter he starts thinking about how science can be used to
discover the nature of the natural phenomenon. We have to find a statement that sums this up.
Option 2 is ruled out as there is no mention of man abandoning myth.
Option 3 is ruled out as there is no indication that myth is a counterpart of science.
Option 4 is not the answer as there is no reference to a historical development of the primitive
mentality
Option 5 is also not the answer since there is no mention of how language influenced mythology.
Option 1 is the answer as it sums up the progression of ideas: from magic to religion to science.
Hence, the correct answer is option 1.
Discuss the solution with Testfunda users.

Solution #231: (29-Nov-09)


In the prime numbers between 100 and 200, which are 101, 103, 107, 113 etc, there is only one with
first two digits same, namely, 113.
But 133 is not prime.
Now, between 221 and 229 there are 3 primes 223, 227 and 229.
Also, 233 is a prime number, therefore, the smallest number satisfying the given condition is 2233.
Sum of digits of the required number = 10
Hence, option 2.
Discuss the solution with Testfunda users.

Solution #232: (30-Nov-09)


The main argument in the passage is that men should have as much say in the decision-making
process when it comes to a pregnancy as women have.
Option 3 sums up the argument presented in the passage and give men the same reproductive
rights as women - hence the author is most likely to agree with it.
There is no data in the passage to make us support option - rather the author is likely to disagree
with the option.
Options 2 and 5 are too generalised as statement to be chosen.
Option 4 is opposite of what the passage states as fact.
Option 5 is ruled out as this is a generalised statement that isnt supported by any data in the
passage.
Hence the correct answer is option 3.
Discuss the solution with Testfunda users.

Book 2- Question of the Day

293

www.TestFunda.com

The LEADER in Online Test Prep

Solution #233: (01-Dec-09)


Given that the roots of x2 + 3px + 36 = 0 are real.
The discriminant of the equation will be either positive or zero, i.e., 0
(3p)2 4 36 0
(3p)2 144 0
(3p)2 122 0
(3p + 12)(3p 12) 0
i.e. p (, 4] *4, )
But, since p > 0, we have
p *4, ) i.e. p 4

(i)

Also, the roots of the equation x2 4x + p = 0 are real.


0
42 4p 0
4p 16 0
p4

... (ii)

From (i) and (ii), it is evident that p = 4


Only one value of p exists such that both the equations have real roots.
Hence, option 2.
Discuss the solution with Testfunda users.

Solution #234: (02-Dec-09)


Option 1 talks about computer viruses which is not pertinent to the world wide web of microbes.
Similarly, option 2 is correct only if we are considering the comparison between microbes and
computer viruses not when considering the www of microbes. There is no data in the passage to
support this option.
Option 3 is irrelevant as it talks about the origin of certain aspects of human genome.
Option 4 is factually correct but is not the assumption in this argument there are no details to
support this option.
Option 5, the answer, is almost stated in the argument as, informational exchange and microbes
exchange information with each other and their environment other organisms in the option
sufficiently accommodates each other and environment.
Hence, the correct answer is option 5.
Discuss the solution with Testfunda users.
Book 2- Question of the Day

294

www.TestFunda.com

The LEADER in Online Test Prep

Solution #235: (03-Dec-09)


Let the amount invested by Mahesh be P, and the interest is compounded annually at the rate of r%.
We know that the Compound Interest for the 5th year is Rs. 2,000.

And similarly we know that the Compound Interest for the 11th year is Rs. 5,000.

Dividing (ii) by (i) we have

= 5000 32
= Rs. 1,60,000
Hence, option 3.
Discuss the solution with Testfunda users.

Book 2- Question of the Day

295

www.TestFunda.com

The LEADER in Online Test Prep

Solution #236: (04-Dec-09)


... many European intellectuals argue that not just capital punishment, but punishment in general,
does not deter criminals. In order to weaken this argument we have to say that punishment does
deter crime. Hence the decrease in the crime rate in the US will show that the existence of capital
punishment (harsh punishments) have had an effect on the crime rate. Similarly the increase in the
rates in Europe also will prove that lack of punishment has had an effect. This weakens the argument
of the European intellectuals.
Option 1 is incorrect as it supports the argument of the intellectuals.
Option 3 also strengthens the intellectuals view.
Option 4 and 5 do not address the link between punishment and crime; hence, they are eliminated.
Hence, the correct answer is option 2.
Discuss the solution with Testfunda users.

Solution #237: (05-Dec-09)


Let log5(3x2 + 10x) = p

p2 5p + 6 = 0
(p 2)(p 3) = 0
p = 2 or 3
Case 1: p = 2
log5(3x2 + 10x) = 2
i.e. 3x2 + 10x = 52
3x2 + 10x 25 = 0
3x2 + 15x 5x 25 = 0
3x(x + 5) 5(x + 5) = 0
(x + 5)(3x 5) = 0

For both these values of x, log5(3x2 + 10x) is well defined.


Case 2 : p = 3
log5(3x2 + 10x) = 3
i.e. 3x2 + 10x = 53
3x2 + 10x 125 = 0
3x2 15x + 25x 125 = 0
3x(x 5) + 25(x 5) = 0
(3x + 25)(x 5) = 0

For both these values of x, log5(3x2 + 10x) is well defined.

Hence, option 1.
Discuss the solution with Testfunda users.
Book 2- Question of the Day

296

www.TestFunda.com

The LEADER in Online Test Prep

Solution #238: (06-Dec-09)


Only if option 4 is true can corruption be reduced by cutting back the role of government/
government officials, since according to the argument the source of corruption.
Option 1 is incorrect because the opposite is correct deregulation decreases corruption.
How regulation affects government officials (option 2), or if does at all is assumed in the argument.
Since it is deregulation that reduces corruption option 3 is incorrect.
Option 5 is beyond the scope of the argument.
Hence, the correct answer is option 4.
Discuss the solution with Testfunda users.

Solution #239: (07-Dec-09)

The minimum value of y = 2 + 1 = 3


This minimum value is attained when (x 2) = 1 i.e. x = 3
But, we have
15x2 214x + 455 0
15x2 175x 39x + 455 0
5x(3x 35) 13(3x 35) 0
(3x 35)(5x 13) 0

Hence, the minimum value of y obtained as 3 by taking x 2 = 1 i.e. x = 3 is not valid.


Book 2- Question of the Day

297

www.TestFunda.com

The LEADER in Online Test Prep

Hence, option 2.
Discuss the solution with Testfunda users.

Solution #240: (08-Dec-09)


Option 1 is eliminated because it is possible that the policy may work in the US and Eastern Europe.
Option 2 is eliminated because we cannot infer that the policies are ineffective simply because they
have caused increase in unemployment social welfare policy per se may be quite effective.
Option 3 would be said about Western Europe there is no data to conclude this about Eastern
Europe.
Option 4 is also not conclusive. Simply because some policies are in place it is not possible to
comment on the general health of a nation. Even the reverse could be true.
Hence the answer is option 5.
Discuss the solution with Testfunda users.

Book 2- Question of the Day

298

www.TestFunda.com

The LEADER in Online Test Prep

Solution #241: (09-Dec-09)


Selling price per unit = 2(80 n)
Total revenue = n 2(80 n) = 2n(80 n)
P(n) = Total profit earned per day = 2n(80 n) *n2 + 10n + 363]
= 150n 3n2 363
Profit earned in a week = 7 P(n)
To maximize the weekly profit earned we have to maximize P(n).

By the second derivative test P(n) is maximum at n = 25.


The factory should produce 7 25 = 175 units of tube lights per week to maximize its weekly
profits.
Hence, option 2.
Discuss the solution with Testfunda users.

Solution #242: (10-Dec-09)


This analogy is based on the secondary meaning of the word habit. A nun's habit refers to the garb
that is commonly worn by her.
The only option that comes close to this is option 3. A uniform is commonly worn by a soldier.
Hence, the correct answer is option 3.
Discuss the solution with Testfunda users.

Book 2- Question of the Day

299

www.TestFunda.com

The LEADER in Online Test Prep

Solution #243 (11-Dec-09)

By LHospitals rule,

Applying L Hospitals rule again,

Hence, option 1.
Discuss the solution with Testfunda users.

Book 2- Question of the Day

300

www.TestFunda.com

The LEADER in Online Test Prep

Solution #244: (12-Dec-09)


The key to solving these type of questions is to first place the phrases in their correct logical order to
understand the flow of the sentence.
The correct logical order here is 4 - 3 - 2 - 5 - 1. Reading the sentence in its correct logical order
makes it clear that the incorrect phrase lies in option 3.
That something can be the volumetric region surrounding a single atom resonating energy or it can
be a body of steam or air in a steam engine or it can be the body of a tropical cyclone or it could also
be just one nuclide (i.e. a system of quarks) as some are theorizing presently in quantum
thermodynamics.
The word, surrounding is a gerund (a verb - noun). The correct form of a gerund is word + ing.
Hence, the correct answer is option 3.
NOTE: A gerund is defined as that form of the verb which ends in - ing, and has the force of a Noun
and a Verb.
For example- He is fond of singing songs from old Hindi movies.
Discuss the solution with Testfunda users.

Solution #245: (13-Dec-09)


We have to look at each possible case, and see whether we get a contradiction.
If A is of type T, B must be of type F, according to his statement.
However, then what B says is true, and this is a contradiction because B is of type F. Hence, A cannot
be of type T.
Since we know that A is of type F, B is of type T by his statement since one of them is of type T and
one of type F.
This assumption does not give a contradiction, therefore, A is of type F and B is of type T.
Hence, option 4.
Discuss the solution with Testfunda users.

Solution #246: (14-Dec-09)


A Slap on the Wrist: A very mild punishment.
A Chip On Your Shoulder: Someone who is troublesome to deal with is said to have a "chip on his
shoulder".
Apple of My Eye: Someone who is cherished above all others.
Bite Your Tongue: To avoid talking.
Break A Leg: A superstitious way to say 'good luck' without saying 'good luck', but rather the
opposite.
Hence, the correct answer is option 4
Discuss the solution with Testfunda users.

Book 2- Question of the Day

301

www.TestFunda.com

The LEADER in Online Test Prep

Solution #247: (15-Dec-09)


Since each sport is played by a different number of people, we need to assign the numbers 1, 2 and
3 in some order to cricket (C), football (F) and hockey (H) corresponding to the number of students
who play that game.
F must correspond to either 2 or 3. Since Amar does not play hockey, H must correspond to either 1
or 2.Since, not all play cricket, C must correspond to either 1 or 2.
From these, we can see that if C = 1, then H = 2 and F = 3
If C = 2, then H = 1 and F = 3
In either case, all three play football.
Since Akbar plays only one sport, he must play football.
Akbar does not play hockey or cricket.
We know that Amar does not play hockey. Therefore, as each sport is played by at least one person,
Anthony plays hockey.
H = 1 and C = 2
Also, as Akbar plays one sport and Cricket is played by 2 people, Amar and Anthony play cricket.
Hence, option 3.
Discuss the solution with Testfunda users.

Solution #248: (16-Dec-09)


Mum's the word means to keep quiet or to say nothing.
Elvis has left the building means the show has come to an end. It's all over.
Pipe Down means to shut-up or be quiet.
Can't cut the mustard means someone who isn't adequate enough to compete or participate.
Let bygones be bygones means to forget about a disagreement or argument.
Options 1 and 3 both convey the same idea.
Hence, the correct answer is option 5.
Discuss the solution with Testfunda users.

Book 2- Question of the Day

302

www.TestFunda.com

The LEADER in Online Test Prep

Solution #249: (17-Dec-09)


First, we need to determine who did better between Laloo 4 and Laloo 5.
At least two candidates must have done better than Laloo 5, since Laloo 2 finished ahead of him, and
there is at least one person ahead of Laloo 2.
If Laloo 5 finished ahead of Laloo 4, Laloo 4 must have finished sixth, since there are two people
between Laloo 4 and Laloo 5.
However, this is not possible, since Laloo 1 did worse than Laloo 4.
Hence, Laloo 4 must have finished ahead of Laloo 5.
Of the two people between Laloo 4 and Laloo 5, one must be Laloo 2, since he is separated from
Laloo 5 by one candidate only.
Since Laloo 6 is ahead of Laloo 4 and Laloo 1 is behind Laloo 5, the other person between Laloo 4
and Laloo 5 must be Laloo 3.
The final order is therefore Laloo 6, Laloo 4, Laloo 2, Laloo 3, Laloo 5 and Laloo 1.
Hence, option 4.
Discuss the solution with Testfunda users.

Solution #250: (18-Dec-09)


In the sentence, The students are coming home by bus. the word come is used to show that the
students are arriving by movement.
In the sentence, May comes after April the word come is used to show that something occurs at a
certain point.
In the sentence, The police ensured the tourists that no harm would come to them the word come
is used to highlight what did not befall someone.
In the sentence, She comes across as quite an intelligent girl the word come is used to show the
kind of impression that the girl creates.
In the sentence, My family comes first the word come is used to indicate priority.
Hence, the correct answer is option 2
Discuss the solution with Testfunda users.

Book 2- Question of the Day

303

www.TestFunda.com

The LEADER in Online Test Prep

Solution #251: (19-Dec-09)


Let the length, breadth and height of the box be l, b and h respectively.
Total internal Surface Area = S = 2(lb + bh + lh) lb = lb + 2bh + 2lh
To find the maximum value of volume for this surface area, we use AM GM,

Now, we know that volume of the box, V = lbh


S 3 22/3 V2/3

Substituting S = 12 cm2, we get, V 4 cm3


Hence, option 1.

Alternatively,
The product of n numbers when their sum is known is maximum when they are equal.
lb 2bh 2lh is maximum when lb = 2bh = 2lh
l = b = 2h
12h2 = 12
h=1
l=b=2
The maximum volume = 2 2 1 = 4 cm3
Hence, option 1.

Discuss the solution with Testfunda users.

Book 2- Question of the Day

304

www.TestFunda.com

The LEADER in Online Test Prep

Solution #252: (20-Dec-09)


A metaphor is a comparison between two entities without the use of the words like or as.
In option 4, opportunity is the currency of the American dream is a metaphor since two different
entities, namely opportunity and currency are being compared without the use of the word like or
as.
The other options do not have metaphors.
Hence, the correct answer is option 4.

Discuss the solution with Testfunda users.

Solution #253: (21-Dec-09)


By the options we can conclude that we have to examine the continuity of this function at x = 0 and
x = 1.
We can do this by finding the limits of the functions slightly above and below each of these points.
f(x) = |x 1| + |x + 1|

For x = 0

f(x) is continuous at x = 0
For x = 1

f(x) is continuous at x = 1.
f(x) is continuous at both x = 0 and 1.
Hence, option 1.

Discuss the solution with Testfunda users.

Book 2- Question of the Day

305

www.TestFunda.com

The LEADER in Online Test Prep

Solution #254: (22-Dec-09)


A metonymy is a figure of speech in which a word or phrase is substituted for another with which it
is closely associated. It is a play of words based on common association.
In option 5, a man of cloth is a metonymy since it is used to denote that the chaplain is a priest. It
is used as a substitute of one word for another which it suggests.
The other options do not have an example of metonymy.
Hence, the correct answer is option 5.

Discuss the solution with Testfunda users.

Solution #255: (23-Dec-09)

Six standard deviations = 0.84 m


Upper limit of the length of the wire = 0.96 + 0.84 = 1.8 m
Lower limit of the length of the wire = 0.96 0.84 = 0.12 m
As 0.12 < 0.7, 1.7 < 1.8 sample A and B both will be accepted.
Hence, option 1.
Discuss the solution with Testfunda users.

Book 2- Question of the Day

306

www.TestFunda.com

The LEADER in Online Test Prep

Solution #256: (24-Dec-09)


Tautology is the needless repetition of the same idea in different words.
In option 2, undecideds could go one way or another is a tautology as it is a needless repetition
conveying the same idea.
The other options do not have tautology.
Hence, the correct answer is option 2.

Discuss the solution with Testfunda users.

Solution #257: (25-Dec-09)


We know that all the diagonal elements of a skew-symmetric matrix must be zero, since they are
equal to their own negatives. We can make all the diagonal elements of C zero if we choose matrix A
such that all its diagonal elements are zero.
Now, consider any non-diagonal term of A. If we choose random values for the non-diagonal terms
of A, we can choose a maximum of three non-diagonal terms of B such that the sum of the
corresponding terms of A and B is zero. This is true because, if we choose one corresponding term of
B to be the negative of A, we are fixing another term of B to equal that term of A, since BT = B. After
choosing three non-diagonal terms of B to cancel out the corresponding non-diagonal terms of A, we
have fixed the other three terms to equal the corresponding terms of A, and these will not cancel
out .
Hence, we can have a maximum of three zeros in the diagonal elements and three zeros in the non
diagonal elements in matrix C, making the maximum possible number of zeros equal to 6.
The matrices will look like follows.

where a, b and c are real numbers.


x=6
Hence, option 3.
Discuss the solution with Testfunda users.

Book 2- Question of the Day

307

www.TestFunda.com

The LEADER in Online Test Prep

Solution #258: (26-Dec-09)


Transferred epithet is a figure of speech in which the epithet (an adjective) which modifies the noun
is transferred from that noun (where it fits in logically) to another (where it fits in grammatical but
not logically).
In option 5, cruel bars of prison is a transferred epithet. The adjective cruel has been transferred
from the person to the bars of prison. The bars themselves are not cruel, but they serve the
purposes of the cruel person who uses the cage to imprison someone.
The other options do not have transferred epithets.
Hence, the correct answer is option 5.
Discuss the solution with Testfunda users.

Solution #259: (27-Dec-09)


The events are mutually exclusive. So each probability must lie between 0 and 1.

Hence, option 2.

Discuss the solution with Testfunda users.

Solution #260: (28-Dec-09)


Disinterest means: impartial.
Uninterested means: one who is not interested.
Appraise means: to evaluate something.
Apprise means: to inform or tell.
The sentence should read, "His uninterested attitude when apprised about the change in policy
does not mean that he would remain disinterested when it was his turn to be one who appraised the
performance of others".
Hence, the correct answer is option 4.

Discuss the solution with Testfunda users.

Book 2- Question of the Day

308

www.TestFunda.com

The LEADER in Online Test Prep

Solution #261: (29-Dec-09)


In 10 days, 1/4th of the substance disappears and 3/4th of the substance is left.
It is given that at any given point of time the amount of substance disappearing is directly
proportional to the amount of substance present at that moment.
In another 10 days 1/4th of the substance left will disappear, therefore, 3/16 of the substance will
disappear.
Total amount of substance disappeared in 20 days = 1/4 + 3/16 = 7/16 < 8/16 = 1/2
Number of days taken for half the substance to disappear is more than 20 days.
We can eliminate options 1 and 3.
Amount of substance left after 20 days = 3/4 3/16 = 9/16
In another 10 days, the amount of substance that will disappear = 1/4 Amount of substance left
after 20 days
= 9/64
Total amount of substance that has disappeared in 30 days = 7/16 + 9/64 = 37/64 > 32/64 = 1/2
It will take less than 30 days for half of the substance to disappear (or for half of the substance to
be left).
We can eliminate option 4.
Hence, option 2.

Alternatively,
Let A be the amount of substance left at any time t.
Since the rate of decay of the radioactive substance is proportional to the amount of radioactive
substance left.
We have,
dA/dt = kA, where k is constant.
dA/A = kdt
Integrating both the sides we get,
ln(A) = kt + c

Book 2- Question of the Day

309

www.TestFunda.com

The LEADER in Online Test Prep

Let the initial amount be A0. Then at t = 0,


c = ln(A0)
ln(A) = kt + ln(A0)
ln(A/A0) = kt
A = A0ekt

(i)

By the condition given in the question, we have that,


At t = 10 days, we have, A = 0.75A0, substituting the given values in (i), we get,
e10k = 0.75
Taking the natural logarithm of both sides, we get,
10k = ln(0.75) = 0.29
k = 0.029

(ii)

For A = 0.5K, using (i) and (ii), we get


ekt = 0.5
Taking the natural logarithm of both sides, we get,
0.029t = ln(0.5) = 0.693
t 693/29 23.8 days
Hence, option 2.

Discuss the solution with Testfunda users.

Solution #262: (30-Dec-09)


Allude means: to refer to something indirectly.
Elude means: to escape or avoid.
'Amount' is used for uncountable nouns.
'Number' is used for countable nouns.
The sentence would read: "He eluded prosecution as the number of witnesses dropped dramatically.
It was alluded to by the Defence that the amount of files generated during his court case led to
complete chaos".
Hence, the correct answer is option 1.

Discuss the solution with Testfunda users.

Book 2- Question of the Day

310

www.TestFunda.com

The LEADER in Online Test Prep

Solution #263: (31-Dec-09)


We can see that the total number of crimes committed = 93 + 76 + 62 + 81 = 312
Let a, b, c and d be the number of people who report 1, 2, 3 and 4 crimes respectively.
Then a + b + c + d = 100
Also, a + 2b + 3c + 4d = 312
If we assume that 100 people suffered at least each of the first 3 types of crime, then the number of
people suffering from the fourth would be minimum.
In order to minimise d, we assume a and b to be zero so that
c + d = 100 and 3c + 4d = 312
Solving these two equations simultaneously,
d = 12
If only 12 people suffer all four crimes, the money to be paid to them = 12 100 = Rs. 1200
Hence, option 1.

Discuss the solution with Testfunda users.

Solution #264: (01-Jan-10)


Climactic is something that constitutes a climax.
Climatic means: related to the climate.
Compliment means: best wishes or admiration.
Complement means: a thing that adds to, completes something, or makes it perfect.
The sentence should read, "The climactic end of the movie was complimented by one and all. The
last scene complemented the social message regarding the climatic changes to be faced in the
future".
Hence, the correct answer is option 3

Discuss the solution with Testfunda users.

Book 2- Question of the Day

311

www.TestFunda.com

The LEADER in Online Test Prep

Solution #265: (02-Jan-10)


Since B = 0, therefore cos B = 1
Also, since A, B and C are in AP, therefore, A = B d and C = B + d, where d is the common difference
of the AP.
A = d and C = d
A = C
Now, cos (C) = cos C = cos A
cos A = cos C
X = cos A cos B cos C = cos2 A
Hence, option 4.

Discuss the solution with Testfunda users.

Solution #266: (03-Jan-10)


"A cash cow" means an easy way to earn money regularly.
"Cat-and-dog life" refers to a life in which partners are constantly or frequently quarreling.
"A fat cat" means a wealthy person.
"For the birds" means something is uninteresting, useless, or not to be taken seriously.
Hence, the correct answer is option 3.

Discuss the solution with Testfunda users.

Book 2- Question of the Day

312

www.TestFunda.com

The LEADER in Online Test Prep

Solution #267: (04-Jan-10)


Total number of elements in set S should be 200 if no terms are common to both the sequences.
Writing the first few elements of each progression, we can see that the first common element is 26.
Also, the LCM of the common differences of the two progressions = 4 7 = 28
Therefore, the common elements in each progression will be 26, 26 + 28, 26 + 56, and so on.
The 100th term of the first sequence = 2 + 99 4 = 398
We need to find the number of common terms less than 398, since the first sequence has a smaller
100th term.
We have to find the largest k such that 26 + 28k < 398.
k = 13
Therefore, there are 14 common terms, and the size or the cardinal number of set S = 200 14 = 186
Hence, option 3.

Discuss the solution with Testfunda users.

Solution #268: (05-Jan-10)


Option 1 is incorrect as the advice given really isn't to manage unruly rather that is a by-product.
Option 3 is incorrect because it goes beyond the scope of the quotation.
Option 4 is incorrect because it doesn't accurately cover what the quotation is saying. The quotation
is not only talking about choices but rather a general rule.
Option 2 states that the author is referring a way to ensure a favorable or prosperous conclusion of
one's endeavours.
Hence, the correct answer is option 2.

Discuss the solution with Testfunda users.

Book 2- Question of the Day

313

www.TestFunda.com

The LEADER in Online Test Prep

Solution #269: (06-Jan-10)


We have,
an + 1 = 2a12 + a22 + a32 + ... + an 12 + an2
an = 2a12 + a22 + a32 + ... + an 12
an + 1 = an + an2
Since all terms of this sequence are positive, we have an2 < an + 1.
Also, an + 1 = an + an2 < an2 + 2an + 1 = (an + 1)2
an2 < an + 1 < (an + 1)2
an + 1 is an integer that lies between two consecutive perfect squares.
Obviously, no integer that lies between two consecutive perfect squares can itself be a perfect
square.
The only member of the sequence that is a perfect square is a1.
Hence, option 2.

Discuss the solution with Testfunda users.

Solution #270: (07-Jan-10)


Below is the list of collective nouns (along with usage):
A crew of technicians
A troupe of performers
A battery of tests
A team of advisors
A band of thieves
A clique of directors
A coterie of prairie dogs
A skein of geese (in flight)
A wad of money
A bevy of larks
A gang of people
Light is not a collective noun.
Hence, the correct answer is option 4.

Discuss the solution with Testfunda users.

Book 2- Question of the Day

314

www.TestFunda.com

The LEADER in Online Test Prep

Solution #271: (08-Jan-10)

A + (x + y + z)

Hence, A 0.5 (x + y + z) for all values of x, y and z.


We can see that P = 0.5
Hence, option 2.
Discuss the solution with Testfunda users.

Book 2- Question of the Day

315

www.TestFunda.com

The LEADER in Online Test Prep

Solution #272: (09-Jan-10)


Option 1 is incorrect as this conclusion is neither implied nor stated.
Options 2 and 4 are partially correct, but they do not refer to the quotation as a whole, but rather a
part of the quotation.
Option 3 is the best among the available options.
Hence, the correct answer is option 3.

Discuss the solution with Testfunda users.

Solution #273: (10-Jan-10)


For n = 1, k2 = 1
For n = 2, k2 = 3
For n = 3, k2 = 9
For n = 4, k2 = 33
For n = 5 and above, n! ends with a zero.
The last digit of (1! + 2! + ... + n!) for n > 4 will be the same as the last digit of (1! + 2! + 3! + 4!).
The last digit of (1! + 2! + 3! + 4!) is 3.
There is no integer whose square ends in 3.
There is no solution for this equation for n > 4.
The only solutions are (n, k) = (1, 1) and (3, 3)
Hence, option 3.

Discuss the solution with Testfunda users.

Book 2- Question of the Day

316

www.TestFunda.com

The LEADER in Online Test Prep

Solution #274: (11-Jan-10)


Apartheid is the abhorrent policy of racial segregation and oppression once practiced in the
Republic of South Africa. Apartheid is a form of prejudice.
Antipathy means from dislike; a dislike
Exacerbate means to infuriate. Soothe means calm or ease.
Hadephobia is the fear of hell, not death.
Guerrilla is a form of warfare.
Hence, the correct answer is option 4.

Discuss the solution with Testfunda users.

Solution #275: (12-Jan-10)


Consider option 1.
yz xw = y (x + w y) xw = y (w y) x (w y) = (y x)(w y)
Now, x > y > z > w > 0, so (y x) and (w y) are both negative, and their product is positive.
yz xw > 0
xw < yz
Option 2 is correct.
Consider option 3.
We know that, x > y > z > w and x, y, z, w are positive numbers.
xy > wz
Option 3 is also correct.
Hence, option 5.

Discuss the solution with Testfunda users.

Book 2- Question of the Day

317

www.TestFunda.com

The LEADER in Online Test Prep

Solution #276: (13-Jan-10)


Umbrage means 'personal displeasure'. The relationship is that of opposites.
Option 1 is eliminated as chagrin and peeve both mean something causing displeasure.
Option 2 is eliminated as pique and vexation both mean irritation.
Option 3 is incorrect as perturb and disquiet both mean unsettling.
Nettle means to to irritate, annoy, or provoke. While, conciliate means to placate.
Hence, the correct answer is option 4.

Discuss the solution with Testfunda users.

Solution #277: (14-Jan-10)


If n6 + 206 is divisible by n2 + 2 then

By inspection we can see that n can be 1, 2, 3, 4, 8 and 14.


There are 6 values of n.
Hence, option 2.

Discuss the solution with Testfunda users.

Book 2- Question of the Day

318

www.TestFunda.com

The LEADER in Online Test Prep

Solution #278: (15-Jan-10)


Options 1 and 2 are incorrect as 'symbiotic' in blank 2 means a mutually beneficial relationship
between two persons/ groups. This does not fit in with the meaning of the sentences. The correct
usage is 'symbolic' which means characterized by or involving the use of symbols
Option 4 is incorrect because the first blank requires the word, 'directly'. 'Indirectly' does not fit in as
it clashes with the word, 'correspond'.
Hence, the correct answer is option 3.
Discuss the solution with Testfunda users.

Solution #279: (16-Jan-10)


If a number n can be expressed as pa qb rc, where p, q and r are prime, then it has (a + 1)(b +
1)(c + 1) factors.
As the required number has 18 factors, it can be expressed in the form
p17 or (p2 q5) or (p1 q8) or (p1 q2 r2)
As 2, 3 and 5 are the smallest prime numbers, we will consider p, q and r to be 2, 3 and 5 in some
order.
The number could be 217, 22 35, 25 32, 21 38, 31 28, 21 32 52, 31 22 52, 51 22 32 and so
on.
But as the number leaves a remainder of 9 when divided by 31, we find that the number could only
be 25 32 = 288.
The sum of digits of 288 is 18.
Hence, option 1.

Alternatively,
25 = 32 gives a remainder 1 on division by 31 and thus 25 9 gives a remainder 9 on division by 31.
We check that the number of divisors is 18.
Hence, option 1.

Discuss the solution with Testfunda users.

Book 2- Question of the Day

319

www.TestFunda.com

The LEADER in Online Test Prep

Solution #280: (17-Jan-10)


Hydrocephalus is a condition in which dilated cerebral ventricles inhibit normal flow of cerebrospinal
fluid. The fluid accumulates in the skull and puts pressure on the brain tissue. Cholecystectomy is the
surgical removal of the gallbladder. It is the most common method for treating symptomatic
gallstones.
The relationship is not analogous.
Atherosclerosis is the condition in which an artery wall thickens as the result of a build-up of fatty
materials such as cholesterol. Angioplasty is the technique of mechanically widening a narrowed or
obstructed blood vessel; typically as a result of atherosclerosis. This option is eliminated as the
relation between the two is analogous.
Hepatectomy consists of the surgical resection of the liver. Hepatectomies are surgeries performed
for the treatment of hepatic neoplasms. Neoplasm is an abnormal mass of tissue - an abnormal
proliferation of cells.
Rheumatoid arthritis is a chronic, systemic inflammatory disorder that may affect many tissues and
organs, but principally attacks the joints producing an inflammation that often progresses to
destruction of the cartilage and ankylosis of the joints. Arthroplasty is the surgical repair of joint. This
option is also eliminated as the relationship is analogous.
A hernia is protrusion of an organ or the muscular wall of an organ through the cavity that normally
contains it. Tracheotomyis a surgical procedure on the neck to open a direct airway through an
incision in the trachea (the windpipe). This relationship is not analogous.
Hence, the correct answer is option 4.

Discuss the solution with Testfunda users.

Solution #281: (18-Jan-10)


There are three ways to flip coins.
Case 1: You flip two tails. You lose two tails, and gain two heads.
Case 2: You flip one tail and one head. The tail changes to head and head changes to tail. Net effect
is null.
Case 3: You flip two heads. You gain two tails and lose two heads.
As the number of coins is 7, which is odd, no number of flips can change all tails to heads.
Hence, option 5.
Discuss the solution with Testfunda users.

Book 2- Question of the Day

320

www.TestFunda.com

The LEADER in Online Test Prep

Solution #282: (19-Jan-10)


Option 1 is incorrect because it is incorrect usage. 'Though' has been incorrectly used.
Option 2 is incorrect as 'high performance within parallelism' is not possible.
Option 3 is incorrect because the sentence is incomplete using 'as'.
Option 5 is incorrect as there is no contradiction in the sentence. Therefore, 'yet' is incorrectly used.
Hence, the correct answer is option 4.

Discuss the solution with Testfunda users.

Solution #283: (20-Jan-10)

We expand the other terms of A in the same way to get

Regrouping, we get

A = 22B

Hence, option 3.

Discuss the solution with Testfunda users.

Book 2- Question of the Day

321

www.TestFunda.com

The LEADER in Online Test Prep

Solution #284: (21-Jan-10)


Option 4 is incorrect as blanks 2 and 3 are incorrect. A CIO cannot be 'indicative'.
Options 2 and 3 are incorrect as the first blank cannot be 'secondary'. If the CIO is the primary
representative for a task then the task itself must be equally important as well.
Hence, the correct answer is option 1.

Discuss the solution with Testfunda users.

Solution #285: (22-Jan-10)


For the tea seller to make Rs. 15 per day, he will have to sell all three glasses he makes.
This will happen in two cases either exactly three of the rooms order a glass of tea, or all four
rooms order a glass of tea.

Hence, option 3.

Discuss the solution with Testfunda users.

Solution #286: (23-Jan-10)


Options 1 and 4 are incorrect as the first blank cannot be 'presence'. In the presence of heat water
can be melted to drink in oredr to prevent dehydration.
Option 3 is eliminated as blanks 2 and 4 are incorrect. 'To prevent quick hypothermia' is incorrect
usage. 'Food energy to induce body heat' is also incorrect usage.
Hence, the correct answer is option 2.

Discuss the solution with Testfunda users.

Book 2- Question of the Day

322

www.TestFunda.com

The LEADER in Online Test Prep

Solution #287: (24-Jan-10)


We can see that f(x) is a polynomial function and hence continuous everywhere except possibly at x2
= 1, that is, at x = 1 and x = 1.

f(1) = 1

The function is continuous at x = 1

f (1) = 1

The function is not continuous at x = 1


There is only one point of discontinuity.
Graphically, this is shown as below.

Hence, option 2.

Discuss the solution with Testfunda users.


Book 2- Question of the Day

323

www.TestFunda.com

The LEADER in Online Test Prep

Solution #288: (25-Jan-10)


Option 1 is incorrect because 'fiction' is incorrectly used.
Option 3 is incorrect as 'mainly' is incorrectly used.
Option 4 is incorrect as 'characters' is incorrectly used as it should be singular.
Option 5 is incorrect as the usage in the main sentence incorrectly uses the word, 'characteristic'.
Hence, the correct answer is option 2.

Discuss the solution with Testfunda users.

Solution #289: (26-Jan-10)

We need to find (a, b, c) such that a + 2b + c 40


11b + 2b + c 40
13b + c 40
For b = 1 we get 27 values of c, for b = 2 we get 14 values of c and for b = 3 we get 1 value of c.
Thus there are 27 + 14 + 1 = 42 possible sets of (a, b, c).
Hence, option 2.

Discuss the solution with Testfunda users.

Book 2- Question of the Day

324

www.TestFunda.com

The LEADER in Online Test Prep

Solution #290: (27-Jan-10)


Options 1 and 3 are incorrect as they contain errors in parallelism.
Option 2 is incorrect as it does not complete the sentence.
Option 4 is incorrect as the word, 'physique' is incorrectly used.
Hence, the correct answer is option 5.

Discuss the solution with Testfunda users.

Solution #291: (28-Jan-10)

Squaring both sides, we have

2x + 2|x 1| = 1
If x > 1, we have |x 1| = x 1. Consequently, we get 2x + 2x 2 = 1, or x = 3/4. However, this is less
than 1, so there is no solution for x > 1.
For x < 1, we have |x 1| = 1 x. However, this gives an inconsistency 1 = 2, and hence there is no
solution for x < 1 either. There is no solution to this equation.
Hence, option 5.

Discuss the solution with Testfunda users.

Solution #292: (29-Jan-10)


A pride is a group of lions.
Similarly, an unkindness is a group of ravens.
Hence, the correct answer is option 3.

Discuss the solution with Testfunda users.


Book 2- Question of the Day

325

www.TestFunda.com

The LEADER in Online Test Prep

Solution #293: (30-Jan-10)


Putting x = 1, we get f(2) + g(0) = 1
Putting x = 1, we get f(0) + g(2) = 1
Equating the LHS of these two equations, we get f(0) g(0) = f(2) g(2)

Hence, option 2.

Discuss the solution with Testfunda users.

Solution #294: (31-Jan-10)


Both abscond and absquatulate means to run away secretly, especially from an open institution or to
avoid prosecution or punishment. The relationship is synonymous in nature.
Contrive means to bring about, succeed with difficulty. Stop means an end, halt; impediment. This is
not a synonymous relationship. This eliminates option 1.
Infernal means damned; underworld. Blessed means to sanctify. This is not a synonymous
relationship. This eliminates option 2.
Condemn means to blame, convict. Exonerate means to excuse, clear of responsibility or blame. This
is not a synonymous relationship. This eliminates option 4.
Pilfer and purloin both mean to steal, especially in small quantities.
Hence, the correct answer is option 3.

Discuss the solution with Testfunda users.

Book 2- Question of the Day

326

www.TestFunda.com

The LEADER in Online Test Prep

Solution #295: (01-Feb-10)


We have that,

Case 1:
X+Y=0

3x 9 + 4x 16 = 0

Case 2:
XY 1 = 0

The sum of possible values of x is

Hence, option 5.
Discuss the solution with Testfunda users.
Book 2- Question of the Day

327

www.TestFunda.com

The LEADER in Online Test Prep

Solution #296: (02-Feb-10)


The way the passage is structured clearly indicates that the inherent, core or real value of a book
would fit in blank 1. Options 2 and 4 can be eliminated.
In blank 2, sustained is incorrect usage in this context. This eliminates option 1
Hence, the correct answer is option 3.
Discuss the solution with Testfunda users.

Solution #297: (03-Feb-10)

Let X and Y be the midpoints of CD and AB respectively and extend AD and BC to meet at E.
As DAB = 40 and ABC = 50, EAB and ECD are right triangles.
X and Y are circumcentres of ECD and EAB, respectively.
EX = CX = DX = 500 cm and AY = EY = BY = 1005 cm
XY = EY EX = 1005 500 = 505 cm
Hence, option 4.
Discuss the solution with Testfunda users.

Book 2- Question of the Day

328

www.TestFunda.com

The LEADER in Online Test Prep

Solution #298: (04-Feb-10)


This question is about association. An interlude is a part of an opera.
A farce is an exaggerated comedy. Burlesque means farcical.
Ditty is a simple song and cadence means balanced rhythmic flow.
Vaudeville was a theatrical genre of variety entertainment and a revue is a type of multi-act popular
theatrical entertainment.
Option 3 seems to be the most suitable as a Canticle is a hymn taken from the bible which is sung
during vespers which means the evening prayer service in the Anglican or western Catholic church
amongst others.
Hence, the correct answer is option 3.

Discuss the solution with Testfunda users.

Solution #299: (05-Feb-10)


The only way we can have two value 3 coins the minimum distance apart is ...31213... .

If we repeat that pattern then the conditions are all satisfied. So the optimal configuration is
312131213... .

Since 2010 divided by 4 leaves remainder 2, the last two coins will be 3 and 1.
Total number of coins of value 3 = 1 + (2010/4)

= 503

Hence, option 3.

Discuss the solution with Testfunda users.

Book 2- Question of the Day

329

www.TestFunda.com

The LEADER in Online Test Prep

Solution #300: (06-Feb-10)


The association is based on them being antonyms. The question gives words which show two
extremes of a persons character.
The words used can be defined as:
Gaucherie : lack of social grace, sensitivity, or acuteness; awkwardness; crudeness; tactlessness.
Urbane: suave, cosmopolitan
Confident : sure of oneself and Clique: exclusive group
Astucious: cunning and Shenanigan: deceitful trick
Narcissistic : egoistic and Altruism: selflessness
Aghast: filled with shock and Annihilation: total destruction
Hence, the correct answer is option 3.

Discuss the solution with Testfunda users.

Book 2- Question of the Day

330

www.TestFunda.com

The LEADER in Online Test Prep

About TestFunda.com
TestFunda.com is an initiative by Enabilon Learning Private Limited. Enabilon is a sisterconcern of Zeus Learning, an e-learning solutions company. Products designed and created by
us are currently being used in thousands of US schools and colleges. Clients include some of the
worlds largest publishers and developers of educational software: Discovery Education,
Readers Digest, Pearson Education, etc.
TESTfunda.com is managed by five IIT-IIM Alumni. It has an 85+ member team of Educationists,
Instructional Designers and Systems Architects with vast experience in MBA Coaching and
Computer-Based Training and Testing. The TestFunda Team has 9 years of experience designing
and creating international award-winning eLearning courses.

Book 2- Question of the Day

331

www.TestFunda.com

You might also like